Download as pdf or txt
Download as pdf or txt
You are on page 1of 190

THANJAVUR MEDICAL COLLEGE

THANJAVUR

DEPARTMENT OF PHARMACOLOGY

RECORD NOTEBOOK
(as per NMC – Curriculum based Medical
Education guidelines)
From academic year 2020 onwards
THANJAVUR MEDICAL COLLEGE
THANJAVUR

Department of Pharmacology

REG No

Certified to be bonafide work done by


in Department of Pharmacology, Thanjavur Medical College during
the year 20___ to 20 ___

Professor of Pharmacology, Date:


Thanjavur Medical College, Thanjavur.

Examiner 1 Date:

Examiner 2 Date:
CONTENTS
Ex. Competency
Date Name of the experiment/exercise Page sign
No No
CLINICAL PHARMACY
1 PH 1.3 & 2.1a Oral solid and liquid drug formulations 2-5
2 PH 1.3 & 2.1b Parenteral and topical drug formulations 6-10
3 PH 2.2 Preparation of ORS 11
4 PH 2.4 Drug dosage calculation 12-14
5 PH 1.12 Drug dosage calculation – special situation 15-17
6 PH 2.3 Preparation of intravenous infusion 18-20
CLINICAL PHARMACOLOGY
PH 1.9,1.10a, Prescription writing – introduction, parts,
7 22-24
3.1a, & 5.7 importance and legal aspects
Prescription writing –OPC poisoning, Motion
8 PH 3.1b sickness, Acute asthmatic attack, Open angle 25-29
glaucoma, Hypertension with BPH
Prescription writing – Paracetamol poisoning,
9 PH 3.1c Acute attack on chronic migraine, Rheumatoid 30-34
arthritis, Acute Gout, COPD
Prescription writing – Hypothyroidism,
10 PH 3.1d Hyperthyroidism, Type I DM, Type II DM, 35-40
Osteoporosis, Contraception
Prescription writing – GTCS, Status epilepticus,
11 PH 3.1e 41-45
Acute schizophrenia, Parkinsonism, Depression,
Prescription writing – Angina pectoris, Acute
12 PH 3.1f myocardial infarction, Hypertension with 46-50
diabetes, CCF, Dyslipidemia
Prescription writing – Iron deficiency
13 PH 3.1g anaemia, Megaloblastic anaemia, Warfarin 51-55
poisoning, Duodenal ulcer, Constipation
Prescription writing - Typhoid fever, Urinary
tract infection, Category I pulmonary
14 PH 3.1h 56-60
tuberculosis, Intestinal amoebiasis, Herpes
zooster
15 PH1.10b&3.2 Prescription Audit 62-73
16 PH 3.5 & 3.7 P drug and essential medicine list preparation 75-81
17 PH 1.6 & 3.4 Pharmacovigilance – reporting of ADR 83-92
Critical appraisal of drug promotion literature &
18 PH 3.3 & 3.6 interaction with pharmaceutical representative 93-105
to get authentic information on drugs
19 PH 1.8 Drug interaction 107-111
CONTENTS
Ex.
Compt No Date Name of the experiment/exercise Page sign
No
CLINICAL PHARMACOLOGY -Contd
20 PH 4.1a OSPE – Loading of drug 113
21 PH 4.1b OSPE – Intramuscular drug administration 114
22 PH 4.1c OSPE – Intravenous drug administration 115
23 PH 4.1d OSPE – Subcutaneous drug administration 116
24 PH 4.1e OSPE – Intradermal drug administration 117
25 PH 4.1f OSPE – Eye drop application 118
26 PH 4.1g OSPE – Drug administration by Nebulizer 119
27 PH 4.1h OSPE – Drug administration by MDI and spacer 120
EXPERIMENTAL PHARMACOLOGY
CAL – Effect of drugs on Heart rate and blood
28 PH 4.2a 122-130
pressure in dogs
29 PH 4.2b CAL – Effect of drugs on Rabbit’s eye 131-136
30 PH 4.2c CAL – Effect of drugs on frog oesophagus 137-140
31 PH 4.2d CAL – Effect of drugs on Frog’s heart 141-144
32 PH 4.2e CAL – Effect of drugs on guinea pig ileum 145-148
COMMUNICATION SKILLS
PH 5.1, 5.2
33 Effective Doctor patient communication 150-156
&5.3
34 PH 5.4 Pharmaco-economics 157-162
Prescribing drugs with dependence liability, OTC
35 PH 5.5 & 5.6 163-174
drug abuse and its management
ANNEXURES
Definition of important terms related to
1 Annexure 1 175-176
pharmacology
2 Annexure 2 Latin terms commonly used in prescription 177
3 Annexure 3 Weights and measures in metric system 178
4 Annexure 4 Sterilisation methods 179-180
5 Annexure 5 Drug regulatory act and schedules 181
6 Annexure 6 WHO-UMC causality assessment scale 182
7 Annexure 7 Naranjo Causality assessment scale 183
CLINICAL PHARMACY

SPECIFIC LEARNING OBJECTIVE

At the end of this section, the learner shall be


able to:

1. Demonstrate understanding of the use of


various dosage forms (oral/local/parenteral;
solid and liquid)
2. Enumerate and identify the drug
formulations and drug delivery systems
3. Prepare the ORS rehydration solution from
ORS packets and explain its use
4. Demonstrate the appropriate setting up of an
intravenous drip in a simulated environment
5. Calculate the dosage of drugs using
appropriate formulae for an individual
patient and special situation

1
NMC- Competency No: PH 1.3 & 2.1a
Ex. No: 1 ORAL-SOLID AND LIQUID DRUG FORMULATIONS
A. Sustained release dosage forms:
1. Define sustained release dosage forms.
2. What are the advantages of sustained release dosage forms?
3. Mention different types of sustained release dosage forms with one example each.
4. Mention the use of sustained release haematinics.

1. sustained release dosage forms


These are solid dosage forms meant for internal use. They release the drug slowly
and continuously, thus maintaining a minimum effective concentration of the drug in
plasma for a prolonged time.

2. Advantages of sustained release dosage forms


Reduction in frequency of administration
Longer duration of action
Less adverse effects

3. Types of sustained release dosage forms


a.Multilayer coated tablets:
Tablets are coated with several layers of resins or gelatin with particular amount of drugs in
between. With degradation of each layer a portion of drug is released leading to long duration of
action. Example: Metoprolol XR

b.Controlled release capsules:


Multiple small coloured spheres containing active drug is placed inside the capsule. After the
dissolution of capsule layers, the coloured spheres are released. Colour of the sphere denotes the
rate of delivery of drugs. Example: Omeprazole capsules.

c.Oily depot injection:


The active drug ingredient is dissolved in oily material and administered intramuscular or
subcutaneous. The active drug is slowly released from the oily vehicle for longer duration. Example:
Medroxyprogesterone acetate injection

d.Biochemically modified solutions for injections.


Based on the chemical nature, the active drug can be precipitated after the administration in the
body. From the precipitate, the active drug will be released for a longer duration. Example: Insulin
glargine.

e.Transdermal patches:
The drug impregnated in adhesive patches, strips or as ointment applied on skin is utilized in some
cases to prolong drug action, e.g. GTN

4. Uses of sustained release hematinics


Used mainly in iron deficiency anemia because of long duration of action, increased
efficacy, reduced local irritation and release of the drug at favourable site

2
NMC- Competency No: PH 1.3 & 2.1a
Ex. No: 1 ORAL-SOLID AND LIQUID DRUG FORMULATIONS

B. Tablets:
1. Define tablets. What are the different types of tablets? Give one example for each.
2. Mention the advantages of enteric coated tablets over film coated tablet.
3. What are the advantages and disadvantages of effervescent tablets?
4. What are the advantages and disadvantages of sugar coated tablets?
5. What is meant by chewable tablet? Give one example.
1. Tablets:
Tablet is defined as a compressed unit solid dosage form containing medicaments with or without
excipients. Tablets are solid unit dosage form, prepared by compressing a drugs or a mixture of
drugs, with or without diluents.

Types of tablet:
Based on coating:
Uncoated tablets Example: Lithium carbonate tablets
Film coated tablet. Example: Cotrimoxazole tablets
Sugar coated tablets Example: Multivitamin tablets
Multilayer coating (sustained release) tablets: Metoprolol XR tablets
Based on action:
Dispersible tablets: Example: Aspirin DT
Sustained release tablets Example: Metoprolol XR
Osmotic tablets: Example: Prazosin osmotic tablets
Based on use:
Chewable tablet Example: Albendazole chewable tablet
Effervescent tablet Example: Ranitidine effervescent tablets
Sublingual tablets Example: Isosorbide dinitrate

2. Advantages of enteric coated over film coated tablet


Enteric coated tablets do not dissolve in mouth
EC tablets are non irritant and preserve enzyme activity in stomach
Active medications can be made to release at desired site with EC.
3. Advantages of Effervescent tablets:
More gentle action on patient stomach
Taste can be improved
Patient who find difficulty in swallowing solid formulation can be overcome
Disadvantage of effervescent tablets:
Size is larger and difficult for manufacturers
Shelf life is reduced
Special packing material is required

4. Sugar coated tablets - advantages:


Bitter and metallic taste will be masked
Protect the drug from environment
Enhances the appearance of the tablets, Drug details can be imprinted on tablets

Disadvantages: Size increases, cost increases and additional equipment for coating

5. Chewable tablet:
They are solid dosage form containing active medications that should be crushed
and chewed and then swallowed.
Example: albendazole 3
NMC- Competency No: PH 1.3 & 2.1a
Ex. No: 1 ORAL-SOLID AND LIQUID DRUG FORMULATIONS

C. Capsules:
1. Define capsule.
2. What are soft gelatin capsules? Give one example.
3. What are hard gelatin capsules? Give one example.
4. Mention the differences between soft and hard gelatin capsules

1. Capsule:
These are solid dosage forms in which one or more medicinal substances are enclosed
within gelatin polymer

2. Soft gelatin capsule:


They are prepared from gelatin in which glycerin or sorbitol has been added to render
the gelatin elastic or plastic like structures. Soft gelatin capsule are used to enclose the
liquid suspension.
Example: Vitamin E capsules, Nifedipine

3. Hard gelatin capsule:


They are prepared from gelatin without any additives. They are used to enclose the
powder or pellet medications
Example: Amoxycillin

4. Difference between soft and hard gelatin capsule

Soft gelatin capsule contains liquid medications while hard gelatin contains powder
Prone for microbial contamination is high in soft gelatin and low in hard gelatin
Shelf life is low for soft gelatin and high for hard gelatin capsules
Moist absorber is required for container having soft gelatin capsules to prevent adhesions
but not required for hard gelatin

4
NMC- Competency No: PH 1.3 & 2.1a
Ex. No: 1 ORAL-SOLID AND LIQUID DRUG FORMULATIONS

D. Syrup, Suspension, solution and Elixir:


1. What are the advantages and disadvantages of oral liquid formulation?
2. Define syrup and suspension. Mention two differences between them.
3. Define solution. What is meant by 1% solution?
4. Define dry syrup. What is the advantage of dry syrup over normal syrup?
5. What is meant by elixir? Give one example.
6. What is the volume of one teaspoon and one tablespoon?

1 What are the advantages and disadvantages of oral liquid formulations?


Advantages:
Masking of unpleasant taste in syrup and elixir
Easy administration for children and elderly who has difficulty in swallowing
Dose titration is easier rather than breaking the oral solid formulations
Disadvantages:
Mass is higher and requires large storage room
Less stability

2 Define syrup and suspension. Mention two differences between them.


Syrups are the liquid oral preparations in which the vehicle is concentrated aqueous solution of
sucrose/other sugar which provides sweet taste.
Suspensions are the liquid medicament containing insoluble solid substances which are
homogeneously distributed throughout the vehicle. It is simply the formulation in which the fine
solid particles are suspended in a liquid. Suspensions and gel are examples for colloids.

Syrup is non- colloid and suspension is colloid


Syrup is homogenous and does not form sediments while suspension does
Suspension should be shaken well before use while for syrup it is not required.

3. Solutions are liquid preparations with one or more chemical substances dissolved in a suitable
solvent or mixture of mutually miscible solvents. Example: Aripiprazole solution
In a solution, 1% indicates that 1 gram of solute is dissolved in 100 ml of solvent.

4. Dry syrup is a type of liquid oral formulation in which suitable solvents (water) should be
added to the container to convert the powder form into liquid before consumption. Dry syrup
has increased shelf life than normal syrup. Example: Amoxyclav drug syrup

5. Elixir is a sweetened liquid usually containing alcohol that is used in medication either for its
medicinal ingredients or as a flavouring. Example: Promethazine elixir

6. What is the volume of one teaspoon and one table spoon?


One teaspoon = 5 ml (Approx)
One table spoon = 15 ml (Approx)

5
NMC- Competency No: PH 1.3 & 2.1b
Ex. No: 2 PARENTERAL DRUG FORMULATIONS
A. Ampoule, Vial and Depot preparation:
1. Define injection. Mention the different routes of injection.
2. What are the advantages and disadvantages of parenteral route?
3. Mention two differences between vial and ampoule.
4. What are depot preparations? Give one example.
5. Mention the advantages of depot preparation.

1. Injection is a technique of administering a drug or other compounds using a needle and syringe.
The different routes of administration of drug by injection are
a.Intra venous b. subcutaneous c. intramuscular d. intradermal e. intrathecal
f. intraosseous g. intra arterial h. spinal/epidural i. intra articular j. retroorbital

2. advantages and disadvantages of parenteral route

Advantages:
Highest bioavailability (100%)
Accurate dose titration
Rapid onset of action of drugs
Can be administered for unconscious patients

Disadvantages:
Cannot be administered by all (requires special training)
Painful often
Can cause severe reactions occasionally

3. Ampoules are used to store small to medium volume of medication while vials are used
for medium to large volume of medications
Ampoules are single use but in vials medications can be withdrawn multiple times
Chances of contamination is high in vials while less in ampoules

4. Depot injections are the meant for sustained release of an active drug from the injection site
resulting in longer duration of action and hence avoids repeated administration.
Example: Medroxyprogesterone acetate

5. Mention the advantages of depot preparation.


Single administration can result in longer duration of action
Increased patient compliance

6
NMC- Competency No: PH 1.3 & 2.1b
Ex. No: 2 TOPICAL DRUG FORMULATIONS

B. Pessaries and suppository:


1. What is pessary? Give one example.
2. Mention the advantages of the pessary.
3. What is suppository? Give one example.
4. What are the advantages and disadvantages of suppository?

1.What is pessary? Give one example.


A pessary is a medical device inserted into the vagina, either to provide structural support, or
as a method of delivering medication. Example: Clotrimazole pessary

2.Mention the advantages of the pessary.


Provides local action of drugs and thereby avoids systemic side effects
Increased patient compliance
Single administration

3. Suppository is solid medical preparation in a roughly conical or cylindrical shape, designed


to be inserted into the rectum or vagina to dissolve.
Example: Diazepam, Paracetamol

4. Advantages of suppository
Can be administered in emergency situation for children for quicker action
Faster onset of action
Can be used in unconscious patient also

Disadvantages:
Cumbersome and can cause local irritation

7
NMC- Competency No: PH 1.3 & 2.1b
Ex. No: 2 TOPICAL DRUG FORMULATIONS

C. Ointment, Cream, Gel and Emulsions:


1. Define ointment, gel, and cream. Give an example for each.
2. What are the different types of ointment bases?
3. What are the advantages and disadvantages of ointment and cream?
4. What is gel? How does gel differ from the ointment?
5. Define emulsion. Mention their types.
6. What are the advantages of emulsion? Give an example.
7. Define Finger tip unit. What is the clinical importance of FTU?
1. An ointment is a homogeneous, viscous, semi-solid preparation, most commonly a greasy, thick oil (oil 80%
- water 20%) with a high viscosity, that is intended for external application to the skin or mucous
membranes. Example: Povidone iodine

Gels are semisolid preparations consisting of dispersion of small or large molecules in an aqueous liquid
vehicle rendered jelly like by the addition of jellying agent. Example: Lignocaine gel

Cream is a semisolid preparation containing mixture of medication, water, and fat (Base) that can be
spread easily over the hairy areas. Example: Clotrimazole

2. Hydrocarbon bases, e.g. hard paraffin, soft paraffin and microcrystalline wax.
Absorption bases like as wool fat, beeswax.
Water soluble bases.
Emulsifying bases like as emulsifying wax.

3. Ointment advantage: Used for dry scaly lesions, occlusion is high and drug penetration is good
Ointment disadvantage: Cannot be used for wet lesions, stains the dresses and messy
Cream advantage: Used for wet lesions, and hairy areas
Cream disadvantage: Local irritant effects can occur, less suitable for dry lesions

4. Gels are semisolid preparations consisting of dispersion of small or large molecules in an aqueous liquid
vehicle rendered jelly like by the addition of jellying agent. Example: Lignocaine gel.
Gel are suitable for wet lesions while ointments are suitable for dry lesions
Gels are good for oily skins while ointments are less suitable for it.

5. Emulsion is a type of colloid in which one liquid is dispersed finely in another liquid which is immiscible or
insoluble.
Types:
a.Oil in water emulsion
b.Water in oil emulsion

6. What are the advantages of emulsion?


Can be applied over a large surface area
Can be applied over hairy area without resistance
Easier to spread on the skin.
Example: Benzybenzoate

7. Finger tip Unit is an approximate measure for topical formulation administration. One finger
tip unit is the medication amount from the tip of the forefinger to the first interphayngeal crease which covers
approximately the twice the palm size lesion of that patient.
Clinical importance:
The quantity of cream in a fingertip unit varies with age: Adult male: 1 fingertip unit provides 0.5 g. Adult female:
1 fingertip unit provides 0.4 g.
8
NMC- Competency No: PH 1.3 & 2.1b
Ex. No: 2 TOPICAL DRUG FORMULATIONS

D. Lotion and liniments:


1. Define lotions and liniments.
2. What are the differences between lotion and liniments?
3. Mention one advantage and disadvantage of liniment.
4. Mention the uses of calamine lotion and turpentine liniment.

1.Define lotions and liniments


Lotions are the dermatological topical formulations which is a suspension of solid medicaments in aqueous
vehicle that can be applied without rubbing.
Liniment is also a dermatological topical preparation called as balms or heat rubs with higher viscosity than
lotions and is rubbed in to create friction.

2.What are the differences between lotion and liniments?


Lotion works by spreading over the skin while liniments work by rubbing on the skin
Lotion covers larger area while liniment covers small area
Viscosity is less for lotion while higher for liniments.

3.Mention one advantage and disadvantage of liniment.


.Advantage: Quick onset of action
Disadvantage: can induce irritation at the site of rubbing

4.Mention the uses of calamine lotion and turpentine liniment.


Calamine lotion: To alleviate skin itching in Urticaria, eczema, sun burns etc.,
Turpentine liniment: To reduce pain in joints, muscle spasm etc.,

9
NMC- Competency No: PH 1.3 & 2.1b
Ex. No: 2 TOPICAL DRUG FORMULATIONS

E. Aerosol:
1. What is an aerosol dosage form? Give one example.
2. Mention the advantages of aerosol.
3. What is meant by nebulisation? Give example.
4. What is meant by metered dose inhaler? Give one example.
5. What is meant by spacer? Mention its advantages.

1.What is an aerosol dosage form? Give one example


Aerosol dosage forms are a type of topical dosage form meant for respiratory tract drug delivery to treat
diseases of lungs without systemic side effects.
Example: Nebulizer, Dry powder inhaler, Metered dose inhaler.

2.Mention the advantages of aerosol.


Topical application results in lesser side effects
Rapid onset of action
Often used in emergency situations
Self administration by patients

3.What is meant by nebulisation? Give example


Nebulization is a technique of delivering an intended drug as aerosol by fine water droplets which carries
drug particular to the lungs with a help of nebulizer. Example: Salbutamol for asthma

4.What is meant by metered dose inhaler? Give one example


Metered dose inhaler is device used for topical drug delivery to lungs in which the drug is delivered as a
short burst of aerosol by the help of propellent. Each press delivers a drug in a prespecified amount and
hence it is called as metered dose.
Example: Salbutamol MDI.

5.What is meant by spacer? Mention its advantage.


Spacer is an accessory device used along with MDI for children to provide a space for aerolised drug to
stay without escaping into the environment. MDI requires proper mouth and hand co-ordination. This hand
mouth co-ordination is absent in infants and children. To avoid this, spacer is used along with MDI.

10
NMC- Competency No: PH 2.2
Ex. No: 3 PREPARATION OF ORS

A. Composition of ORS:

New formula WHO ORS (2002)


Content Concentration achieved per litre
Sodium chloride - 2.6g; Na+ --- 75mM; Cl- --- 65mM
Potassium chloride - 1.5g; K+ --- 20mM
Trisodium citrate - 2.9g; Citrate -- 10mM
Glucose - 13.5g; Glucose -- 75mM
Water - 1L Total osmolarity = 245mosm/L

B. Steps to prepare ORS (Quantity litre)

1. Wash your hands throughly and dry it. Put the contents of the ORS packet in a clean
container.

2. Check the packet for directions and add the amount of clean water as indicated (Too little or
too much water will alter the target concentration and make the diarrhoea worse).

3. Only water should be added. Water should be clean (boiled and cooled water is preferrable)

4. Stir well,

5. Feed it to the child from a clean cup (Do not use a bottle)

C. Instruction regarding use of ORS

1. ORS is indicated for mild to moderate dehydration and not in severe dehydration (IV fluids is
used for severe dehydration). Do not use the powder if it has turned into a yellow-brownish
sticky substance.

2. Once prepared, the solution must be used within 24 hours. Add water only. Do not add ORS
to milk, soup, fruit juice or any soft drinks. Do not add sugar.

3. The ORS solution should be covered and not kept for more than 24 hours, due to the risk of
bacterial contamination.

4. ORS should be used as long as dirrhoea and signs of dehydration persist. If case of vomiting
while consuming ORS, stop ORS for 10 min and then resume at a slower rate (very small,
frequent, amounts); do not stop rehydration.

5. If child is exclusively breastfed, then it should be continued in addition to the ORS


supplementation. Breastfeeding should not be stopped.

6. ORS should be given immediately after an episode of diarrhoea/vomiting or every 30 minutes


to one hour interval.

7. Dose of ORS = 75ml X weight of child in Kg for the first four hours. If the weight is not known
that dose of ORS can be given according to age:
50 -100ml/h for child < 4 months; 100 - 175ml/hr for child 4m - 1year;
175 - 225ml/hr for 1 -2y; 225 - 350ml/hr for child 2 - 5yr;
500 - 1L/hr for adults. 11
NMC- Competency No: PH 2.4
Ex. No: 4 Drug dosage calculation

1. A patient with 50 kg body weight is to be given i.v thiopentone sodium in a dose of 3 mg/kg.
If the strength of the solution is 2.5%. What is the volume of the drug required?

Thiopentone dose = 3mg/kg. =3 x 50. =150 mg.

2.5% means 2500mg in 100ml. 25 mg = 1 ml.

150mg is present in 150/25


= 6ml

volume of drug required = 6 ml.

2. A patient with weighing 60Kg has to be given diazepam at a dose of 0.3 mg/kg. If the
strength of the solution is 5 mg/ml, what is the volume of the drug you will administer?

Dose of diazepam = 0.3mg/kg.


=0.3 x 60.
=18 mg.

Strength of the solution is 5 mg in 1ml


18mg = 18×1 /5

volume of drug required = 3.6ml.

12
NMC- Competency No: PH 2.4
Ex. No: 4 Drug dosage calculation
3. In a patient with acidosis, 50mEq of sodium bicarbonate has to be given by intravenous
route. Calculate the amount (ml) of this solution required in this patient. Sodium
bicarbonate available as 7.5% solution .

The molecular weight of NaHCO3 : Na = 23, H = 1, C=12, and O=16


Thus molecular weight of NaHCO3 is equals to 23+1+12+(16X3) = 84
84 g of sodium bicarbonate in one litre of water given 1M solution
similarly, 84mg of NaHCO3 in IL water given ImM solution (1meq solution)

50 meq means 50 X 84mg = 4200 mg

7.5% solution means 7.5g in 100ml


so, 7.5 X 1000 mg = 7500mg in 100ml

Since, 7500mg is present in 100ml,

4200mg is present in (100/7500) X 4200 = 56ml

4. Calculate the amount of fluids to be infused to a 50 kg patient with 20% burns.

The amount of fluids to be infused =4ml x body weight x %of burns

= 4 x 50 x 20 = 4000 ml.

The preferred fluid is Ringer lactate.


Half of the amount to be given in first 8hrs.
The remaining to be given in next 16 hrs.

13
NMC- Competency No: PH 2.4
Ex. No: 4 Drug dosage calculation

5. Prepare a solution of dopamine to infuse at a rate of 4 microgram/kg/min in a 50 kg man,


suffering from hypotension following myocardial infarction. 1 ml of solution in ampoule
contains 200 mg of dopamine. (Note: Indian iv set gives 16 drops/ml.)

Dopamine dose = 4 µg /kg. =4 x 50 =200µg /mt


5ml of dopamine contains 200mg.
Add 1 ampoule of dopamine in 495 ml of normal saline.
500ml contains 200mg.
1ml contains 200 = 0.4 mg
500
=400 microgram.
Dose required for the patient is 200microgram/mt.
1ml = 16 drops = 400 microgram.
Hence to infuse 200 microgram/mt , 8 drops/mt

6. Prepare a solution of lignocaine to be infused at a rate of 1mg / minute for a case of


premature ventricular depolarization. Note: Lignocaine is available as 2% solution in 25 ml
vial and Indian iv set delivers 16 drops/ml

Lignocaine is available as 2%solution in 25ml vial. 2gm in 100ml


2000mg in 100ml.
1ml = 2000 = 20 mg
100
25ml = 25 x20 = 500mg.
To prepare a solution to give 1mg/mt
add 1 vial (25ml) of lignocaine in 500ml of 5%dextrose.
500 ml contains 500mg. 1 ml contains 1 mg .
Required rate of infusion is 1mg/mt . that is 1ml/mt. 1ml = 16drops/mt,
Hence to infuse 16drops/mt

14
NMC- Competency No: PH 1.12
Ex. No: 5 Drug dosage calculation – Special situation

1. A 2 year old child brought with the complaint of fever. Weight of the child is unknown.
Calculate the dose of paracetamol required for this child using Young’s and Dilling’s formula
(Adult dose of paracetamol is 500 mg three times a day)

Young's formula: Dose for child = (Age of child) X Adult dose


(Age of child+12)
= (2)/(14) X 500
= 71 mg
approximately 70 mg three times a day

Dilling's formula: Dose for child = (Age of child) X Adult dose


20
= (2)/(20) X 500
= 50 mg
approximately 50 mg three times a day

Dilling's formula is applicable for children between 4 - 20 years. Hence for


this child young's formula is appropriate.

2. A 60 year old male weighing 54Kg whose serum creatinine is 2.4 mg/dl is suffering from
oesophageal candidiasis. What is the recommended dose of fluconazole for this patient
(Standard dose is 200mg i,v once daily).

Cockroft Gault's formula for estimation of creatinine clearance

CrCL for men = (140 - age in years) X weight in Kg


72 X Serum creatinine
= (140 - 60 ) X 54
72 X 2.4
= (80 X 54) / (72 X 2.4) = 4320/172.8
= 25 ml/min

Recommended dose reduction in reduced SrCL value is


50 - 70 ml/min - 70% of standard dose
30 - 50 ml/min - 50% of standard dose
10 - 30 ml/min - 30% of standard dose
5 - 10ml/min - 20% of standard dose

Thus, for this patient the recommended dose of fluconazole is


30 % of 200 mg = 60mg i.v Once daily

15
NMC- Competency No: PH 1.12
Ex. No: 5 Drug dosage calculation – Special situation

3. A 4year old baby is suffering from severe dehydration. The weight of the baby is 18 kg.
Calculate the amount of fluid to be given
1. Bolus infusion: Intravenous Ringer Lactate infusion or normal saline is the ideal choice for bolus
infusion in severe dehydration
Bolus dose: 20ml/kg = 20*18 = 360 ml stat

2. Deficit calculation: % of dehydration X body weight X 1000ml


in case of severe dehydration usually >10% of dehydration will be seen
therefore, deficit fluid = 10% X 18 X 1000ml = 1800 ml

3. Maintenance fluid: (as per Holliday - Segar calculation method)


For total 24 hours the maintenance fluid required = 100 ml/kg/day for child weighing less than 10 Kg
+ 50ml/Kg/day for next 8 kg=1400ml
Thus, for first 8 hours:
1/2 of the deficit fluid + 1/3 of the maintenance fluid to be infused
= 1/2 (1800) + 1/3 (1400)
= 900 + 466.7
= 1366.7 ml to be infused in 8 hours

For next 16 hours:


1/2 of the deficit fluid + 2/3 of the maintenance fluid to be infused
= 1/2 (1800) + 2/3 (1400)
= 900 + 933.3
= 1833.3 ml to be infused in 16 hours

4. A 60 year old male patient weighing 60 Kg and height of 180cm requires cisplatin for
treating lung cancer. Calculate the dose of cisplatin required for this patient using Mostellar
formula. (Standard dose of cisplatin is 20mg/m2/day)

Mostellar formula for calculation of body surface area

BSA (m2) = [(Height in cm X Weight in Kg)/3600]1/2

= [(180 X 60)/3600]1/2

= [10800/3600]1/2

= [3]1/2 = 1.732 m2

Dose of cisplatin = 20mg/m2/day

= 20*1.732/day

= 34.64 mg/day

16
NMC- Competency No: PH 1.12
Ex. No: 5 Drug dosage calculation – Special situation

5. A 40 year old female with heart failure has been prescribed T. digoxin 0.125mg OD. After
one month, the therapeutic drug monitoring was done and the measured steady state
serum concentration of digoxin is 0.23 ng/ml rather than predicted level of 0.7ng/ml. What
is the new dose of digoxin to be prescribed for this patient for every 24 hours?

New dose = (predicted steady state concentration ) X previous dose


(measured steady state concentration)

= (0.7 X 0.125) / 0.23

= 0.38 mg OD/day

Thus, digoxin should be increased from 0.125mg OD to 0.38mg OD


for this patients.

6. A 25 year old female weighing 60Kg with hypothyroidism has been prescribed T.
Levothyroxin (0.16mcg/Kg/day) for the past 5 years and her TSH is under control. Now she
is in first trimester of pregnancy. What is dose of T. Levothyroxine to be prescribed now?
Recommended dose of T. Levothyroxin = 1.6mcg/kg/day

Thus the patient is currently taking


= 1.6*60/day
= 96 mcg

T. Levothyroxine should be increased during pregnancy to meet the increased


demand in pregnancy by 30%

Hence, the newer dose of T. Levothyroxine for this patient is


New dose = Old dose + 30% of old thyroxine dose
= 96 + 96(0.3)
= 96 + 28.8
= 124.8 mcg
Approximately 125mcg OD

17
NMC- Competency No: PH 2.3
Ex. No: 6 Preparation of intravenous infusion drip

A. Mention the parts of intravenous infusion setup and its use:

Bag spike - to pierce the fluid container

Access port - to administer additional drugs into the


fluids

Murphy's chamber - to visualize flow of iv fluid


to visualize the drop rate setting

Back check valve - to prevent iv fluid travelling up

Slider or roller clamp - to adjust the rate of IV fluid flow

Hub or port for cannula - to connect to the venous


cannula

B. Mention the fluids that are used for intravenous infusion. Mention two compatible and
non-compatible drugs for each fluid.

Name of the IV fluid Compatible drug Incompatible drug

1. Normal Saline (0.9%) 1. Phenytoin 1. Aminophylline


2. Dopamine 2. Amphotericin-B

2. Dextrose 5% 1. Aminophylline 1. Ampicillin


2. Amphotericin-B 2. Magnesium sulfate

3. Ringer Lactate (RL) 1. Norepinephrine 1. Diazepam


2. Dopamine 2. Nitroglycerine

Other fluids: DNS, Half normal saline, 3%saline; 25% dextrose

C. What are the advantages and disadvantages of intravenous infusion drip?


Advantages - Immediate action of drugs in emergency situation,
100% bioavailability
Fine tuning of drug dosage

Disadvantages: Requires skilled assistant help,


Cannot be used for long time duration
Thrombophlebitis, local site reactions, air embolism

18
NMC- Competency No: PH 2.3
Ex. No: 6 Preparation of intravenous infusion drip

D. Set up an aminophylline drip for a known asthmatic 12 years male child weighing 35 kg.
Rate of infusion is 1mg/kg/hour.

Rate of infusion is 1mg/Kg/hour


Hence the dose of aminophylline for this patient = 35mg/hour = 0.58mg/minute

Aminophylline is available as 250mg/10ml ampoule

Mix one ampoule of aminophylline in 490ml of 5% dextrose

500 ml of 5%Dextrose contains 250mg of aminophylline


500 ml = 250mg
each ml contains = 0.5mg

Required aminophylline per minute = 0.58mg


Therofore, 0.58mg is present in 1.16ml

16 drops = 1 ml
To achieve 1.16ml, drop should be kept as 18 drops/minute

E. Set up an insulin drip for 40 year old female weighing 60kg with diabetic ketoacidosis. Her
blood glucose is 500mg/dL. The rate of infusion of insulin is 0.1U/kg/hour. When will you
discontinue this infusion?
Rate of infusion of insulin = 0.1U/Kg/ hour
This patient required --> 0.1U*60 = 6U/hour = 0.1U/minute

50U of regular insulin should be mixed with 500 ml of Normal Saline

Each ml contains = 0.1U of insulin

Target insulin delivery = 0.1U/minute

16 drops = 1ml

Therefore, 16 drops per minute should be set

Infusion should be reduced to 2-3U/hour when blood sugar level falls below 300mg/dL and
maintained till the patient becomes fully conscious. The infusion should be stopped when
routine s.c therapy with insulin is initiated.

19
NMC- Competency No: PH 2.3
Ex. No: 6 Preparation of intravenous infusion drip

F. Set up a dopamine drip for 40 year male weighing 60Kg with hypovolemia and oliguria. The
rate of infusion of dopamine for renal vasodilation is 2mcg/kg/min. When will you increase
the rate of infusion?

Rate of dopamine for renal vasodilation = 2mcg/kg/min

Required dopamine for this patient = 2*60 = 120mcg/min

Dopamine is available as 5ml ampoule and each ml contains 40mg


Thus 200mg of dopamine is present in 5ml of dopamine ampoule

Mix 5ml of dopamine in 495ml of Normal Saline or 5%dextrose


500 ml of NS contains 200 mg of dopamine
thus each ml contains = 0.4mg = 400mcg

Required dose = 120mcg/min


Each ml delivers 400mcg; Thus, 0.3ml per minute delivers 120mcg

16 drops = 1ml
Therefore, 0.3ml will be delivered with 5 drops approx

G. Set up a nitroglycerin drip for a 50 year female weighing 60kg with hypertensive emergency
with pulmonary edema. The rate of infusion of nitroglycerin is 5mcg/kg/min.

Recommended rate of infusion of nitroglycerine = 5mcg/min

Nitroglycerine is available as 10mg/2ml ampoule

Mix 2ml of nitroglycerine in 50ml of Normal saline

100 ml of NS contains 10mg and hence each ml contains = 0.1mg = 100mcg

Required rate of nitroglycerine is 5mcg/min

each ml = 16 drops
To deliver 5mcg/min, drops should be set to 0.8 drops (apprx 1drop/min)

Dose can be increased if adequate response is not produced

20
PRESCRIPTION WRITING

OBJECTIVES

At the end of the prescription writing exercises,


the learner shall be able to:

1. Describe the nomenclature of drugs (generic


or brand)
2. Describe parts of a correct, complete, and
legible generic prescription

3. Write a rational, correct and legible generic


prescription for a given condition and
communicate the same to the patient.

4. Understand the legal aspects of prescription

21
NMC- Competency No: PH 1.9, 1.10a, 3.1a, & 5.7
Ex. No: 7 Prescription writing- Introduction, parts, importance and
legal aspects

A. Fill the blank cells in the following table

S.No Chemical name Generic Name Brand name


1 Acetyl salicylic acid Aspirin Dispirin
2 Aminobenzyl penicillin Ampicillin Ampisyn
3 azidothymidine Zidovudine Zidovir
4 Diphenylhydantoin Phenytoin Dilantin
5 N-acetyl-paramino-phenol Paracetamol Dolo

B. Mention the importance of following parts of prescription


a. Doctor’s detail
A valid prescription should contain the details of the prescriber (doctor)
with his/her full name, degree and specilization, registration and contact number
It is required as an authentication to dispense the medication by pharmacists.
b. Patient’s detail
A valid prescription should contain the details of patient's name, age, sex and address
It is required for proper identification of patients with age and trace the patient in case of
any future follow up or medication errors.
c. Diagnosis
Prescription without mentioning the proper diagnosis is not valid

d. Inscription
Rx is present in superscription. Inscription follows superscription and it should contain the details of
the prescribing medications (Formulation, Generic name in capital letter, strength, frequency
duration of regimen, and timing of medication.
e. Subscription
Subscription contains instruction to the pharmacists how the compounds are dispensed (e.g.
mix and make 10 capsules, make a solution etc.,). Modern prescription does not have these
instructions as formulations are available readymade.
f. Signature
Prescription is not valid and considered illegal without signature and medication cannot
be dispensed by the pharmacist.

g. Advice
Advice section deals with the instruction to patient/attending nurse regarding the possible
use of that formulation, parameters to be monitored, possible ADRs to be forseen etc.,
h. Number of refill
Number of refill section is important for pharmacist to dispense the medication to the patient
For example is number of refill is mentioned as 3 for T. Diazepam 5 mg H.s for 10 days, then
pharmacist can dispense 10 tablets three times once previous 'quota' of 10 tablets are over.

C. Define E-prescribing

E-prescribing is a digital form of communication that helps a prescriber, such as a family doctor or
other healthcare provider, to send a prescription order directly to a pharmacy.

22
NMC- Competency No: PH 1.9, 1.10a, 3.1a, & 5.7
Ex. No: 7 Prescription writing- Introduction, parts, importance and
legal aspects

D. A 25 year old female Ms. Kannagi was prescribed T. Doxylamine OD for one month for
morning sickness by Dr. M. Thamarai (D.G.O) whose registration number is AB1234. Due
to illegible handwriting in the prescription, the pharmacist has provided C. Doxycycline
for the patient.
a. Is the doctor liable medico legally in this scenario? Explain.
b. Is the pharmacist liable medico legally in this scenario? Explain
c. What are the legal punishments that can be given for the concerned person in
this scenario?
a. Yes the doctor is medico legally liable in this scenario. The judiciary system of India mandates
healthcare providers to use generic drug names in CAPITAL letter with legible handwriting.
Thus the above mentioned healthcare provider is medico legally liable

b. Yes, the pharmacist is also medico legally liable. He fails to communicate with the healthcare
provider in case of any queries or clarification which would avoid such kind of scenarios.

c. The IPC that apply (but not limited to below) are as follow
1. IPC Section - 336: Act endangering life or personal safety of others.
2. IPC Section - 337: Causing hurt to any person by doing any act so rashly or negligently as to endanger
human life (Fine 500/-, 6 month imprisonment or both)
3. IPC Section - 338: Causing hurt to any person by doing any act so rashly or negligently as to endanger
human life (Fine 1000/-, 24 month imprisonment or both)

When patients get admitted and stays in hospital for more than 20days because of miscarriage due to
teratogenic effect of Doxycycline then IPC Section - 319 and 320 also apply (Hurt and grievous hurt)

E. A 19 year old male Mr. Karthick was prescribed T. PARACETAMOL for simple fever by Dr.
M. Mullai M.D (General medicine) whose registration number is AB4321. The
pharmacist issued T. DOLO-B (contains Paracetamol with ibuprofen). Mr. Karthick
developed Steven Johnson syndrome due to ibuprofen.
a. Is the doctor liable medico legally in this scenario? Explain.
b. Is the pharmacist liable medico legally in this scenario? Explain

a. No. The doctor is NOT medico legally liable in this scenario. The judiciary system of India mandates
healthcare providers to use generic drug names in CAPITAL letter with legible handwriting and he
prescribed accordingly.

b. Yes, the pharmacist is also medico legally liable. He voluntarily provided different drug combination
without consulting the healthcare provider.

c. The IPC that apply (but not limited to below) are as follow
1. IPC Section - 336: Act endangering life or personal safety of others.
2. IPC Section - 337: Causing hurt to any person by doing any act so rashly or negligently as to endanger
human life (Fine 500/-, 6 month imprisonment or both)
3. IPC Section - 338: Causing hurt to any person by doing any act so rashly or negligently as to endanger
human life (Fine 1000/-, 24 month imprisonment or both)

When patients get admitted and stays in hospital for more than 20 days because of Steven Johnson
Syndrome; then IPC Section - 319 and 320 also apply (Hurt and grievous hurt)
When patients dies due to SJS, then IPC Section

23
NMC- Competency No: PH 1.9, 1.10a, 3.1a, & 5.7
Ex. No: 7 Prescription writing- Introduction, parts, importance and
legal aspects

F. A 45 year old male Mr. Praveen was prescribed inj. MEDZOL for peptic ulcer by Dr. M.
Murugan M.S (General Surgery) whose registration number is AD3256. The pharmacist
issued inj. MEDZOL (contains midazolam). Patient went for respiratory depression and
coma after administration of the drug.
a. Is the doctor liable medico legally in this scenario? Explain.
b. Is the pharmacist liable medico legally in this scenario? Explain
a. Yes. The doctor is medico legally liable in this scenario. The judiciary system of India mandates
healthcare providers to use generic drug names in CAPITAL letter with legible handwriting but he
prescribed in brand name that led to this incident.

b. Yes, the pharmacist is also medico legally liable. He did not consult with the physician in case of
doubtful scenarios.

c. The IPC that apply (but not limited to below) are as follow
1. IPC Section - 336: Act endangering life or personal safety of others.
2. IPC Section - 337: Causing hurt to any person by doing any act so rashly or negligently as to
endanger
human life (Fine 500/-, 6 month imprisonment or both)
3. IPC Section - 338: Causing hurt to any person by doing any act so rashly or negligently as to
endanger
human life (Fine 1000/-, 24 month imprisonment or both)

When patients get admitted and stays in hospital for more than 20 days because of Coma; then IPC
Section - 319 and 320 also apply (Hurt and grievous hurt). When patients dies then IPC Section
304A also applies.

G. A 5 year old male Mr. Kumar was prescribed Lotion. BENZYL-BENZOATE for Scabies by
Dr. Raja M.D (Dermatology) whose registration number is AM1258. The child was
brought to the clinic by his father. Dr. Raja has explained the procedure of application of
lotion to the father and written the same in the prescription. Next day, the child was
brought to the clinic with seizures as the mother administered the medication orally.
a. Is the doctor liable medico legally in this scenario? Explain.
b. Who is liable medico legally in this scenario? Why?
c. How can this issue be avoided in future?
a. No. The doctor is NOT medico legally liable in this scenario. The judiciary system of India mandates
healthcare providers to use generic drug names in CAPITAL letter with legible handwriting and he
prescribed in similar way.

b. The father of the child is medico-legally available in this scenario as he failed to give proper
communication to the mother regarding the use of lotion despite doctor's explanation.

c. The primary care taker of the child should be present while prescribing formulations with special
instructions

24
NMC- Competency No: PH 3.1 b
Ex. No: 8 Prescription writing
1. Prescribe suitable drugs for a 40 year old farmer weighing 65 kg suffering from Organo-
Phosphorous compound poisoning. Provide necessary instruction to the nurse.

Doctor’s Name: Date:


(With degree)
Name of the patient:
Registration No: Age:
Address: Sex:
Address:
Phone No: Weight:

DIAGNOSIS: Organo-phosphorous compound poisoning

Rx:

Rx:
General measures:
1.Access airway for any obstruction and secure airway patency
(intubation if necessary)
2.Remove the clothes and wash the whole body to remove OPC
3.Secure intravenous cannula

Specific measures:
1.Inj. ATROPINE SULPHATE 1.2mg i.v stat repeat after 3 – 5 mins
till pupil dilates to maximum and heart rate maintains above 120bmpm
2.Infusion. PRALIDOXIME 2g (30mg/Kg ) I.V for 15 minutes –
repeat the same after one hour and continue as BD till recovery.

Advice to the patient/Instruction to the Nurse:

1.Continuous PR, SpO2, ECG, and RR monitoring


2.Monitor blood pressure every 15 minutes.
3.In put/output chart maintenance
4.Monitor for convulsions.

Signature with date:

25
NMC- Competency No: PH 3.1 b
Ex. No: 8 Prescription writing
2. Prescribe suitable drugs for a 35 year female weighing 55 kg who is suffering from
motion sickness

Doctor’s Name: Date:


(With degree)
Name of the patient:
Registration No: Age:
Address: Sex:
Address:
Phone No: Weight:

DIAGNOSIS: Motion sickness

Rx:

Transdermal patch. SCOPOLAMINE (1mg/72hr) application


behind the ear 4 – 12 hour before journey

Or

T. PROMETHAZINE 25mg 30 – 60 minutes before journey

Advice to the patient/Instruction to the Nurse:

1.Apply the patch behind the pinna of the ear


2.Do not cut the patch and use
3.Drugs cause sedation and hence driving should be avoided

Signature with date:

26
NMC- Competency No: PH 3.1 b
Ex. No: 8 Prescription writing

3. Prescribe suitable drugs for a 5 year male child weighing 16 kg who is suffering from
acute attack of asthma.

Doctor’s Name: Date:


(With degree)
Name of the patient:
Registration No: Age:
Address: Sex:
Address:
Phone No: Weight:

DIAGNOSIS: Acute attack of asthma

Rx:
Neb. SALBUTAMOL 2.5 mg + IPRATROPIUM BROMIDE 0.25 mg in 3 ml of
Saline for 20 minutes (repeat the dose if adequate response is not obtained)

Inj.HYDROCORTISONE HEMISUCCINATE 100 mg i.v stat


(followed by 100 mg infusion 8th hourly)

Inj.SODIUM BICARBONATE50 meq/L in normal saline i.v

General measures:
Nasal humidified oxygen
SpO2 monitoring
Electrolyte and blood pressure monitoring
Antibiotics if indicated

Maintenance:
Tab. PREDNISOLONE1 mg/kg o.dp.c for 7 days
MDI. Salbutamol (50 mcg/puff) s.o.s
MDI.BUDESONIDE (100 mcg/puff)o.d for 1 month

Advice to the patient/Instruction to the Nurse:

1.Avoid exposure to dust and smoke.


2.Take the inhalational drugs regularly.
3.Always keep the inhaler with you and don’t adjust the dose by yourself.
4.Review after one month

Signature with date:

27
NMC- Competency No: PH 3.1 b
Ex. No: 8 Prescription writing

4. Prescribe suitable drugs for a 45 year female weighing 62 kg suffering from open angle
glaucoma .

Doctor’s Name: Date:


(With degree)
Name of the patient:
Registration No: Age:
Address: Sex:
Address:
Phone No: Weight:

DIAGNOSIS: Open angle Glaucoma

Rx:

Eye drop. LATANOPROST (0.005%) one drop in the affected eye OD (0-0-1)
for 30 days

Or
Eye drop. BIMATOPROST (0.01%) one drop in the affected eye OD (0-0-1)
for 30 days

Advice to the patient/Instruction to the Nurse:

1.Apply pressure over the medial canthus immediately after eye drop application
2.Take the eye drop regularly without missing
3.If you develop any severe eye pain or irritation report immediately
4.Keep the eyedrop in cool place and not under direct sunlight

Signature with date:

28
NMC- Competency No: PH 3.1 b
Ex. No: 8 Prescription writing

5. Prescribe suitable drugs for a 75 year male weighing 65 kg who is suffering from
hypertension with benign prostate hypertrophy.

Doctor’s Name: Date:


(With degree)
Name of the patient:
Registration No: Age:
Address: Sex:
Address:
Phone No: Weight:

DIAGNOSIS: HYPERTENSION WITH BENIGN PROSTATE HYPERTROPHY

Rx:

T. TAMSULOSIN 0.4mg OD (0-0-1) p.c for 30 days

T. FINASTERIDE 1mg OD (0-0-1) p.c for 30 days

Advice to the patient/Instruction to the Nurse:

1.Avoid stand up immediately after getting up from bed.


2.If you develop excessive dizziness or headache, report immediately.

Signature with date:

29
NMC- Competency No: PH 3.1 c
Ex. No: 9 Prescription writing

1. Prescribe suitable drugs for a 25 year male weighing 65 kg who is suffering from
paracetamol poisoning.

Doctor’s Name: Date:


(With degree)
Name of the patient:
Registration No: Age:
Address: Sex:
Address:
Phone No: Weight:

DIAGNOSIS: ACUTE PARACETAMOL POISONING

Rx:

General measures:
Gastric lavage and cannulation of vein
Oral administration of activated charcoal after stomach wash
Airway, circulation and breathing measures.

Specific measures:
Infusion . N-ACETYLCYSTEINE (150mg/Kg) in 200ml of 5% Dextrose over
15 minutes followed by 50mg/kg for 4 hours followed by 100mg/kg for 16 hours.

(or)

Tablet. N-ACETYLCYSTEINE (140mg/Kg) stat followed by 17 doses of 70mg/kg


N-ACETYLCYSTEINE every four hours.

Advice to the patient/Instruction to the Nurse:

•Monitoring of vitals (BP, PR, SpO2 and continuous ECG)


•Fluid Input/Output record
•Monitoring of LFT, RFT, and serum acetaminophen levels.
•Monitoring of blood glucose

Signature with date:


30
NMC- Competency No: PH 3.1 c
Ex. No: 9 Prescription writing

2. Prescribe suitable drugs for a 25 year female weighing 65 kg who is suffering from acute
attack on chronic migraine.

Doctor’s Name: Date:


(With degree)
Name of the patient:
Registration No: Age:
Address: Sex:
Address:
Phone No: Weight:

DIAGNOSIS: ACUTE ATTACK ON CHRONIC MIGRAINE PATIENT

Rx:

To terminate the acute attack:

Inj. SUMATRIPTAN 6mg s.c stat


Inj. METOCLOPRAMIDE 10mg i.m stat
Tab. PARACETAMOL 500mg TDS p.c for 2 days

Prophylaxis:

Tab. PROPRANOLOL 40mg BD p.c for one month

(or)

Tab. TOPIRAMATE 25mg h.s p.c for one month

Advice to the patient/Instruction to the Nurse:

1.Avoid exposure to strong fragrance, bright light and loud sounds.


2.Take the preventive tablets regularly and do not adjust the dose by yourselves
3.If you develop any dizziness or excessive sleepiness report to me immediately.

Signature with date:


31
NMC- Competency No: PH 3.1 c
Ex. No: 9 Prescription writing

3. Prescribe suitable drugs for a 50 year male weighing 55 kg who is suffering from
rheumatoid arthritis.

Doctor’s Name: Date:


(With degree)
Name of the patient:
Registration No: Age:
Address: Sex:
Address:
Phone No: Weight:

DIAGNOSIS: RHEUMATOID ARTHRITIS

Rx:

To relieve pain:

Tab. PREDNISOLONE 10mg OD (1-0-0) p.c for 5days


Tab. ACECLOFENAC POTASSIUM 100 mg b.d (1-0-1) p.c for 5 days
Cap. OMEPRAZOLE 20mg (0-1-0) a.c for 5 days

For disease modulation:

Tab. METHOTREXATE5 mg once weekly p.c for one month


Tab. FOLIC ACID 1mg OD (0-0-1) for one month

Advice to the patient/Instruction to the Nurse:

1.Take the methotrexate tablet once weekly not daily


2.Avoid consuming alcohol as the above tablets also have risk of liver injury
3.Do not adjust the dose of methotrexate by yourselves.
4.If you develop excessive fatigue, fever, yellow discoloration of eyes and
skin, report to me immediately.

Signature with date:


32
NMC- Competency No: PH 3.1 c
Ex. No: 9 Prescription writing

4. Prescribe suitable drugs for a 50 year male weighing 60 kg who is suffering from acute
gout.

Doctor’s Name: Date:


(With degree)
Name of the patient:
Registration No: Age:
Address: Sex:
Address:
Phone No: Weight:

DIAGNOSIS: Acute Gout

Rx:

To relieve pain:

Tab. PREDNISOLONE 10mg OD (1-0-0) p.c for 5 days


Tab. NAPROXEN 750mg stat followed by 250mg TDS for 5 days
Cap. OMEPRAZOLE 20mg (0-1-0) a.c for 5 days

For prophylaxis of acute flare:

Tab. ALLOPURINOL 100 mg OD (0-0-1) p.c for one month

Advice to the patient/Instruction to the Nurse:

1.Avoid purine rich foods like red meat, seafoods, spinach, and cauliflower.
2.Take low fat milk and vitamin C rich fruits like amla, oranges, etc.,
3.Avoid consuming alcohol and take plenty of water
4.If you develop any skin rashes. Oral ulcers or redness over body report immediately.

Signature with date:


33
NMC- Competency No: PH 3.1 c
Ex. No: 9 Prescription writing

5. Prescribe suitable drugs for a 65 year male weighing 60 kg who is suffering from Chronic
Obstructive Pulmonary disease. He has been smoking for the past 32 years.

Doctor’s Name: Date:


(With degree)
Name of the patient:
Registration No: Age:
Address: Sex:
Address:
Phone No: Weight:

DIAGNOSIS: Acute exacerbation of Chronic Obstructive Pulmonary disease

Rx:
To terminate acute exacerbation:

•Nebulizer. SALBUTAMOL (2.5mg) + IPRATROPIUM BROMIDE (500 mcg) 2.5ml


solution diluted in normal saline to 3ml STAT and repeat every hour if needed

•Tab. PREDNISONE (0.5mg/kg) 1-0-0 p.c for 5 days

•Humidified Oxygen supplementation


•Appropriate antibiotic coverage if required

For prevention of acute attack:

MDI. IPRATROPIUM BROMIDE (20mcg) + SALBUTAMOL (100mcg) one puff


TDS for one month

Advice to the patient/Instruction to the Nurse:


1.Stop smoking and do the breathing exercises regularly

2.Avoid exposure to air polluted areas and smokes

3.If you develop tremors, palpitation and dizziness report to me immediately

Signature with date:


34
NMC- Competency No: PH 3.1d
Ex. No: 10 Prescription writing

1. Prescribe suitable drugs for a 25 year female weighing 75 kg who is suffering from
hypothyroidism.

Doctor’s Name: Date:


(With degree)
Name of the patient:
Registration No: Age:
Address: Sex:
Address:
Phone No: Weight:

DIAGNOSIS: Hypothyroidism

Rx:
Tab. L-Thyroxin 50 mcg OD on empty stomach for 1 month
( dose adjusted by clinical response and serum TSH level )

Advice to the patient/Instruction to the Nurse:


1.Always take the tablet regularly in the empty stomach in morning.
2.Avoid taking cabbage, radish, cauliflower and turnip.
3.Take iodized salt regularly and iodine rich foods like sea foods.
4.Always keep the tablet container closed tightly and should be kept in the dark
place

Signature with date:


35
NMC- Competency No: PH 3.1d
Ex. No: 10 Prescription writing

2. Prescribe suitable drugs for a 35 year female weighing 45 kg who is suffering from
hyperthyroidism (diffuse toxic goitre). She is also pregnant (8 weeks).

Doctor’s Name: Date:


(With degree)
Name of the patient:
Registration No: Age:
Address: Sex:
Address:
Phone No: Weight:

DIAGNOSIS: HYPERTHYROIDISM

Rx:
Tab. Prophylthiouracil 50 mg TDS for 15 days

Advice to the patient/Instruction to the Nurse:

1.Take the tablet regularly and review after 15 days


2.Do thyroid profile and liver function test while coming for review
3.If there is any skin rashes and joint pain inform to me immediately
4.Continue the other medications and check up instructed by your obstetrician

Signature with date:


36
NMC- Competency No: PH 3.1d
Ex. No: 10 Prescription writing

3. Prescribe suitable drugs for a 12 year male child weighing 25 kg who is suffering from
type I diabetes mellitus.

Doctor’s Name: Date:


(With degree)
Name of the patient:
Registration No: Age:
Address: Sex:
Address:
Phone No: Weight:

DIAGNOSIS: TYPE I DIABETES MELLITUS

Rx:
Inj.Human insulin 0.4-0.8 U/kg/Day ie 20 U/day in 2 divided doses

10U-0-10U S.C for 30 days

Advice to the patient/Instruction to the Nurse:


1.Advised to take the diabetic diet and do exercise at least 1 hr .
2.Keep the food ready and inject subcutaneously with insulin pen or needle
and alter the site of injection every time .
3.Keep the candy in hand if there is any symptoms of hypoglycaemia
(giddiness,blackouts,tremors and excessive sweating )
4.Check the blood sugar every fortnightly and HBA1C every 3 months ,

Signature with date:


37
NMC- Competency No: PH 3.1d
Ex. No: 10 Prescription writing

4. Prescribe suitable drugs for a 55 year female weighing 50 kg who is suffering from type
II diabetes mellitus.

Doctor’s Name: Date:


(With degree)
Name of the patient:
Registration No: Age:
Address: Sex:
Address:
Phone No: Weight:

DIAGNOSIS: TYPE II DIABETES MELLITUS

Rx:
Tab. Metformin 500 mg BD for 30 days

Advice to the patient/Instruction to the Nurse:

1.Advised to take the diabetic diet and do exercise at least 1 hr .


2.Keep the food ready and take the tablet half an hour before each meal
(breakfast & dinner )
3.Check the blood sugar every fortnightly and HBA1C every 3 months .
4.Avoid smoking and alcohol.

Signature with date:


38
NMC- Competency No: PH 3.1d
Ex. No: 10 Prescription writing

5. Prescribe suitable drugs for a 65 year female weighing 50 kg who is suffering from post
menopausal osteoporosis.

Doctor’s Name: Date:


(With degree)
Name of the patient:
Registration No: Age:
Address: Sex:
Address:
Phone No: Weight:

DIAGNOSIS: Post menopausal osteoporosis

Rx:
Tab. Aceclofenac 10 mg TDS for 1 week

Tab. Risedronate 35 mg /week every morning on empty stomach for 15 days


Tab.Calcium gluconate 500 mg once daily for 15 days
Tab. Alfacalcidol 1 mcg once daily for 15 days

Advice to the patient/Instruction to the Nurse:


1.To take one risedronate sachet (5mg) and mix it with full glass of water
on empty stomach
2.Not to lie down immediately after drinking the risedronate
3.Not to repeat the pain killer without prescription
4.Rest to the joints and review after 15 days

Signature with date:


39
NMC- Competency No: PH 3.1d
Ex. No: 10 Prescription writing

6. Prescribe suitable drugs for a 28 year female weighing 60kg who requires oral
contraceptive pills to avoid pregnancy. She is anaemic and mother of one child.

Doctor’s Name: Date:


(With degree)
Name of the patient:
Registration No: Age:
Address: Sex:
Address:
Phone No: Weight:

DIAGNOSIS: Oral contraception

Rx:
Tab. Norgestrol 75 mcg daily irrespective of the menstrual cycle for 3 months

Advice to the patient/Instruction to the Nurse:


1.Take the tablet regularly and ignore the spotting /bleeding .
2.Periods may be irregular and inform if there is missed period of 2 consecutive cycle.
3.Inform me immediately if there is presence of breast discomfort and leg cramps.
4.Take iron and folic acid tables and check for blood haemoglobin level periodically

Signature with date:


40
NMC- Competency No: PH 3.1e
Ex. No: 11 Prescription writing
1. Prescribe suitable drugs for a 25 year female weighing 50 kg who is suffering from
Generalized Tonic Clonic Seizures.

Doctor’s Name: Date:


(With degree)
Name of the patient:
Registration No: Age:
Address: Sex:
Address:
Phone No: Weight:

DIAGNOSIS: Generalized Tonic Clonic Seizures

Rx:
Tab. Carbamazepine 200 mg BD 1-0-1 for 1 month

Advice to the patient/Instruction to the Nurse:

1.Take the tablet regularly without missing the tablet .


2.if there is appearance of rashes and vomiting report me immediately
3.advice about the oral contraceptive inefficacy
4.ask the attender to count the number of seizures per week and dose
optimization can be done according to the seizure frequency

Signature with date:

41
NMC- Competency No: PH 3.1e
Ex. No: 11 Prescription writing

2. Prescribe suitable drugs for a 21 year male weighing 60 kg who is suffering from status
epilepticus.

Doctor’s Name: Date:


(With degree)
Name of the patient:
Registration No: Age:
Address: Sex:
Address:
Phone No: Weight:

DIAGNOSIS: Status epilepticus

Rx:
Inj LORAZEPAM 4 mg I.V bolus over 10 minutes
(if not controlled repeat the dose after 10 minutes)
If not controlled with Lorazepam:
Inj FOSPHENYTOIN 100 – 150 mg/min I.V maximum 1000 mg under the
Continuous EEG monitoring

General measures:
Maintain patent airway to prevent aspiration
Assisted ventilation to ensure adequate ventilation
Maintenance of fluid and electrolyte
To monitor:
1.Cardiac output and heart rate
2.Serum electrolyte – Sodium and Potassium levels
3.Blood pressure and EEG

Advice to the patient/Instruction to the Nurse:


1.Ask the attender to keep a diary to count the number of seizures per day
2. Dose optimization can be done according to the seizure frequency
3.Once convulsions are controlled switch over to oral anti-convulsant therapy
4.Ask the patient to come for regular visit to the clinic periodically

Signature with date:


42
NMC- Competency No: PH 3.1e
Ex. No: 11 Prescription writing

3. Prescribe suitable drugs for a 25 year female weighing 50 kg who is suffering from acute

schizophrenia.

Doctor’s Name: Date:


(With degree)
Name of the patient:
Registration No: Age:
Address: Sex:
Address:
Phone No: Weight:

DIAGNOSIS: Schizophrenia

Rx:
Tab. Risperidone 2mg OD for 30 days
Tab. Clonazepam 5 mg BD for 30 days

Advice to the patient/Instruction to the Nurse:


1.Advice to take the tablet regularly and dose adjustment can be done
according to the response
2.Avoid the precipitating factors like sleeplessness and stress
3.Monitor the weight , BP and cholesterol level periodically
4.If there is any appearance of rashes report me immediately

Signature with date:


43
NMC- Competency No: PH 3.1e
Ex. No: 11 Prescription writing

4. Prescribe suitable drugs for a 75 year male weighing 60 kg who is suffering from
Parkinsonism.

Doctor’s Name: Date:


(With degree)
Name of the patient:
Registration No: Age:
Address: Sex:
Address:
Phone No: Weight:

DIAGNOSIS: Parkinsonism.

Rx:
Tab. Syndopa 110 mg (carbidopa 10 mg+levodopa 100 mg) OD for 30 days

Advice to the patient/Instruction to the Nurse:


1.To monitor BP, heart rate, cardiac output
2.If there is nausea, vomiting, dizziness, fainting attacks to report me immediately
3.Avoid taking multi vitamins along with the above medication

Signature with date:


44
NMC- Competency No: PH 3.1e
Ex. No: 11 Prescription writing

5. Prescribe suitable drugs for a 25 year female weighing 50 kg who is suffering from
depression.

Doctor’s Name: Date:


(With degree)
Name of the patient:
Registration No: Age:
Address: Sex:
Address:
Phone No: Weight:

DIAGNOSIS: Depression

Rx:
Tab. Sertraline 25 mg BD for 15 days
Tab. Clonazepam 10 mg OD for 30 day

Advice to the patient/Instruction to the Nurse:

1. To take the tablet regularly after food.


2. Drug effect starts only after 7- 10 days of therapy .
3.If there is nausea, loose motions and epistaxis report me immediately
4.ask the attender to monitor the patient periodically for mood changes.

Signature with date:


45
NMC- Competency No: PH 3.1f
Ex. No: 12 Prescription writing

1. Prescribe suitable drugs for a 55 year male weighing 85 kg who is suffering from angina
pectoris. He is a smoker and alcoholic for 25 years.

Doctor’s Name: Date:


(With degree)
Name of the patient:
Registration No: Age:
Address: Sex:
Address:
Phone No: Weight:

DIAGNOSIS: ANGINA PECTORIS

Rx:
Tab. NITROGLYCERINE 0.5 mg sublingually (repeat if needed after 3-5 mins)

Maintenance:

Tab.ISOSORBIDE DINITRATE 10 mg tds(1-1-1) p.c X 15 days

SR. Tab.METOPROLOL 25 mg o.d (1-0-0)p.c X 15 days

Tab.ASPIRIN 75 mg o.d(0-1-0) p.c X 15 days

Tab. ATORVASTATIN 40 mg o.d(0-0-1) p.c X 15 days

Advice to the patient/Instruction to the Nurse:


1.Stop smoking
2.Avoid consuming high salt and high fatty diet
3.Avoid strenuous exercises
4.If you develop excessive headaches, fatigue, giddiness report immediately

Signature with date:


46
NMC- Competency No: PH 3.1f
Ex. No: 12 Prescription writing

2. Prescribe suitable drugs for a 60 year male weighing 70 kg who is suffering from acute
myocardial infarction. He is a smoker and alcoholic for 30 years.

Doctor’s Name: Date:


(With degree)
Name of the patient:
Registration No: Age:
Address: Sex:
Address:
Phone No: Weight:

DIAGNOSIS: Acute Myocardial infarction

Rx:
General measures:
1.Continuous nasal humidified oxygen
2.Monitor Pulse, BP, ECG, Respiratory rate

Specific:

Inj. MORPHINE SULPHATE5mg i.v stat


Tab. ASPIRIN 325 mg stat followed by 325 mg o.d
Tab. CLOPIDOGREL 300 mg stat followed by 75 mg o.d
Infusion STREPTOKINASE 1.2 MU in 100 ml of saline i.v over 1 hour
Inj. ENOXAPARIN 1 mg/kg s.c b.d for 7 days

Maintenance (For 1 month)

Tab. ISOSORBIDE DINITRATE 10 mg b.d (1-0-1) p.c


Tab. ASPIRIN325 mg o.d (0-1-0) p.c
Tab. METOPROLOL 25 mgb.d (1-0-1) p.c
Tab. ATORVASTATIN 80 mg o.d (0-0-1) p.c
Cap. OMEPRAZOLE 20 mg o.d (1-0-0) a.c

Advice to the patient/Instruction to the Nurse:


1.Liquid diet on the first day
2.Soft diet with no added salt and cholesterol on the second day
3.Stool softeners on the third day

Signature with date:


47
NMC- Competency No: PH 3.1f
Ex. No: 12 Prescription writing

3. Prescribe suitable drugs for a 60 year female weighing 60 kg who is suffering from
hypertension with type II diabetes.

Doctor’s Name: Date:


(With degree)
Name of the patient:
Registration No: Age:
Address: Sex:
Address:
Phone No: Weight:

DIAGNOSIS: hypertension with type II diabetes mellitus

Rx:

T. ENALAPRIL 5mg O.D 0-0-1 for 30 days


T. Metformin 500mg 1 – 0 -1 for 30 days

Or
T. Telmisartan 20mg O.D 0-0-1 for 30 days
T. Metformin 500mg 1 – 0 -1 for 30 days

Advice to the patient/Instruction to the Nurse:

1.Avoid consuming high salt foods like pickle, dry fish etc.,
2.If you develop any dry cough dizziness, headache report immediately
3.Reduce salt intake to less than 3 g per day.

Signature with date:


48
NMC- Competency No: PH 3.1f
Ex. No: 12 Prescription writing

4. Prescribe suitable drugs for a 70 year male weighing 60 kg who is suffering from grade I
congestive cardiac failure.

Doctor’s Name: Date:


(With degree)
Name of the patient:
Registration No: Age:
Address: Sex:
Address:
Phone No: Weight:

DIAGNOSIS: Congestive Cardiac Failure

Rx:
SR. Tab. METOPROLOL 25 mg o.d x 15 days (p.c)
Tab. ENALAPRIL 2.5 mg b.d X 15 days (p.c)
Tab. CHLORTHALIDONE 25 mg o.d X 15 days (p.c)

Advice to the patient/Instruction to the Nurse:

1.Restricted Physical activity


2.Reduce salt intake in your diet (5-6 g per day)
3.Maintain ideal body weight
4.Review after 15 days

Signature with date:


49
NMC- Competency No: PH 3.1f
Ex. No: 12 Prescription writing
5. Prescribe suitable drugs for a 40 year male weighing 75 kg who is suffering from
dyslipidemia with elevated cholesterol, VLDL and LDL levels. His triglycerides and HDL
are in normal range.

Doctor’s Name: Date:


(With degree)
Name of the patient:
Registration No: Age:
Address: Sex:
Address:
Phone No: Weight:

DIAGNOSIS: Dyslipidemia

Rx:

T. ATORVASTATIN 20 mg 0-0-1 for 30 days

Advice to the patient/Instruction to the Nurse:


1.Avoid oily food
2.Consume flax seed, sunflower or safflower oil
3.Take high fiber diet
4.Perform moderate exercises daily for 45 minutes.

Signature with date:

50
NMC- Competency No: PH 3.1g
Ex. No: 13 Prescription writing

1. Prescribe suitable drugs for a 28 year female weighing 52 kg who is suffering from iron
deficiency anaemia. (Hemoglobin = 9.5 g/dL)

Doctor’s Name: Date:


(With degree)
Name of the patient:
Registration No: Age:
Address: Sex:
Address:
Phone No: Weight:

DIAGNOSIS: Iron deficiency anemia

Rx:
C. Ferrous sulphate 325 mg OD for 30 days along with food

Chewable T. albendazole 400mg 1 OD h.s after food x 1 tablet

Advice to the patient/Instruction to the Nurse:

1.Take iron rich foods like jaggery, pomegranate, drumstick leaves, liver, eggs
2.Chew the albendazole tablet to fine powder and then swallow
3.Avoid taking milk and iron tablet together

Signature with date:


51
NMC- Competency No: PH 3.1g
Ex. No: 13 Prescription writing

2. Prescribe suitable drugs for a 28 year female weighing 52 kg who is suffering from
megaloblastic anemia.

Doctor’s Name: Date:


(With degree)
Name of the patient:
Registration No: Age:
Address: Sex:
Address:
Phone No: Weight:

DIAGNOSIS: Megaloblastic anemia

Rx:

Inj CYANOCOBOLAMINE 1000 mcg i.m daily for 1 week


then weekly once for one month
and then monthly once for lifetime

Tab. FOLIC ACID 5 mg daily X 6 months

Advice to the patient/Instruction to the Nurse:


1.Take plenty of animal products like red meat, egg yolk, liver etc
2.Vegetarians can take cheese, soy milk and fortified cereals

Signature with date:


52
NMC- Competency No: PH 3.1g
Ex. No: 13 Prescription writing

3. Prescribe suitable drugs for a 30 year female weighing 55 kg who is suffering from
warfarin overdose. Her INR value is 7.8. No clinical sign of bleeding is present

Doctor’s Name: Date:


(With degree)
Name of the patient:
Registration No: Age:
Address: Sex:
Address:
Phone No: Weight:

DIAGNOSIS: Warfarin Toxicity

Rx:

Stop warfarin for 2 – 3 days

If clinical sign of bleeding present


Tablet. Vitamin K1 2.5mg OD once

Advice to the patient/Instruction to the Nurse:

Monitor for signs of bleeding (bleeding gums, hematemesis, malena, hematuria)


Monitor INR every 6 to 12 hours once

Signature with date:


53
NMC- Competency No: PH 3.1g
Ex. No: 13 Prescription writing

4. Prescribe suitable drugs for a 35 year male weighing 52 kg who is suffering from
duodenal ulcer.

Doctor’s Name: Date:


(With degree)
Name of the patient:
Registration No: Age:
Address: Sex:
Address:
Phone No: Weight:

DIAGNOSIS: Duodenal Ulcer

Rx:

Cap. OMEPRAZOLE 20 mg o.d a.c for 1 month

Advice to the patient/Instruction to the Nurse:

1.Take meals at regular intervals.


2.Avoid spicy foods and stop alcohol and smoking.
3.Avoid taking self medications like analgesic

Signature with date:


54
NMC- Competency No: PH 3.1g
Ex. No: 13 Prescription writing

5. Prescribe suitable drugs for a 75 year female weighing 60 kg who is suffering from
constipation. She is a known Ischemic heart disease patient on medication.

Doctor’s Name: Date:


(With degree)
Name of the patient:
Registration No: Age:
Address: Sex:
Address:
Phone No: Weight:

DIAGNOSIS: Acute constipation in known ischemic heart disease patient

Rx:

To treat acute constipation:

T. Bisacodyl 5 mg tablet 0 – 0 – 2 (h.s) after food for 2 days

To prevent constipation:
Powder. ISPAGHULA 3g 0 – 0 – 1 (B.D) after food daily

Advice to the patient/Instruction to the Nurse:

1.Do not Take the Ispaghula powder as dry powder directly.


Mix with warm water or milk or fruit juices and consume

2.Avoid taking drugs along with Ispaghula powder

3.Consume fiber rich foods like guava, papaya etc.,

Signature with date:


55
NMC- Competency No: PH 3.1h
Ex. No: 14 Prescription writing

1. Prescribe suitable empirical drugs for a 25 year female weighing 50 kg who is suffering
from typhoid fever.

Doctor’s Name: Date:


(With degree)
Name of the patient:
Registration No: Age:
Address: Sex:
Address:
Phone No: Weight:

DIAGNOSIS: Typhoid fever

Rx:

Inj. CEFTRIAXONE 2g i.v b.d for 2 days and 2g i.v od till fever subsides
or

T. AZITHROMYCIN 500mg 1-0-0 on empty stomach for 7 days

or

T. CIPROFLOXACIN 750mg b.d (1-0-1) p.c for 7 days

T. PARACETAMOL 500mg s.o.s

Advice to the patient/Instruction to the Nurse:

1. Avoid taking carrot juices. Take plenty of fruit juices and tendernut
2. Maintain person hygienic measures. Wash the hands properly before food and after
using restroom
3. In case of any severe abdominal pain, report immediately.

Signature with date:


56
NMC- Competency No: PH 3.1h
Ex. No: 14 Prescription writing

2. Prescribe suitable empirical drugs for a 30 year female weighing 60 kg who is suffering
from urinary tract infection.

Doctor’s Name: Date:


(With degree)
Name of the patient:
Registration No: Age:
Address: Sex:
Address:
Phone No: Weight:

DIAGNOSIS: Urinary tract infection - uncomplicated

Rx:

T. COTRIMOXAZOLE 160/800mg b.d for 3 days

or

T. NITROFURANTOIN 100mg b.d for 5 days

syrup. ALKALINE CITRATE 10ml in 100ml of water three times a day

Advice to the patient/Instruction to the Nurse:

1. If you develop any rash or allergic reaction, report immediately


2. Drink plenty of water and grapefruit juice
3. Maintain toilet hygienic measures.

Signature with date:


57
NMC- Competency No: PH 3.1h
Ex. No: 14 Prescription writing

3. Prescribe suitable drugs for a 45 year male weighing 48 kg who is suffering from
pulmonary tuberculosis. He was treated already for the same one year ago.

Doctor’s Name: Date:


(With degree)
Name of the patient:
Registration No: Age:
Address: Sex:
Address:
Phone No: Weight:

DIAGNOSIS: Pulmonary tuberculosis - relapse

Rx:

Intensive phase:
FDC. tablet H/R/Z/E 75/150/400/275 mg 3 tablets daily for 2 months
Inj. Streptomycin 15mg/Kg for i.v o.d for 2 months

followed by FDC. tablet H/R/Z/E 75/150/400/275 mg 3 tablets daily for 1 month

Continuation phase:

FDC. tablet H/R/E 75/150/275 mg 3 tablets daily for 2 months

Tablet. PYRIDOXINE 10mg O.D daily p.c 0-0-1 for 6 months

Advice to the patient/Instruction to the Nurse:


1. If you develop yellow discoloration of urine or eyes, numbness in hands or legs, and
difficulty in breathing report immediately
2. Avoid spitting in public places. Always wear mask while coughing
3. Take plenty of protein rich foods like mutton, liver, eggs and soya

Signature with date:


58
NMC- Competency No: PH 3.1h
Ex. No: 14 Prescription writing

4. Prescribe suitable drugs for a 25 year male weighing 60 kg who is suffering from
intestinal amoebiasis.

Doctor’s Name: Date:


(With degree)
Name of the patient:
Registration No: Age:
Address: Sex:
Address:
Phone No: Weight:

DIAGNOSIS: Intestinal amoebiasis

Rx:
T. METRONIDAZOLE 400mg t.d.s (1-1-1) p.c for 7 days
T. DILOXANIDE FUROATE 500mg t.d.s (1-1-1) p.c for 7 days

Advice to the patient/Instruction to the Nurse:

1. If you develop rash or allergy, report immediately


2. Avoid taking food in hotels and restaurants
3. Always follow good hygienic measures . Wash your hands before food intake and
after use of restrooms.

Signature with date:


59
NMC- Competency No: PH 3.1h
Ex. No: 14 Prescription writing

5. Prescribe suitable drugs for a 50 year diabetic female weighing 55 kg who is suffering
from herpes zoster. She is already on diabetic medication.

Doctor’s Name: Date:


(With degree)
Name of the patient:
Registration No: Age:
Address: Sex:
Address:
Phone No: Weight:

DIAGNOSIS: HERPES ZOSTER in a diabetic patient

Rx:
T. ACYCLOVIR 800 mg five times a day p.c for 5 days
T. BRUFEN 400mg p.c s.o.s

Ointment. ACYCLOVIR 5% cream topical application twice daily for 5 days

Advice to the patient/Instruction to the Nurse:


1. If you develop tremors, lethargy, loss of balance report immediately
2. Take the drugs for five days and do not stop in between without consultation.
3. Avoid contact with young children.
4. Continue the diabetic medication and ensure that glucose levels are under control.

Signature with date:


60
PRESCRIPTION AUDIT

OBJECTIVES

At the end of the prescription audit exercises,


the learner shall be able to:

1. Understand the importance of medication


error

2. Identify the errors in the prescription and


correct the prescription appropriately by
selecting rationale drugs

3. Suggest ways in improving prescribing format

61
NMC- Competency No: PH 1.1 & 3.2
Ex. No: 15 Prescription criticism and rewriting

1. Criticize the following prescription and rewrite with rationale drugs.

Doctor’s Name: Dr. Ajay Date:


Patient’s Name: Mr. Kumar
Registration No: 5625421 Age: 55 Sex: Male
Address: Eswari Nagar, Thanjavur Address:
Phone No: Weight: 55 kg

DIAGNOSIS: Upper respiratory infection with peptic ulcer

Rx:
T. omez 200 mg TDS after food
C. Erythromycin 500mg TDS after food for 7 days

Advice to the patient/Instruction to the Nurse:


ajay
Signature

Criticism:

1. Degree of doctor, phone number and date are missing.


2. Address of the patient is missing
3. Formulation of omeprazole is wrong
4. Name of the drugs are mentioned in brand name and not capitalized
5. Advices are not given
6. Omeprazole bioavailability is reduced by food. So it should be given as 20 mg once daily
before food. The dose mentioned is wrong
7. Erythromycin produces mild to severe epigastric pain. Instead Amoxycillin can be used.

62
Corrected Prescription:

fill the other details

Doctor’s Name: Date:


(With degree)
Name of the patient:
Registration No: Age:
Address: Sex:
Address:
Phone No: Weight:

DIAGNOSIS: Upper respiratory tract infection with peptic ulcer

Rx:

C. OMEPRAZOLE 20mg O.D a.c for 7 days


C. AMOXYCILLIN 500mg TDS p.c for 5 days

Advice to the patient/Instruction to the Nurse:

1. Avoid spicy foods and take plenty of water


2. Perform steam inhalation often with plain hot water

Signature with date:

63
NMC- Competency No: PH 1.1 & 3.2
Ex. No: 15 Prescription criticism and rewriting

2. Criticize the following prescription and rewrite with rationale drugs.

Doctor’s Name: Dr. Kumar M.D (Gen Med) Date: 15.08.2021


Patient’s Name: Mr. Kathir
Registration No: Age: 25 Sex: Male
Address: Eswari Nagar, Thanjavur Address:
Phone No: +91-745214587 Weight:

DIAGNOSIS: Anaphylactic shock after penicillin administration

Inj. Adrenaline 0.5ml (1/100) i.v stat


Tab. Pheniramine 250mg stat

Advice to the patient/Instruction to the Nurse:


1. Avoid penicillin injections in future.
Signature

Criticism:

1. Registration number of the doctor, address and weight of the patient are missing
2. The symbol Rx is missing
3. Signature of the doctor is missing
4. Injection.Adrenaline is given 0.5cc intramuscularly (1/1000 sol) in anaphylactic shock and iv route is
not indicated unless shock is immediately life threatening.
5. The dose of Pheniramine is very high. It should be administered intramuscularly in the dose 22.5mg
in anaphylactic shock.
6. Drug names are not capitalized.

64
Corrected Prescription:

Fill the other details

Doctor’s Name: Date:


(With degree)
Name of the patient:
Registration No: Age:
Address: Sex:
Address:
Phone No: Weight:

DIAGNOSIS: Anaphylactic shock after penicillin administration

Rx:

Injection. ADRENALINE 0.5cc (1/1000 solution) im. statum


Injection. HYDROCORTISONE 100mg iv. statum
Injection. PHENIRAMINE 1cc (22.5mg) im. statum

Advice to the patient/Instruction to the Nurse:

1. Monitor the vitals - BP, HR and RR


2. Avoiid penicillin and cephalosporin related drugs in future.

Signature with date:

65
NMC- Competency No: PH 1.1 & 3.2
Ex. No: 15 Prescription criticism and rewriting

3. Criticize the following prescription and rewrite with rationale drugs.

Doctor’s Name: Dr. A. Akash Date: 15.08.2021


Patient’s Name: Mrs. Thamarai
Registration No: Age: 25 Sex: Female
Address: Address:
Phone No: +91-6655887744 Weight: 50Kg

DIAGNOSIS: NSAID induced gastritis / 14th week of pregnancy

Rx:
Tab. Misoprostol 200mcg qid for 5 days
Gel. Antacid 30ml three times a day after food.

Advice to the patient/Instruction to the Nurse:


1. akash
Signature

Criticism:

1. Degree of the doctor, address and registration number of doctor are missing
2. Address of the patient is missing
3. Name of the drugs is not Capitalized.
4. Advice to the patient is missing
5. Misoprostol should not be given to pregnant mother as Prostaglandin will increase uterine
contraction and induce abortion. Safe alternative drugs like ranitidine can be given.
6. Antacids reduce absorption of ranitidine if taken concurrently .So antacids should be taken 2
hours after drug

66
Corrected Prescription:

Fill the other details

Doctor’s Name: Date:


(With degree)
Name of the patient:
Registration No: Age:
Address: Sex:
Address:
Phone No: Weight:

DIAGNOSIS: NSAID induced gastritis/ 14th week of pregnancy

Rx:

Tablet. RANITIDINE 150 mg orally twice daily x 7 days.


Gel. ANTACID 2 tsp thrice daily 2 hours after Ranitidine x 7 days

Advice to the patient/Instruction to the Nurse:

1. Avoid spicy food


2. Avoid taking analgesics (pain killer) without consultation

Signature with date:

67
NMC- Competency No: PH 1.1 & 3.2
Ex. No: 15 Prescription criticism and rewriting

4. Criticize the following prescription and rewrite with rationale drugs.

Doctor’s Name: Dr. A. Akilan M.D., D.M (Neuro) Date:


Patient’s Name: Mr. Paneer
Registration No: 6688547 Age: 18 Sex: Male
Address: Address:
Phone No: Weight:

DIAGNOSIS: Uncontrolled GTCS with Phenytoin allergy

Carbamazepine 20 mg Three times a day after food


Valproate 200 mg OD after food at morning
Clonazepam 5mg Three times a day after food

Advice to the patient/Instruction to the Nurse:


1. Take the drugs regularly akilan
Signature

Criticism:

1. Address and phone number of doctor are missing


2. Date of prescription is not present
3. Address and weight of the patient are not provided
4. Rx symbol is absent
5. Type of formulation for all the drugs are not mentioned
6. Carbamazepine also has tendency to produce allergic rashes.As the patient has already
developed rashes carbamazepine is not preferred. Dose of carbamazepine is wrong
7. The dose of valproate is low.It should be given as 200 mg tds. As the rashes are infrequent
with valproate it is preferred.
8. Clonazepam has no value in grandmal epilepsy prophylaxis.

68
Corrected Prescription:

Doctor’s Name: Date:


(With degree)
Name of the patient:
Registration No: Age:
Address: Sex:
Address:
Phone No: Weight:

DIAGNOSIS: Uncontrolled GTCS with phenytoin allergy

Rx:

T. SODIUM VALPROATE 200 mg three times daily p.c for 1 Month

Advice to the patient/Instruction to the Nurse:

1. If you develop yellow discoloration of urine or eyes, excessive fatigue report


immediately
2. Maintain a seizure dairy.
3. Do not forget to take the tablets regularly

Signature with date:

69
NMC- Competency No: PH 1.1 & 3.2
Ex. No: 15 Prescription criticism and rewriting

5. Criticize the following prescription and rewrite with rationale drugs.

Doctor’s Name: Dr. L. Mugilan Date:


Patient’s Name: Mr. Kamal
Registration No: Age: 18 Sex: Male
Address: Eswari Nagar, Thanjavur Address: Mullai Nagar, Thanajvur
Phone No: +91-6854785411 Weight: 42 kg

DIAGNOSIS: Diabetic keto acidosis

Rx:
Tab. Metformin 2g OD after food
Tab. Sodium Bicarbonate 50mg after food

Advice to the patient/Instruction to the Nurse:


1. mugilan
Signature

Criticism:

1. Degree, registration number of the doctor are missing


2. Drug names are not capitalized
3. Advice is missing
4. Tab Metformin produces lactic acidosis, moreover Insulin is the drug of choice for diabetic
ketoacidosis,So Metformin is not indicated.
5.Sodium Bicarbonate is indicated to correct acidosis but route of administration is through
i.v. route.
7. I.V. fluids are compulsory in DKA. ½ N saline, Normal saline, 5% Glucose solution are
given i.v and are not mentioned in the prescription.

70
Corrected Prescription:
Fill the other details

Doctor’s Name: Date:


(With degree)
Name of the patient:
Registration No: Age:
Address: Sex:
Address:
Phone No: Weight:

DIAGNOSIS: Diabetic keto acidosis

Rx:

Infusion. REGULAR INSULIN bolus 0.1 to 0.2 units/kg i.v. followed by


0.1 units/kg/hr till blood sugar is brought to 300 mg /dl

IV fluids- NORMAL SALINE 1-2 litre/hr till dehydration is corrected and then 0.5 litre/4 hrs.
Once BP &HR is stabillised ½ NS in 5% glucose to be given i.v.

iNJ. KCL upto 50 meq/hr after 4 hours of initiation of therapy,with insulin


to be added in I.v. fluids
.
Inj. SODIUM BICARBONATE 50 meq added to i.v.fluids till blood pH raises to 7.2

Suitable antibiotics.

Advice to the patient/Instruction to the Nurse:

1. Monitor RBS every 30 minutes


2. Monitor blood pH and electrolytes every hour
3. Monitor vitals - BP, HR, RR and urine output

Signature with date:

71
NMC- Competency No: PH 1.1 & 3.2
Ex. No: 15 Prescription criticism and rewriting

6. Criticize the following prescription and rewrite with rationale drugs.

Doctor’s Name: Dr. A. Madhan Date:


Patient’s Name: Mr. Kamal
Registration No: 5521454 Age: 45 Sex: Male
Address: Eswari Nagar, Thanjavur Address:
Phone No: Weight: 56 kg

DIAGNOSIS: Type II diabetes with hypertension

Rx:
Tab. Propranolol 1g twice daily before food
Tab. Lisinopril 1mg twice daily after food
Syrup. Potassium chloride 5ml twice daily
Cap. Glibenclamide 5mg Twice daily after food

Advice to the patient/Instruction to the Nurse:


1.
Signature

Criticism:

1. Degree and phone number of the doctor are missing


2. Date of the prescription is missing
3. Address of the patient is missing
4. Drugs are not written in capital letters
5. Advice to the patient and signature of the doctor are missing
6. Propranolol reduces carbohydrate tolerance by reducing insulin release, so it is to be avoided.
Also hypoglycemic symptoms will be masked by propranolol and the dose mentioned is wrong.
7. ACE or ARB are the drug of choice as it retards diabetic nephropathy. Dose of lisinopril is wrong
8. Potassium Chloride syrup causes hyperkalemia when it is combined with ACE or ARBs
so it is not indicated.
9. Standard guidelines recommend to start metformin as first line agent unless contraindicated
and hence glibenclamide should be replaced with metformin.

72
Corrected Prescription:
Fill the other details

Doctor’s Name: Date:


(With degree)
Name of the patient:
Registration No: Age:
Address: Sex:
Address:
Phone No: Weight:

DIAGNOSIS: Hypertension with diabetes mellitus

Rx:
T. LISINOPRIL 5mg o.d (0-0-1) p.c for one month
T. METFORMIN 500mg b.d (1-0-1) p.c for one month.

Advice to the patient/Instruction to the Nurse:

1. Avoid foods rich in carbohydrates like icecream, sweets, honey and melon
2. Avoid salt rich foods like dryfish, pickles etc.,
3. Do mild exercises regularly and drink plenty of water.

Signature with date:

73
P -DRUG & ESSENTIAL DRUG LIST

OBJECTIVES

At the end of this exercise, the learner shall be


able to:

1. Prepare and explain a list of p drugs for a


given condition/case
2. Make a 'p' drug list for common diseases
encountered in clinical practice
3. Define the concept of essential medicines
and appreciate its importance
4. List the guidelines for selection of essential
medicines
5. Able to prepare the essential drug list for
various health care

74
NMC- Competency No: PH 3.5 & 3.7
Ex. No: 16 P drug and essential medicine preparation
P-drug: Personal drug (p-drug) means personal choice of drug of a clinician for a particular
condition based on the essential drug list, availability of the drug, safety of the drug and cost,
national drug list, guidelines, hospital formulary. It should include the name of the drug,
strength, dosage form, schedule and duration of treatment.
Steps in selection of P-drug:
 Step 1:Clearly define the diagnosis/patient problem
 Step2: Specify the therapeutic objective
 Step3: Make the list of effective group of drugs
 Step 4: Select an effective group based on the efficacy, safety and economy
 Step 5: Select a drug (P-drug) from that group again based on the above criteria with
justification
 Step 6: Conclusion (drug formulation, strength, dosage form, schedule and duration of
treatment)

Essential Medicine List (EML): Those drugs that satisfy the health care needs of the majority of
the population; they should therefore be available at all times in adequate amounts and in
appropriate dosage forms, at a price the community can afford.

P-drug list: This indicates list of drugs selected by the physician from the existing medicine list
(WHO-EML, or National EML or hospital formulary) to meet his/her specific objectives

Worked Example:
Choose a “p” drug for a patient with angina pectoris (assume no other clinical complications
are associated in this case).
Step 1: Define the diagnosis:
Stable angina pectoris caused by partial occlusion of coronary artery
Step 2: Specify therapeutic objectives:
 Stop an attack as soon as it happens
 Reduce myocardial oxygen need by decreasing preload, contractility, and heart rate or
after load
Step 3: Make inventory of effective groups:
 Nitrates Beta blockers Calcium channel blockers

Step 4: Choosing a group according to the criteria


Drug group Efficacy Safety Suitability Cost
Nitrates (tablet) + +/- ++ +
Beta blockers (injection) + +/- - -
CCB (injection) + +/- - -
Hence, nitrate group is selected
Step 5: Choose a p drug:
Nitrates Efficacy Safety Suitability Cost
GTN (tablet) + +/- ++ ++
GTN (spray) + +/- +/- -
ISDN (tablet) + +/- + +/-
ISMN (tablet) + +/- + +/-
Step 6: Conclusion: Sublingual tablet. Glyceryl trinirate 1mg 1 tablet as needed

75
NMC- Competency No: PH 3.5 & 3.7
Ex. No: 16 P drug and essential medicine preparation

1. An adult male patient aged 34 years came to the hospital with complaints of acute
breathlessness. The provisional diagnosis is acute attack of bronchial asthma. Select a
‘p’ drug for this patient.

Step 1: Diagnosis:
Acute attack of bronchial asthma

Step 2: Therapeutic objective:


1. Create bronchodilation in order to provide relief from symptoms

Step 3: Inventory of effect drugs for bronchodilation

1. Beta 2 agonists 2. Anticholinergics 3. Methylxanthines 4. Magnesium

Step 4: Choosing a group according to objective - bronchodilation


Drug group Efficacy Safety Suitability Cost
Beta-2 agonist +++ ++ +++ ++
Anticholinergic ++ ++ ++ +
Methylxanthines ++ - + ++
Magnesium + -- -- ++
Hence, beta 2 agonist group is selected

Step 5: Choosing a p drug:


Drug Name Efficacy Safety Suitability Cost
Salbutamol tablet ++ ++ -- ++
Salbutamol Nebu ++ +++ ++ ++
Salmeterol +++ - -- --
Formoterol ++ - + --
Terbutaline ++ ++ -- ++

Hence, salbutamol nebulizer solution is selected.

Step 6: Conclusion
Salbutamol nebulizer solution 2.5mg in 3ml of normal saline for 20 minutes

76
NMC- Competency No: PH 3.5 & 3.7
Ex. No: 16 P drug and essential medicine preparation
2. An adult male patient of 58 years has developed pain and swelling at first metatarso
phalangeal joint in both foot for last 2 days. Pain was severe since last week. A diagnosis
of acute gouty arthritis was made. Select a ‘p’ drug for this patient.

Step 1: Diagnosis
Acute attack of gouty arthritis

Step 2: Therapeutic objective


1. To relieve the pain in the acute gouty arthritis
(Recent guidelines - ACR 2020 does not recommend to start urate lowering therapy for first attack)

Step 3: Inventory of effective drugs in analgesics


1. NSAIDs 2. Opioids 3. Colchicine 4. Corticosteroids
Step 4. Choosing a group according to the objective - pain reduction
Drug group Efficacy Safety Suitability Cost
NSAIDs ++ ++ +++ ++
Opioids + - - +
Colchicine ++ -- + ++
Corticosteroids + - - ++
Hence , NSAIDs group is selected
Step 5. Choosing a p drug
Drug Name Efficacy Safety Suitability Cost
Naproxen tablet +++ + ++ +++
Indomethacin capsule +++ - ++ ++
Diclofenac tablet ++ - ++ +++
Diclofenac topical + ++ - -
Meloxicam tablet ++ - ++ -
Etoricoxib tablet ++ -- - -

Hence, Tablet Naproxen is selected

Step 6. Conclusion:
Tablet. Naproxen 750mg stat followed by 250mg TDS for 5 days is selected for pain reduction
in acute gout.

77
NMC- Competency No: PH 3.5 & 3.7
Ex. No: 16 P drug and essential medicine preparation

3. An adult male patient of 42 years weighing 82 kg was diagnosed with type II diabetes
mellitus. Select a ‘p’ drug for this patient.
Step 1: Diagnosis
Type II diabetes mellitus

Step 2: Therapeutic objectives


1. To lower the blood glucose level and promote weight loss to prevent insulin resistance.

Step 3: Inventory of effective anti-diabetic drug groups


1. Sulfonylureas 2. Biguanides 3. Thiazolidinediones 4. Insulin
5. SGLT-2 inhibitor 6. DPP-4 inhibitor 7. GLP-1 analogue

Step 4. Choosing the group of drug according to objective - weight loss and lower blood glucose
Drug Group Efficacy Safety Suitability Cost
Sulfonylurea ++ + + ++
Biguanides +++ ++ +++ +++
Thiazolidinediones ++ - ++ +
Insulin +++ - - -
SGLUT-2 inhibitor + - + -
DPP-4 inhibitor ++ + + -
GLP-1 analogue ++ - - -
Hence, biguanide group of drug is selected

Step 5. Selecting a p drug


Drug Name Efficacy Safety Suitability Cost
Metformin tablet +++ ++ +++ +++
Metformin SR tablet +++ +++ ++ +

Hence Metformin tablets are selected.


Step 6. Conclusion
Tablet Metformin 500mg B.D for one month is selected for type II diabetes mellitus.

78
NMC- Competency No: PH 3.5 & 3.7
Ex. No: 16 P drug and essential medicine preparation
4. A 50 year-old male patient complains of bloody, mucoid stools and abdominal pain.
There is no history of alcohol abuse. You have diagnosed it as a case of acute amoebic
dysentery. Select a ‘p’ drug for this patient.

Step 1: Diagnosis
Acute amoebic dysentry

Step 2: Therapeutic objectives


1. to eradicate the trophozoite form of ameoba, prevent its invasion, and cure the dysentry.

Step 3: Inventory of effective anti-amoebic drug groups


1. Nitroimidazoles 2. Alkaloids 3. Chloroquine 4. 8-hydroxyquinolines
5. Tetracyclines

Step 4. Choosing the group of drug according to objective - eradication of trophozoite in tissue
and cure dysentry.
Drug Group Efficacy Safety Suitability Cost
Nitroimidazoles +++ ++ +++ +++
Alkaloids ++ -- -- +
Chloroquine ++ + ++ +++
8-hydroxyquinolines + - - +++
Tetracyclines + -- + ++
Hence, nitroimidazole group of drugs are selected

Step 5. Choosing a p drug


Drug Name Efficacy Safety Suitability Cost
Metronidazole tablet +++ ++ +++ +++
Metronidazole infusion +++ ++ ++ ++
Tinidazole tablet +++ ++ +++ +
Ornidazole tablet +++ ++ +++ +
Satranidazole tablet +++ +++ +++ -
Secnidazole tablet +++ ++ +++ +
Hence, metronidazole tablet is selected

Step 6. Conclusion
Tablet Metronidazole 800 mg TDS for 7 days is selected for acute amoebic dysentry.

79
NMC- Competency No: PH 3.5 & 3.7
Ex. No: 16 P drug and essential medicine preparation
5. You are a general practitioner in a small village with a population of 5000. You are
frequently called at night to attend to patients in their homes. Prepare a list of drugs
that you will carry with you to treat these emergencies

Step 1: Enumeration of common medical emergencies in villages at night


1. Anaphylactic shock due to insect bite 2. Acute angina
3. Dysmennorhea 4. Hypoglycemic shock 5. Acute asthmatic attack
6. Renal colic 7. Severe vomiting 8. Severe diarrhoea
9. Seizures 10. Exacerbation of peptic ulcer

Snake bite, scorpion bite & poisonings cannot be attended at home and should be referred.
Step 2. List of EML to treat the common medical emergencies in villages at night
Drug name Formulation Strength Capacity
1. Adrenaline Ampoule for injection 1:1000 i 1 ml
2. Atropine Ampoule for injection 0.6mg/ml 2 ml
3. Diclofenac Ampoule for injection 75mg/ml 1ml
4. Paracetamol Ampoule for injection 150mg/ml 2 ml
Paracetamol Suppository 125mg --
Paracetamol Scored Tablet 500mg --
5. Diazepam Ampoule for injection 5mg/ml 2 ml
Diazepam Suppository 10mg --
6. Dicyclomine Ampoule for injection 10mg/ml 2 ml
7. Metoclopramide Ampoule for injection 5mg/ml 2ml
8. Dextrose solution for infusion 5% 500ml
9. Ringer Lactate Solution for infusion -- 500ml
10. ORS powder for reconstitution --- to dissolve in 1L
11. Cetirizine Tablet 10mg --
12. Hydrocortisone Vial for injection 50mg/ml 2ml
13. Mefenamic acid Tablet 250mg --
14. Salbutamol Nebulizer solution 2.5mg/3ml 3ml
15. Isosorbide dinitrate Sublingual tablet 5mg --
16. Ranitidine Ampoule for injection 25mg/ml 2ml
Ranitidine Tablet 150mg ---

80
NMC- Competency No: PH 3.5 & 3.7
Ex. No: 16 P drug and essential medicine preparation
6. Prepare an essential medicines list of antibiotics for a primary care centre in Thanjavur,
Tamilnadu.

Step 1: Enumeration of common indications for antibiotics in PHC in thanjavur


1. Upper RTI 2. Urinary tract infection 3. Typhoid
4. Lower RTI 5. Tuberculosis 6. Leprosy

Step 2. EML of antibiotics for a PHC in Thanjavur

Drug name Formulation Strength Capacity


1. Amoxicillin Capsules 250mg, 500mg ---
2. Ampicillin Vial for injection 500mg, 1g/vial 5ml
3. Ceftriaxone Vial for injection 1g/vial 10 ml
4. Ciprofloxacin Solution for infusion 200mg/100ml 100 ml
Ciprofloxacin Tablets 250mg, 500mg --
5. Metronidazole Solution for infusion 500mg/100ml 100 ml
Metronidazole Tablets 200mg, 400mg --
6. Doxycycline Capsules 100mg/cap --
7. Azithromycin Tablets 500mg --
8. Cotrimoxazole Tablets 400-80mg/tab --
9. Gentamicin Vial for injection 80mg/2ml 2 ml
10. Rifampicin Capsules 300mg --
11. Dapsone Tablet 100 mg --
12. Clofazamine tablet/capsule 50/100 --
13. H/R/Z/E FDC tablet 75/150/400/275mg --
14. H/R/E FDC tablet 75/150/275mg --
15. Povidone iodine Ointment 5% w/w 15g

81
PHARMACOVIGILANCE &
ADR REPORTING

OBJECTIVES

At the end of this exercise, the learner shall be


able to:

1. Describe the principles of Pharmacovigilance


and ADR reporting system in India .

2. Recognize and report an adverse drug


reaction.

3. Do the causality assessment for a particular


ADR as per WHO and Naranjo’s causality
assessment scale.

82
NMC- Competency No: PH 1.6 & 3.4
Ex. No: 17 Pharmacovigilance and reporting of ADR
Pharmacovigilance: As per WHO, Pharmacovigilance is the science and activities related to
detection, assessment, understanding and prevention of adverse effects or any other drug-
related problem.

Necessity for Pharmacovigilance:


 It promotes rationale use of medicine and ensures patient compliance and confidence
 Long term safety and rare adverse events can be established for both old and new drugs
in market.
 Unnecessary burden due to increased cost of treatment from ADR can be reduced
 It is ethical and moral duty of health professionals to report the ADRs

Pharmacovigilance programme of India (PvPI): The CDSCO established PvPI in July 2010 to
identify and respond to drug safety problems. The National Coordination centre of PvPI is
Indian Pharmacopoeia commission, Ghaziabad. PvPI also collaborates with the WHO-Uppsala
Monitoring Centre, Sweden. As a collaborating center, PvPI assists WHO in developing
international policy for other countries to manage their own drug safety program. Around 250,
ADR monitoring centers in India sends the ADR reports to the National coordination center,
Ghaziabad. PvPi also includes hemovigilance (ADR related to blood products), biovigilance (ADR
due to tissue, organ, cell therapy), materiovigilance (ADR due to biological implants/devices),
and ADR following immunization (AEFI).

Uppsala Monitoring Centre (UMC): It is located in Uppsala, Sweden. UMC is the field name for
the WHO-Collaborating center for International Drug Monitoring. It was founded in 1978. UMC
works by collecting, assessing and communicating information from member countries’
national pharamcovigilance center in regard to the benefit, harm, effectiveness and risks of
drugs.

Vigiflow: It is the unique WHO global database of individual case safety reports (ICSRs). It is the
largest data base with 25 million reports of suspected adverse drug events of medicine
submitted since 1968 by the member countries of WHO programme for International Drug
Monitoring.

Vigiflow: It is a web-based ICSR management system that is available for use by national
Pharmacovigilance centers of WHO programme for International Drug Monitoring. Vigiflow
supports the collection, processing and sharing of data of ICSRs to facilitate effective data
analysis.

Adverse drug Event (ADE): Any untoward medical occurrence that may present during
treatment with a pharmaceutical product but which does not necessarily have a causal
relationship with the treatment.

Adverse drug reaction (ADR): ADR is defined as a noxious, undesired or unintended effect of a
drug, which may occur at doses normally used in man for various purposes (prophylactic,
therapeutic or diagnostic) or for the modification of physiological state.

83
NMC- Competency No: PH 1.6 & 3.4
Ex. No: 17 Pharmacovigilance and reporting of ADR
Signal: As per WHO-Uppsala Monitoring center, signal is reported information on a possible
causal relationship between an adverse event and a drug, the relationship being unknown or
incompletely documented previously. Signal is the first alert about the problem/adverse effect.
Signal generation implies detection of early warning signs of adverse effects and can be
generated by all the health professionals.

Reporting of ADR:
 What to report?: All the suspected reactions even if it is minor effect related to drug
should be reported.
 Who to report? All healthcare professionals (clinicians, dentists, pharmacist, nurses,
paramedical staff etc,) should report the ADR. Patient can also report the ADR
voluntarily.
 Where to report? Duly filled and signed form should be reported to ADR monitoring
centre in the concerned institute or directly to the National Coordination Centre,
Ghaziabad via pvpi.ipcindia@gmail.com and toll free no is 18001803024.
 What happened when submitted report? The causality assessment is carried out at
AMCs by using WHO-UMC scale. The analysed forms are forwarded to NCC India
through ADR data base. The report received at NCC is dealt with strict confidence.
Ultimately, data are analyzed and sent to global Pharmacovigilance data base managed
by Uppsala Monitoring centre, Sweden.

Causality assessment: It is the assessment of relationship between drug/medicine and


occurrence of adverse drug event/effect. Various methods for causality assessment are WHO-
UMC scale (refer annexure 6), Naranjo algorithm (refer annexure 7), Karch and Lasgna scale,
Kramer Scale and Jones algorithm.

Group tasks: (Fill the ADR form and do causality assessment for the given scenarios)

A. A 35yr old female patient Mrs. A. Manjula weighing 56Kg was diagnosed to have
generalized tonic clonic convulsions. She was started on Tab. Phenytoin (Eptoin) 100 mg bd
for 5 months on 12.04.2021. She noticed gum hypertrophy on 17.09.2021 and reported to
the dentist. Dentist ruled out local infection and asked her to consult with her physician.
Tab. Phenytoin was stopped on 19.09.2021 and replaced with Tab. Phenobarbitone. The
gum hypertrophy was subsided by 12.10.2021. Owing to improper seizure control with
Phenobarbitone, Tab. Phenytoin was reintroduced on 13.12.2021 and the gum hypertrophy
relapsed. Fill the ADR form and assess the causality of this ADR as per WHO and Naranjo
Scale.

B. A 52 year old male patient Mr. P. Shankar weighing 75 Kg was diagnosed as hypertensive
and started on Tab. Amlodipine 5mg OD at night on 14.08.2021. He was not on any other
medication. He is an occasional alcoholic and not a smoker. On the morning of 21.08.2021,
he noticed edema over both the ankles and reported to the treating physician at evening.
His blood pressure is under control. His serum total protein level was 7.8g/dL and serum
albumin level was 4.1g/dL. Amlodipine was stopped on 22.08.2021 and he was started on
Tab. Losartan 50mg OD on the same day. Ankle edema was resolved on 24.08.2021. Fill the
ADR form and assess the causality of this ADR as per WHO and Naranjo Scale.
84
NMC- Competency No: PH 1.6 & 3.4
Ex. No: 17 Pharmacovigilance and reporting of ADR
C. A 64 year old male Mr. S. Kumarasamy weighing 60Kg was diagnosed with Stage IIa
seminoma testis on 24.03.2020 and was planned to start chemotherapy with three cycles of
the following regimen.
a. Injection. Cisplatin (20mg/m2) intravenous for 5 days
b. Injection. Etoposide (100mg/m2) intravenous for 5 days
c. Injection. Bleomycin (10U/m2) intravenous once a week.
The first cycle was administered on 30.03.2020. On 03.04.2021, the patient developed
hematemesis and the following investigation was done on 03.04.2021
a. Hemoglobin – 6g/dL
b. Red blood cell count – 1.3 million cells/mm3
c. Absolute neutrophil count – 300cells/mm3
d. Platelet count – 70000 cells/mm3
The third cycle was planned on 2.05.2021 as the blood counts improved with repeat tests
on 18.05.2021. Fill the ADR form and assess the causality of this ADR as per WHO and
Naranjo Scale.

D. A 25 year old female Ms. R. Kamalam weighing 52kg was diagnosed to have systemic lupus
erythematosus and was started on Tab. Prednisolone 5 mg bd on 23.05.2021. Her serum
creatinine level measured on 21.05.2021 was 1.8 mg/dL. On 23.05.2021, her serum
creatinine level measurement was repeated and found to be 1.85mg/dL. Fill the ADR form
and assess the causality of this ADR as per WHO and Naranjo Scale.

E. A 30 year old female Ms.R. Saraswathy weighing 60kg was diagnosed as schizophrenia and
started on Tab. haloperidol 5 mg OD on 22.06.2020. After 2 weeks of therapy (on
06.07.2020), she developed tremors and muscular dystonia. She is also taking Tab. Ferrous
sulphate 200mg TDS and T. Vitamin B complex 1 OD for her iron deficiency anemia
regularly. Haloperidol was replaced with Tab. Olanzapine 5mg OD on 07.07.2020 and the
symptoms were subsided on 09.07.2020. Fill the ADR form and assess the causality of this
ADR as per WHO and Naranjo Scale.

F. A 35 year old female Mrs. R. Kannagi weighing 53Kg has developed symptoms and signs of
urinary tract infection for which she was given inj. Ciprofloxacin 200mg I.V B.D for 5 days on
13.06.2021. She developed pain and erythematous rash at the site of injection within 15
minutes and subsided after one hour. At the evening of 13.06.2021, the erythematous rash
reappeared within 5 minutes of second dose of inj. Ciprofloxacin which involves the entire
arm and subsided only after three hours. She is also on T. Paracetamol 500mg three times a
day. Fill the ADR form and assess the causality of this ADR as per WHO and Naranjo Scale.

85
NMC- Competency No: PH 1.6 & 3.4
Ex. No: 17 Pharmacovigilance and reporting of ADR- group A

__ ---
A.M 35 56

17.09.2021
12.10.2021 --

Diagnosed as gum hypertrophy due to tablet phenytoin


when the drug was stopped and replaced with ___
T. Phenobarbitone. The gum hypertrophy subsided
Again the gum hypertrophy occurs after reintroduction of
T. Phenytoin

Phenytoin induced gum hypertrophy

_______

t. eptoin 100mg oral b.d 12.04.21 19. 9.21 GTCS certain

YES Yes same as above

Fill the appropriate details


NIL

02.02.2022

86
NMC- Competency No: PH 1.6 & 3.4
Ex. No: 17 Pharmacovigilance and reporting of ADR- group B

S. Protein = 7.8g/dL
_ S. Albumin = 4.1g/dL
P.S 52
75

21.08.2021
24.08.2021 NIL

Patient has been started with T. amlodipine for hypertension 


and he developed ankle edema after 7 days. Amlodipine was
replaced with T.Losartan and ankle edema resolved.

Amlodipine induced ankle edema

 _______

t. amlodipine 5mg oral OD 14.08.21 22.08.21 Hypertension Probable

Yes Not applicable

Fill the appropriate details

02.02.2022

87
NMC- Competency No: PH 1.6 & 3.4
Ex. No: 17 Pharmacovigilance and reporting of ADR- group C


on 03.04.2021
Hb = 6g/dL
 RBC count = 1.3 million cells/cc
S.K 64 Absolute neutrophil count = 300 cells/cc
60 Platelet count = 7000 cells/cc

03.04.2021
18.05.2021

Diagnosed as Stage IIa seminoma and was administered


standard anticancer regimen. Patient developed 
hemetemesis after 4 days. Second and third cycle was
administered as the symptoms resolved and relevant lab

parameters improved.

Anticancer drug induced bone marrow depression

Inj. Cisplatin 20mg/m2 iv OD 5day 30.03.21 -- Stage IIa Possible


Inj. Etoposide 100mg/m2 iv OD 5day 30.03.21 -- Seminoma
Inj. Bleomycin 10U/m2 iv once week 30.03.21 --

Yes Not applicable

Fill the appropriate details


NIL

03.02.2022

88
NMC- Competency No: PH 1.6 & 3.4
Ex. No: 17 Pharmacovigilance and reporting of ADR- group D

Sr. Creatinine on 21.05.2021 = 1.8mg/dL


 Sr. Creatinine on 23.05.2021 = 1.85mg/dL
R.K 25
52

21.05.2021
Not applicable Known case of SLE

? Prednisolone induced elevated renal parameters 


Diagnosed as SLE. Her baseline serum creatinine was high
even before starting the prednisolone and remains elevated
after starting T.Prednisolone.

Not applicable

T. Prednisolone 5mg Oral BD 23.05.21 --- SLE UNLIKELY

Not applicable Not applicable

Fill the appropriate details

NIL

03.02.2022

89
NMC- Competency No: PH 1.6 & 3.4
Ex. No: 17 Pharmacovigilance and reporting of ADR- group E

 Nil
R.S 30 years 60

06.07.2020
09.07.2020

Haloperidol induced acute muscular dystonia



Patient was diagnosed as schizophrenia and was prescribed
T. Haloperidol. After two weeks of therapy, she developed
acute muscular dystonia. Haloperidol was replaced with
T. Olanzapine and the symptoms subsided.

T. Haloperidol 5mg Oral OD 22.06.20 07.07.20 Schizophrenia Probable

Yes Not applicable

T. Ferrous sulphate 200mg Oral TDS 22.06.20 ---


T. Vitamin BC --- Oral OD 22.06.20 ---

Fill the appropriate details

04.02.2022

90
NMC- Competency No: PH 1.6 & 3.4
Ex. No: 17 Pharmacovigilance and reporting of ADR- group F

 Nil
R.K 35y
53

13.06.2021
13.06.2021

Ciprofloxacin induced erythematous rashes



Patient was infused with ciprofloxacin for UTI and she developed
pain and erythematous rash at the site of injection within 15
minutes after infusion and subsided after one hour. Same
reaction reappeared for the evening (second) dose.

Inf. Ciprofloxacin 200mg iv BD 13.06.21 --- UTI Certain

Yes Yes same as above

T. Paracetamol 500mg Oral TDS 13.06.21 -- Pyrexia

Fill the appropriate details

NIL

04.02.2022

91
NMC- Competency No: PH 1.6 & 3.4
Ex. No: 17 Pharmacovigilance and reporting of ADR
Causality assessment – Group tasks.

Causality
Assessment Criteria met as per WHO scale Causality
Group Naranjo
as per Assessment as
Task Score
Naranjo per WHO scale
Scale Criteria Yes/ No
Temporal association Yes
De-challenge Yes
Certain
9 Definite Re-challenge Yes
A
ADR explained pharmacologically Yes
Concomitant drugs cause same ADR No
Disease also cause same ADR No
Temporal association Yes
De-challenge Yes
7 Probable Re-challenge NO
B Probable
ADR explained pharmacologically Yes
Concomitant drugs cause same ADR No
Disease also cause same ADR No
Temporal association Yes
De-challenge No
4 Possible Re-challenge No
C Possible
ADR explained pharmacologically Yes
Concomitant drugs cause same ADR Yes
Disease also cause same ADR Yes
Temporal association No
De-challenge No
-2 Doubtful Re-challenge No Unlikely
D
ADR explained pharmacologically No
Concomitant drugs cause same ADR No
Disease also cause same ADR Yes
Temporal association Yes
De-challenge Yes
7 Probable Re-challenge No Probable
E
ADR explained pharmacologically Yes
Concomitant drugs cause same ADR No
Disease also cause same ADR No
Temporal association Yes
De-challenge Yes
9 Definite Re-challenge Yes
F Certain
ADR explained pharmacologically Yes
Concomitant drugs cause same ADR No
Disease also cause same ADR No

92
NMC- Competency No: PH 3.3 & 3.6
Ex. No: 18 Critical appraisal of drug promotion literature & interaction
with pharmaceutical representative to get authentic information on
drugs
OBJECTIVES:
At the end of this session, the student shall be able to analyse critically, drug promotional
literature for proprietary preparations, in terms of the
 Claims of pharmaceutical companies on the pharmacological actions of their ingredients
 Economics of use
 Rational or irrational drug combinations, if any
 Identify unethical marketing practices
 Realize the extent to which drug advertisements can influence prescribing behaviour.
 Impact of direct to consumer advertisement in patients.

SESSION PLAN:
 A brief introduction (15 min) on drug promotion will be given highlighting its advantages
and disadvantages, ethical and unethical aspects.
 The ethical criteria for medicinal drug promotion and its key messages will be discussed.
 The batch will be divided into 6 groups and each group will be given one advertisements
and asked to carry out a group task (30 minutes) followed by a plenary (45 mins)

Group Tasks (A-F):

Critically analyze the given drug promotional literature and advertisements. Give your opinion
on the following:
 Validity of scientific claims especially regarding pharmacological actions of the drugs
 Content of scientific information
 Appropriateness of illustrations
 Relevance of references cited
 Rational or irrational nature of fixed dose drug combinations.

Refer the following six pages for group tasks problems.

93
NMC
NMC- Competency No: PH 3.3 & 3.6
Ex. No: 18 Critical appraisal of drug promotion literature & interaction
with pharmaceutical representative to get authentic information on
drugs – group A task

94
NMC- Competency No: PH 3.3 & 3.6
Ex. No: 18 Critical appraisal of drug promotion literature & interaction
with pharmaceutical representative to get authentic information on
drugs – group A criticism

A. Validity of scientific claims


The name and mechanism of action of the active ingredient are not mentioned. Only the broad
actions of the formulation is mentioned.

B. Content of scientific information


The scientific information provided is inadequate. The study described in the advertisement does
not have control arm (placebo or standard drug) which indicates the study results are inferior.
Moreover, the main parameter measured is the cough score which is very subjective and not
validated.

C. Appropriate of illustrations:
No attractive illustrations is given in the advertisement. Moreover, the generic name of the drug
is not mentioned in the advertisement.

D. Relevance of reference cited


The reference provided in the advertisement is 'personal communication' which is not a
acceptable scientific reference.

E. Rational/irrational FDC:
Not applicable for this advertisement as the active ingredients in the formulation are not provided

95
NMC
NMC- Competency No: PH 3.3 & 3.6
Ex. No: 18 Critical appraisal of drug promotion literature & interaction
with pharmaceutical representative to get authentic information on
drugs – group B task

96
NMC- Competency No: PH 3.3 & 3.6
Ex. No: 18 Critical appraisal of drug promotion literature & interaction
with pharmaceutical representative to get authentic information on
drugs – group B criticism

A. Validity of scientific claims


Hyperhomocysteinemia is one of the causes of infertility and adverse pregnancy outcome. Thus
supplementation of Vitamin B12, folic acid and pyridoxine is one approach to reduce the homocysteine
levels. Hence this advertisement has valid scientific claims.

B. Content of scientific information


The scientific information provided is inadequate. Information regarding the study details or
relevant parameters measured are NOT given.

C. Appropriate of illustrations:
Illustration provided is appropriate for infertility. Generic name of the active ingredient are given in the
advertisement. However, the illustrations related to parameter measured should also be mentioned.

D. Relevance of reference cited


No reference was given in the advertisement which is compulsory for any scientific claim.

E. Rational/irrational FDC:
Combining pyridoxine and folic acid is inadequate. Vitamin B12 should also be combined in this FDC.

97
NMC- Competency No: PH 3.3 & 3.6
Ex. No: 18 Critical appraisal of drug promotion literature & interaction
with pharmaceutical representative to get authentic information on
drugs –group C task

98
NMC- Competency No: PH 3.3 & 3.6
Ex. No: 18 Critical appraisal of drug promotion literature & interaction
with pharmaceutical representative to get authentic information on
drugs –group C Criticism

A. Validity of scientific claims


Valdecoxib being a selective COX-2 inhibitor has not central action. It does not "remove worry of
pain through additional central action". Thus the advertisement has inappropriate scientific claims.
Moreover, currently valdecoxib is withdrawn due to high cardiovascular ADRs.

B. Content of scientific information


The scientific information provided is inadequate. Information regarding the study details or
relevant parameters measured are NOT given.

C. Appropriate of illustrations:
Generic name of the active ingredient are given in the advertisement but not mentioned clearly in
similar way to their brand name. Illustrations in the form of table regarding the cost is appreciated.

D. Relevance of reference cited


No reference was given in the advertisement which is compulsory for any scientific claim.

E. Rational/irrational FDC:
Not applicable for this advertisement.

99
NMC
NMC- Competency No: PH 3.3 & 3.6
Ex. No: 18 Critical appraisal of drug promotion literature & interaction
with pharmaceutical representative to get authentic information on
drugs – group D task

100
NMC- Competency No: PH 3.3 & 3.6
Ex. No: 18 Critical appraisal of drug promotion literature & interaction
with pharmaceutical representative to get authentic information on
drugs – group D Criticism

A. Validity of scientific claims


Pegylaed filgrastim is a colony stimulating factor which increases granulocyte counts. Thus the
advertisement has valid scientific claims.

B. Content of scientific information


The scientific information provided is adequate. Information regarding the study details or
relevant parameters measured like median number of days for neutrophil engraftment in days,
patients requiring blood transfusion, and median duration of grade 4 neutropenia are given.

C. Appropriate of illustrations:
All the figures mentioned in this advertisement are "vertical bar diagram" chart which has
inappropriate starting base number (Instead of zero). This gives an impression that 'Pegylated-
filgrastim' is performing better than filgrastim. This is not true because Pegylated filgrastim just
increases the median days for neutrophil engraftment and median neutropenia by only 1 day.
This inappropriate illustration is called "lost-Zero" graph problem which is unacceptable.

D. Relevance of reference cited


Reference number has been provided but No reference was given in the advertisement which is
compulsory for any scientific claim.

E. Rational/irrational FDC:
Not applicable for this advertisement.

101
NMC- Competency No: PH 3.3 & 3.6
Ex. No: 18 Critical appraisal of drug promotion literature & interaction
with pharmaceutical representative to get authentic information on
drugs – Group E task

102
NMC- Competency No: PH 3.3 & 3.6
Ex. No: 18 Critical appraisal of drug promotion literature & interaction
with pharmaceutical representative to get authentic information on
drugs – Group E criticism

A. Validity of scientific claims


Sparfloxacin, one of the fluoroquinolone has extended spectrum of activity and commonly used
for ocular conditions. Thus the advertisement has valid scientific claims.

B. Content of scientific information


The scientific information provided is adequate. The parameters measured are mentioned.

C. Appropriate of illustrations:
illustrations regarding the actions of sparfloxacin is not given. The generic name is mentioned in
the advertisement however it is not clearly visible like the brand name.

D. Relevance of reference cited


Reference number has been provided in the claims and references are also cited. The second
reference is a textbook which is considered inferior standard.

E. Rational/irrational FDC:
Not applicable for this advertisement.

103
NMC
NMC- Competency No: PH 3.3 & 3.6
Ex. No: 18 Critical appraisal of drug promotion literature & interaction
with pharmaceutical representative to get authentic information on
drugs – Group F task

104
NMC- Competency No: PH 3.3 & 3.6
Ex. No: 18 Critical appraisal of drug promotion literature & interaction
with pharmaceutical representative to get authentic information on
drugs – Group F Criticism

A. Validity of scientific claims


Cispride is used in GERD owing to its prokinetic action. Thus the advertisement has valid
scientific claim

B. Content of scientific information


The scientific information provided is adequate and the action is compared with the metoclopramide
which is appreciated. However, no study has been done or cited for these scientific statements.

C. Appropriate of illustrations:
The generic name is mentioned in the advertisement however it is not clearly visible like the
brand name.

D. Relevance of reference cited


No reference has been provided which is compulsory for scientific statements.

E. Rational/irrational FDC:
Not applicable for this advertisement.

105
DRUG INTERACTIONS

OBJECTIVES

At the end of the drug interaction exercises, the


learner shall be able to:

1. Foresee the possible drug interactions in


multiple drug therapy and poly-pharmacy

2. Identify and manage the adverse effects


caused by drug interactions

3. Know the ways to implement to prevent drug


interactions in a tertiary care hospital.

106
NMC- Competency No: PH 1.8
Ex. No: 19 Drug interactions
1. Mr. Joseph a known hypertensive for the past 5 years was on Propranolol and
hydrochlorthiazide. He was put on digoxin 0.25 mg/day for the treatment of mild CCF.
a. If he developed severe bradycardia, which drug interactions due you suspect?
b. If he developed ectopic beats and tachyarrhythmia, which drug interaction due you
suspect? Explain.
c. Which anti-hypertensive agent received by him might reduce inotropic efficacy of
digoxin? Explain.
a. Digoxin causes bradycardia by vagomimetic action. Propranolol also causes
bradycardia by blocking beta-1 adrenergic receptor. Thus propranolol and digoxin
combination is the reason for severe bradycardia

b. Hypokalemia due to hydrochlorthiazide is the reason for increased action of digoxin


leading to development of ectopic beats and tachyarrhythmia.

c. Digoxin increases inotropic action and it is antagonized by beta receptor antagonist


(propranolol), Thus the inotropic efficacy of digoxin is antagonized by propranolol.

2. A patient suffering from angina pectoris was prescribed warfarin 5 mg daily and
subsequently Tablet aspirin was also prescribed to control the symptoms of Rheumatic joint
pain. One week later the patient was brought to the casualty with severe hematemesis.
a. What is your explanation for the hematemesis?
b. Could it be prevented? Explain.
c. How will you treat the present emergency condition of this patient?

a. Warfarin is an anticoagulant which blocks vitamin k epoxide reductase. Aspirin is an


COX inhibitor which reduces thromboxane synthesis by platelets. Combination of both
these drugs lead to increased prothrombin and bleeding time and hence hemetemesis
occurs

b. Instead of non-selective COX inhibitors like aspirin for pain, a selective COX-2 inhibitor
like celecoxib or Etoricoxib can be prescribed.

c. Blood transfusion, Fresh frozen plasma, Vitamin K injection, and IV fluids.

107
NMC- Competency No: PH 1.8
Ex. No: 19 Drug interactions
3. A patient suffering from difficulty in walking, excessive salivation, fine tremors of fingers
was put on L-DOPA and Benzhexol. He was fairly relieved of his symptoms. Subsequently,
he took B-Complex tonic to improve his general health. His symptoms then started
manifesting slowly.
a. Explain the pharmacological mechanism behind the development of his symptoms
after Vitamin B complex intake.

a. The patient is suffering from Parkinsonism. The usual drug given for Parkinsonism
is L-DOPA. The L-DOPA is metabolized by DOPA decarboxylase into dopamine.
Vitamin B6 is a cofactor for peripheral DOPA decarboxylase and supplementation
of vitamin B6 increases the peripheral conversion of Levodopa to dopamine. Due to
increased peripheral conversion, lower amount of Levodopa is available for entry into
brain. Thus efficacy of L-DOPA is reduced leading to manifestation of his symptoms
again.

4. A schizophrenic patient put on long term therapy with chlorpromazine developed


pulmonary tuberculosis for which the patient was given short course therapy of
streptomycin + INH + Rifampicin and pyrazinamide. The patient subsequently complained
of nausea, vomiting, loss of appetite and passing yellow coloured urine.
a. What could be the probable cause for these complaints?
b. What is the future course of the treatment?

a. The probable cause for these complaints is 'Drug induced hepatitis'. Rifampicin and
pyrazinamide causes acute hepatitis. Chlorpromazine causes cholestatic jaundice.
Thus combination of all these hepatotoxic drugs lead to development of 'drug induced
hepatitis'.

b. Pyrazinamide and rifampicin should be stopped temporarily. Alternative regimen like

One hepatotoxic regimen: 2 months of isoniazid, ethambutol and streptomycin, followed


by 10 months of isoniazid and ethambutol.

No hepatotoxic drug regimen: 18–24 months treatment with a combination of


ethambutol, fluoroquinolone, cycloserine and capreomycin or aminoglycoside
can also be tried.

108
NMC- Competency No: PH 1.8
Ex. No: 19 Drug interactions
5. A male executive aged 42 years is suffering from angina pectoris. He is already on nitrates.
A beta adrenergic blocking drug is added to the treatment. He is a known case of asthmatic.
a. What is the justification for using the beta blockers?
b. Which beta blockers should be selected? Why?

a. Nitrates causes postural hypotension leading to reflex tachycardia via sympathetic


stimulation. Reflex tachycardia increases oxygen demand and causes aggravation of
angina. Thus a cardio selective blocker should be added and it prevents the reflex
tachycardia and thereby increases subendocardial perfusion and exercise tolerance.

b. Only cardioselective beta blockers like atenolol, metoprolol or acebutalol should be


added. Non selective beta blockers should be avoided as it can cause precipitation of
asthma.

6. Mrs. Padmavathy aged 45 years was on Phenytoin 100 mg TID for the past 2 years. Her
plasma phenytoin level was 15 mcg/ml. She has developed rheumatoid arthritis for which
she was given phenylbutazone 100 mg TID. After 5 days she developed ataxia, nystagmus
and macular rash on the skin. Her plasma phenytoin level was 30 mcg/ml.
a. What is the reason for development of these symptoms?
b. Mention the further management.

a. This is an example of pharmacokinetic drug interaction between phenylbutazone


and phenytoin. Phenylbutazone has high plasma protein binding and displaces
phenytoin from its bound state. Increased free phenytoin level causes development
of toxic symptoms described above.

b. Instead of phenylbutazone, alternative NSAID with less plasma protein binding like
diclofenac, Nimesulide etc can be prescribed.

109
NMC- Competency No: PH 1.8
Ex. No: 19 Drug interactions
7. A 26 year old married female, weighing 50 kg, on the second month of continuation phase
of Anti tuberculosis treatment was brought to the hospital by her husband. She was now on
T. RIFAMPICIN 600 mg and T. ISONIAZID 600 mg thrice weekly. Since the couple wishes to
have a child in near future, they asked the junior resident about the risks and benefits. The
junior resident advised to follow contraception for next 6 months in order to avoid
teratogenicity due to ATT and he prescribed her low dose combined oral contraceptive pills.
One month later she presented to the clinic with complaints of missed periods. She was
found to be pregnant.
a. What is/are the reason(s) for the failure of contraception?
b. How could it have been avoided?

a. The failure of contraception is due to pharmacokinetic interaction between rifampicin


and OCPs. Rifampicin is a potent inducer of CYP3A4 leading to increased metabolism
of ethinylestradiol in OCP. This causes failure of contraception.

b. Instead of OCPs, barrier method of contraception can be prescribed.

8. An adult female patient of 59 years was well maintained on T. Telmisartan for last 6 months
and she was suffering from stage I hypertension. Her blood pressure was within normal
range. She developed joint pain and prescribed tablet Diclofenac sodium 50mg BD for 1
month. After one month, she developed giddiness and morning headache. Her BP was
raised on further examination.
a. What is the likely cause of raised BP?
b. How will you manage and prevent such problem in future?

a. The likely cause of raised BP is due to sodium and water retention as a result of
NSAID use. All NSAID raise the circulatory volume on prolonged use.

b. NSAID for such cases should be used for short period of time and sodium salt
intake should be restricted. NSAID with least effect on circulatory volume like
aspirin, paracetamol or sulindac can be prescribed.

110
NMC- Competency No: PH 1.8
Ex. No: 19 Drug interactions
9. A 54 years old adult male weighing 70kg with DVT was prescribed warfarin (5 mg OD) and
enoxaparin (1mg/kg s.c OD) for 5 days. Tab. Ciprofloxacin 500mg BD was added on 6th day
for his UTI. On 8th day patient developed hematuria and bleeding of gums.
c. What is the likely cause of the problem?
d. How will prevent such problem in future.

a. The problem was probably due to toxicity of warfarin due to inhibition of its metabolism
by ciprofloxacin. (Ciprofloxacin inhibits CYP1A2 enzyme that metabolises warfarin)

b. Problem can be avoided by using cotrimoxazole or second generation fluoroquinolones


(Levofloxocin) that has no drug inhibition effect

10. A male patient of 56 years was well maintained on T. Isosorbide dinitrate 20mg BD for
prophylaxis for last one year for angina pectoris. Five days back, he was prescribed
T.Sildenafil 25mg for his erectile dysfunction. He was brought to the hospital emergency at
night as he developed severe chest pain and dizziness
a. What is the likely cause of his problem?
b. How will you manage and prevent such problem in future?

a. Both sildenafil and nitrates causes venodilation leading to aggravated hypotension.


This causes reflex tachycardia and increased oxygen demand leading to exacerbation
of angina.

b. Instead of sildenafil, alternate drugs like intra urethral administration of prostaglandin


analogue like alprostadil (PGE1) should be prescribed.

111
OSPE

OBJECTIVES
At the end of the objective structured practical
exercises, the learner shall be able to:

1. Understand the various routes of parenteral


drug administration and its application in
clinical practice.
2. Follow aseptic techniques in handling a
syringe and practice universal safety
precautions.
3. Measure the required volume of a drug in a
syringe and administer drugs through various
parenteral routes.

112
NMC- Competency No: PH 4.1a
Ex. No: 20 OSPE- Loading of drugs

Requirements:
Syringe (5 ml/2ml ), Needle with 22 to 24 G with 1.5 " (inch) length, Drug (Vial/ampoule ) ,cotton, spirit,

Procedure/Steps:

1. Take the drug container (vial) and check the following


Intended drug name and strength
Expiry date of the formulation
Content inside the container (presence of turbidity or any abnormal color)
2. Take a cotton swab with spirit and wipe the vial top. Wait for 30 seconds for the spirit to dry
3. Select the appropriate syringe as per the volume required for administration
4. Open the wrapper of the syringe without touching the needle
5. Open the needle cap and withdraw air of volume that is to be injected
6. Insert the needle into the vial cap and push the air into the vial
7. Invert the vial and withdraw the required volume
8. Change needle before administration to the patient.

113
NMC- Competency No: PH 4.1b
Ex. No: 21 OSPE- Intramuscular drug administration in mannequin
Requirements:
Syringe (5 ml/2ml ), Needle with 22 to 24 G with 1.5 " (inch) length, Drug (Vial/ampoule ) ,cotton, spirit,distilled
water , Emergency tray( adrenaline,avil and dexamethasone ), ET tube ,laryngoscope .

Site selection:
1.Lateral aspect of deltoid muscle just above the insertion point
2.Upper outer quadrant of gluteal region (gluteus medius and minimus)
3.Quadriceps muscles in thigh (for children)

Site preparation:
1.Make the patient to sit and lie down comfortably
2.Wash your hands and put on the gloves
3.Take a cotton swab soaked with surgical spirit
4.Wipe the site of injection in circular manner form centre to periphery
5.Dispose the cotton in a general waste dustbin
6.Allow the spirit to dry (at least 30 seconds)

Loading of drugs:
1.Take the drug in the vial or ampoule and check for “intended drug name and date of expiry”
2.Select the syringe with appropriate volume and needle with 22 to 24G with 1.5” inch length
3.Load the drug in the syringe with required volume

Steps to administer the drug intramuscularly:


1.Check whether the site of injection is dry and all the spirit has evaporated.
2.Without touching the site of injection, position the syringe in 90 degree angle close to the skin.
3.Quickly insert the needle deep into the muscle with same 90 degree angle.
4.Withdraw the plunger to check for any blood inside the syringe.
5.If no blood, then inject the drug steadily. Abort the procedure in case if blood is present inside the syringe while
withdrawing the plunger.
6.Pull out the needle and apply pressure over the muscle.
7.Rub gently the site of injection.
8.Dispose the syringe in the appropriate waste disposal bins.
9.Never recap the syringe. Dispose the needle with the needle destroyer.
10. Ask the patient to sit or lie down for 5 more minutes.

114
NMC- Competency No: PH 4.1c
Ex. No: 22 OSPE- Intravenous drug administration in mannequin
Requirements:

Syringe (5 ml/2ml ), Needle with 22 to 24 G with 1.5 " (inch) length, Drug (Vial/ampoule ) ,cotton, spirit,distilled
water , Emergency tray( adrenaline,avil and dexamethasone ), ET tube ,laryngoscope .

Site selection:
1.Antecubital vein in the antecubital fossa of the forearm 2.Cephalic vein
3.Venous arch over the dorsal aspect of the hand
4.Great saphenous vein 5.Femoral vein
6.Any prominent vein in case of emergency

Site preparation:
1.Make the patient to lie down comfortably and wash your hands and put on the gloves
2.Apply tourniquet above the site of selection to make the vein prominent.
3.Take a cotton swab soaked with surgical spirit
4.Wipe the site of injection in circular manner form centre to periphery
5.Dispose the cotton in a general waste dustbin
6.Allow the spirit to dry (at least 30 seconds)

Loading of drugs:
1.Take the drug in the vial or ampoule and check for “intended drug name and date of expiry”
2.Select the syringe with appropriate volume and needle with 22 to 24G with 1.5” inch length
3.Load the drug in the syringe with required volume

Steps to administer the drug intravenously:


1.Check whether the site of injection is dry and all the spirit has evaporated.
2.Without touching the site of injection, position the syringe in 45 degree angle close to the skin with bevelled edge
facing upwards.
3.Insert the needle into the vein with same 45 degree angle.
4.Withdraw the plunger to check for any blood inside the syringe.
5.If blood is present, then inject the drug steadily and slowly after the removal of tourniquet. Abort the procedure in
case blood is absent inside the syringe while withdrawing the plunger or swelling develops while injecting.
6.Pull out the needle and apply pressure over the vein with cotton swab.
7.Dispose the syringe in the appropriate waste disposal bins.
8.Never recap the syringe. Dispose the needle with the needle destroyer.
9. Ask the patient to sit or lie down for 5 more minutes.

115
NMC- Competency No: PH 4.1d
Ex. No: 23 OSPE- Subcutaneous drug administration in mannequin
Requirements:
Syringe (1 ml =40 U), Needle with 28 gauge 0.5 (inch) length, Drug (Vial/ampoule ) ,cotton, spirit,distilled water ,
Emergency tray( adrenaline,avil and dexamethasone ), ET tube ,laryngoscope .

Site selection:
1.Anterior abdominal skin
2.Skin over the thighs
3.Skin over the arm and forearm

Site preparation:
1.Make the patient to sit and lie down comfortably
2.Wash your hands and put on the gloves
3.Take a cotton swab soaked with surgical spirit
4.Wipe the site of injection in circular manner form centre to periphery
5.Dispose the cotton in a general waste dustbin
6.Allow the spirit to dry (at least 30 seconds)
Loading of drugs:
1.Take the drug in the vial or ampoule and check for “intended drug name and date of expiry”
2.Select the syringe with appropriate volume and needle with 28 G with 0.5” inch length
3.Load the drug in the syringe with required volume

Steps to administer the drug subcutaneously:


1.Check whether the site of injection is dry and all the spirit has evaporated.
2.Without touching the site of injection, position the syringe in 30 degree angle close to the skin.
3.Quickly insert the needle into the skin with same 30 degree angle.
4.Withdraw the plunger to check for any blood inside the syringe.
5.If no blood, then inject the drug steadily. Abort the procedure in case if blood is present inside the syringe while
withdrawing the plunger.
6.Pull out the needle and apply pressure over the muscle.
7.Never rub the site of injection as it makes the drug to get absorbed faster rate which is not intended.
8.Dispose the syringe in the appropriate waste disposal bins.
9.Never recap the syringe. Dispose the needle with the needle destroyer.

116
NMC- Competency No: PH 4.1e
Ex. No: 24 OSPE- Intradermal drug administration in mannequin
Requirements:
Syringe (1 ml =40 U), Needle with 28 gauge 0.5 (inch) length, Drug (Vial/ampoule ) ,cotton, spirit,distilled water ,
Emergency tray( adrenaline,avil and dexamethasone ), ET tube ,laryngoscope .

Site selection:
1.Ventral aspect of forearm and Skin over the back for hypersensitivity testing
2.Skin over the lateral aspect of deltoid for the administration of vaccines.

Site preparation:
1.Make the patient to lie down comfortably on the bed.
2.Wash your hands and put on the gloves
3.Take a cotton swab soaked with surgical spirit
4.Wipe the site of injection in circular manner form centre to periphery
5.Dispose the cotton in a general waste dustbin
6.Allow the spirit to dry (atleast 30 seconds)
Loading of drugs:
1.Take the drug in the vial or ampoule and check for “intended drug name and date of expiry”. Dilute the drug to
appropriate dilution.
2.Take the 1 ml syringe with 40 division units and needlewith 28 G with 0.5” inch length or specific needle designed
for intradermal purpose.
3.Load the drug after proper dilution in the syringe.
Steps to administer the drug Intradermaly:
1.Check whether the site of injection is dry and all the spirit has evaporated.
2.Without touching the site of injection, position the syringe in 0 to 10 degree angle almostparallel to the skin.
3.Insert the needle into the skin superficially with same <10 degree angle.
4.Withdraw the plunger to check for any blood inside the syringe.
5.If no blood, then inject the drug steadily. Abort the procedure in case if blood is present inside the syringe while
withdrawing the plunger.
6.Inject 0.1 ml of volume till a small bleb is raised.
7.Pull out the needle and encircle the bleb with a pen. The circle should be with a diameter of 1 cm approximately.
8.Never rub the site of injection
9.Dispose the syringe in the appropriate waste disposal bins.
10.Never recap the syringe. Dispose the needle with the needle destroyer.
11.Wait for 15 minutes and observe for any hypersensitivity reactions. Redness or wheel crossing the circle will be
taken positive for allergic tendency to the drug.

117
NMC- Competency No: PH 4.1f
Ex. No: 25 OSPE- Application of eye drop in mannequin

Precautionary measures:
1.Make the patient to sit or lie down comfortably on the bed.
2.Wash the hands thoroughly with soap and put on the gloves.
3.Take the eye drop and check the “intended drug name” and date of expiry.

Requirements:
Eye drops , Cotton, Glove and emergency tray .

Steps/procedure:
1.Ask the patient to tilt his/her head slightly backwards (around 30 degrees)
2.With proper neck extension, ask the patient to look upwards.
3.Shake the eye drop bottle and open the cap.
4.Pour the recommended drops into the eye.
5.Ask the patient to close the eyes as soon as the eye drop is applied.
6.Immediately press the medial canthus for 30 seconds. This will prevent the drug from entering into the nose via
nasolacrimal duct.
7.Ask the patient to keep the eye closed for 5 minutes.
8.Blinking should be avoided as it will increase the conduct of eye drops into nasolacrimal duct resulting in loss of
efficacy and emergence of systemic side effects of the drug.
9.Ask the patient to convey immediately if he/she develops irritation or itching in the eye.
10.Recap the eye drops tightly and store it in a cool dark space for future use.

118
NMC- Competency No: PH 4.1g
Ex. No: 26 OSPE- Administration of aerosol drug via nebulizer

Precautionary measures:
1.Make the patient to sit or lie down comfortably on the bed.
2.Take the drug in the available formulation and check the “intended drug name” and date of expiry.
3.Dilute the drug with appropriate diluting fluid (distilled water or normal saline).
4.No dilution is required for drugs available as readymade nebulizer solution formulation

Requirements:
Nebulizer, Power socket with supply , mannequin, aerosol drug , distilled water , mask .

Steps/procedure:
1.Connect the nebulizer to the power socket.
2.Take the nebulizer cup at the distal end of the air tube and check whether it is cleaned well for ready to use.
3.Pour the drug mixed with the appropriate dilution fluid into the nebulizer cup
4.Connect the mask to the mouth piece which has been fitted with the nebulizer cup.
5.Place the mask over the face of the patient and check whether nose and mouth have been covered by the
mask properly.
6.Ask the patient about the convenience of the mask and ascertain it.
7.Switch on the nebulizer.
8.Emergence of aerosol in the mask indicates that the drug solution is being aerosolized by the nebulizer.
9.Ask the patient to breathe in and exhale slowly and deeply
10.Keep the nebulizer for 20 minutes and check the response to the drug.
11. Switch of the nebulizer and remove the mask.
12. Dismantle the parts used. Clean the parts with hot water and dry it well so that it can be used for next
patient.

119
NMC- Competency No: PH 4.1h
Ex. No: 27 OSPE- Administration of aerosol drug via MDI with
spacer

Precautionary measures:

1.Make the patient to sit or lie down comfortably on the bed.


2.Take the drug in the inhaler formulation and check the “intended drug name” and date of expiry.

Requirements:
Metered dose inhaler/ spacer ,

Steps/procedure:
1.Remove the mouth piece cover and shake the inhaler well
2.Hold the inhaler upright with the thumb on the base and one or two fingers on the canister.
3.Instruct the model to exhale the air from the lungs
4.Keep the mouth piece in the mouth and close the lips
5.Press the canister to release one dose
6.Instruct to breath steadily and deeply
7.Remove the inhaler and ask to hold breath for 10 to 20 seconds
8.Breathe out slowly
9.Replace the mouth piece cover
10.Instruct to gargle the mouth after inhalation

120
COMPUTER ASSISTED LEARNING
(CAL)

OBJECTIVES
At the end of the CAL practical exercises, the
learner shall be able to:

1. Use the simulation software of various animal


experiment

2. comprehend action of agonists and


antagonists on various receptors in animal
tissues by observing the effect using CAL
software

3. Record, analyse and interpret observation


obtained using CAL software

121
NMC- Competency No: PH 4.2a
Ex. No: 28 CAL – Effects of drugs on heart rate and blood pressure in
dogs.
A. Aim:
 To demonstrate the effect of various drugs on the heart rate and blood pressure of
anesthetized dog.

B. Requirements:
 Adult Male Mongrel dog weighing approximately 15Kg
 Chloralose (100mg/Kg) for anaesthesia
 Dissection sets
 Kymograph and physiograph
 Epinephrine (2 mcg/Kg for alpha action and 0.1mcg/Kg for beta action)
 Norepinephrine (3 mcg/kg)
 Isoprenaline (3 mcg/Kg)
 Acetylcholine ( 5 mcg/kg)
 Histamine (3 mcg/kg)
 Ephedrine (100 mcg/Kg)
 Phentolamine (1000 mcg/Kg)
 Propranolol (1000 mcg/Kg)
 Atropine (750 mcg/Kg)
 Mepyramine (5000 mcg/Kg)
 Cimetidine (5000 mcg/Kg)
 Normal saline (to act as neutral)

C. Procedure (Actual):
 A dog is weighed and anaesthetized using intravenous chloralose (100 mg/kg of body
weight). It is fixed on a dog table in supine position. Right femoral vein is cannulated
with a catheter to inject drugs. Its neck is dissected to expose carotid arteries and vagus
nerves. Left common carotid is inserted with an arterial cannula connected to a mercury
manometer or to a pressure transducer, which is connected to a polyrite / physiograph.
A chart recorder or a kymograph is set up to receive signals from the polyrite/
physiograph to produce graphical representation of changes in blood pressure.
 The left side vagus is cut into central and peripheral ends for applying electrical
stimulation later in the experiment. The right common carotid is identified and exposed
for later use. Drugs are injected (one by one) into the femoral vein and BP is recorded
on the chart. The heart rate (beats/min) is counted and noted on the chart. Drug
administration is also marked

D. Procedure (Using CAL):

 Software Used: Expharm (Offline-trial version)


 The CAL software was opened and the above experiment was selected. The procedures
like stimulation of central vagus, stimulation of peripheral vagus, and carotid occlusion
were done by clicking the appropriate buttons. The changes in the heart rate and blood
pressure were recorded and tabulated. Drugs were injected one by one and the
changes in parameters were observed. Normal saline was injected between each drug
to record the baseline values.
122
NMC- Competency No: PH 4.2a
Ex. No: 28 CAL – Effects of drugs on heart rate and blood
pressure in dogs.
E. Observation:

Heart rate Mean arterial Net Change


(bpm) pressure (mm (Inc/Dec)
S. Drug/ Dose
of Hg)
No Procedure (mcg/kg) Base After
Base After HR MAP
line drug
line drug
1 Carotid occlusion NA 150 160 90
130 10 40
Central vagus
2 NA 150 90 140 10 50
stimulation 160
Peripheral vagus 50 -40
3 NA 150 120
90
-30
stimulation
4 Epinephrine 0.1 150 150 90 90 0 0
5 Epinephrine 3 150 130 -->150 90 130-->90 -- --
6 Norepinephrine 3 150 120 90 160 -30 40
7 Isoprenaline 3 150 190 90 60 40 -30
8 Acetylcholine 5 150 130 90 60 -30 -30
9 Histamine 3 150 180 90 50 30 -40
10 Ephedrine 200 150 190 90 200 40 110
11 Phentolamine 1000 150 170 90 70 20 -20
12 Propranolol 1000 150 140 90 70 -10 -20
13 Atropine 750 150 190 90 90 40 0
14 Mepyramine 5000 150 150 90 90 0 0
15 Cimetidine 5000 150 150 90 90 0 0
Atropine followed Atropine 1500
16 160 180 90 170 20 80
by Acetylcholine + Ach 150
Propranolol Propranolol
17 followed by 1000 + 150 150 90 90 0 0
isoprenaline Isoprenaline 3
Phentolamine Phentolamine
18 followed by 1000 + 150 160 90 70 10 -20
epinephrine Epinephrine 3
Phentolamine Phentolamine
19 followed by 1000 + nor 150 150 90 90 0 0
norepinephrine Epinephrine 3
Mepyramine Mepyramine
20 followed by 5000 + 150 150 90 90 0 0
histamine Histamine 3

123
NMC- Competency No: PH 4.2a
Ex. No: 28 CAL – Effects of drugs on heart rate and blood
pressure in dogs.

F. Discussion: (Answer the following questions)

1. When both the carotids are occluded for 15 seconds, what changes are noted in the
mean arterial pressure? Explain the mechanism.

When both carotids are occluded, there will be increase in heart rate and mean arterial
pressure. This is because occlusion of carotid will activate the carotid body chemoreceptors
resulting in increased vasomotor tone leading to increase in mean arterial pressure

2. What are the changes observed with the electrical stimulation of central cut end and
peripheral end of vagus? Explain the mechanism.

When central cut end of vagus is stimulated, increased heart rate and mean arterial pressure
are noted. This is because the central cut end of vagus is closely attached with sympathetic
nerves and electrical stimulation leads to increased vasomotor tone resulting in increased
HR and MAP

When peripheral cut end of vagus is stimulated, decreased heart rate and blood pressure are
noted. This is because, the peripheral cut end of vagus releases Ach upon electrical stimulation
leading to bradycardia and fall in MAP.

3. List the types, location and physiological role of receptors involved in sympathetic and
parasympathetic system.

Refer KDT 8th edition Page 112, 113 and 139

124
NMC- Competency No: PH 4.2a
Ex. No: 28 CAL – Effects of drugs on heart rate and blood
pressure in dogs.

4. Carefully observe the effect of drugs on the dog’s heart rate and MAP in the given chart
and answer the following (Epi = Epinephrine; Nepi = Nor-epinephrine)

a. What is the response noted with administration of epinephrine 6mcg/kg dose?


Explain the mechanism.
The response noted is called biphasic response. There will initial rise and later fall in MAP after
administration of relatively higher doses of epinephrine. Biphasicresponse occurs due to difference
in affinity of adrenergic receptor to epinephrine.
Initial high concentration of adrenaline stimulates alpha and beta receptors but alpha activity
predominates leading to increase in MAP. Later after rapid uptake, the concentration falls and only
beta receptors are stimulated leading to fall in MAP.

b. Why epinephrine does not produce the same response at 0.1mcg/kg dose
similar to 6mcg/kg dose? Explain with reasons.
Stimulation of alpha receptors requires relatively higher concentration of the adrenalin.
Hence adrenaline at 0.1mcg/Kg does not produce response similar to 6mcg/Kg which
produces 'biphasic response'.

c. Why the response produced with norepinephrine 6mcg/kg dose does is higher
than that of epinephrine 6mcg/kg dose? Explain the mechanism.

Norepinephrine predominately acts on the alpha receptors with almost minimal action
on the beta receptors. This results in effective vasoconstriction.

Adrenaline acts on both alpha and beta receptors at high concentration and hence
vasoconstriction produced by the alpha receptors are counteracted to some extend
by vasodilation by beta receptors.

Hence the increased MAP produced with norepinephrine is higher than epinephrine
at the same dose.
125
NMC- Competency No: PH 4.2a
Ex. No: 28 CAL – Effects of drugs on heart rate and blood
pressure in dogs.

5. Carefully observe the effect of drugs on dog’s heart rate and MAP in the given chart and
answer the following questions (Epi=Epinephrine; Nepi=Norepinephrine;
Phen=Phentolamine)

a. Why does phentolamine decrease MAP and increases HR? Explain the
mechanism.
Phentolamine is an alpha 1&2 antagonist and cause vasodilation. Vasodilation causes
decreased Total peripheral resistance and thereby reduces DBP and MAP. This causes
reflex tachycardia and hence HR increases.

b. Why nor-epinephrine does not produce any response after phentolamine?


Explain with reasons.
Noradrenaline acts predominantly on the alpha receptors in vivo. Since phentolamine,
being an alpha-1 &2 antagonists, blocks the alpha receptors, norepinephrine did not
produce any responses.

c. Why does the response produced by epinephrine differ ‘before’ and ‘after’
phentolamine administration? Explain the mechanism. What is this
phenomenon called?
This phenomenon is called 'Vasomotor reversal of dale'. When an non selective alpha
blocker is given before epinephrine, only fall in MAP is seen instead of biphasic response
is noted.
This is because, the alpha receptors are blocked and epinephrine acts only on beta
receptors leading to only fall in MAP.

126
NMC- Competency No: PH 4.2a
Ex. No: 28 CAL – Effects of drugs on heart rate and blood
pressure in dogs.

6. Carefully observe the effect of drugs on dog’s heart rate and MAP in the given chart and
answer the following questions(Ephe = Ephedrine; Phen=Phentolamine)

a. How does ephedrine increase heart rate and blood pressure?


Ephedrine is a mixed acting sympathomimetic drug. It directly stimulates alpha and
beta adrenergic receptors and indirectly releases NA in the neuronal synapse. Because
of these actions HR and MAP are increased.

b. Why the response decreases rapidly with subsequent doses of ephedrine? What
is this phenomenon called?
This phenomenon is called tachyphylaxis. This occurs because the neuronal pool of
NA available for the displacement by the ephedrine (indirect action) is small and hence
this action is quickly saturated.

c. Why ephedrine does not produce any effect after phentolamine administration?
Explain the mechanism.

Ephedrine is an alpha agonist and phentolamine is an alpha antagonist. The alpha


adrenergic receptors are blocked by phentolamine and hence ephedrine cannot act
resulting in no response.

127
NMC- Competency No: PH 4.2a
Ex. No: 28 CAL – Effects of drugs on heart rate and blood
pressure in dogs.

7. Carefully observe the effect of drugs on dog’s heart rate and MAP in the given chart and
answer the following questions(Epi =Epinephrine; Iso =Isoprenaline; Prop = Propranolol)

a. How does propranolol decrease heart rate and blood pressure? Explain the
mechanism.
Propranolol being a beta adrenergic receptors reduces HR by blocking the beta-1
adrenergic receptor in the myocardium. Cardiac output reduces leading to decrease
in SBP and MAP. It also blocks the renin release in kidneys resulting in reduction of MAP.

b. Why does the response produced by epinephrine differ ‘before’ and ‘after’
propranolol administration? Explain the mechanism. What is this phenomenon
called?
This phenomenon is called 'Re-reversal of Vasomotor reversal of dale'.
When a non selective beta blocker is given before epinephrine, only rise in MAP is seen
instead of biphasic response is noted.
This is because, the beta receptors are blocked and epinephrine acts only on alpha
receptors leading to only rise in MAP.

c. Why isoprenaline does not produce any response after propranolol


administration? Explain the mechanism.
Propranolol is a non selective beta blockers and isoprenaline is a non selective
beta agonist. Since all beta receptors are blocked by propranolol, isoprenaline does
not produce any response.

128
NMC- Competency No: PH 4.2a
Ex. No: 28 CAL – Effects of drugs on heart rate and blood
pressure in dogs.

8. Carefully observe the effect of drugs on dog’s heart rate and MAP in the given chart and
answer the following questions (Ach = Acetylcholine; Atro = Atropine)

a. How does acetylcholine decrease heart rate and blood pressure? Explain the
mechanism.
Ach stimulates M2 receptor present in heart leading to opening of K+ channels and decrease
action potential in SA node. This causes reduced HR and thereby decreases cardiac output.
SBP is reduced due to decreased cardiac output and hence MAP falls.

b. Why the response produced by acetylcholine is very short? Explain with reasons.
Ach is rapidly deactivated by plasma cholinesterase enzyme and hence the response
produced by ACh is very short.

c. How does atropine increase heart rate? Why atropine causes only minimal
increase in the MAP?
Atropine being a muscarinic antagonist, blocks M2 receptor present in the myocardium
This reduces the ' basal inhibitory vagal tone' in heart and hence tachycardia occurs.
Blood vessels does not have parasympathetic nerve supply and atropine administration
will not affect blood vessels. Thus slight increase in MAP is due to increased HR and
increased cardiac output by atropine.
d. Why does acetylcholine cause increase in heart rate and blood pressure after
atropinization? Explain the mechanism.
After atropinization (relatively higher dose of atropine like 1500 mcg/Kg), all the muscarinic
receptors are blocked. Thus when acetlycholine at relatively high dose (150 mcg/Kg) is given
now, it will stimulate only the ganglionic nicotinic receptors leading to increased sympathetic
discharge leading to increased HR and MAP.

129
NMC- Competency No: PH 4.2a
Ex. No: 28 CAL – Effects of drugs on heart rate and blood
pressure in dogs.

9. Carefully observe the effect of drugs on dog’s heart rate and MAP in the given chart and
answer the following questions (Hist = Histamine; Mep = Mepyramine;Cime-Cimetidine)
10.

a. How does histamine increase heart rate and decrease blood pressure? Explain
the mechanism.
Histamine causes vasodilation by acting on both H1 and H2 receptors present in the blood
vessels. Vasodilation causes reflex tachycardia and reduced TPR leading to reduced MAP.

b. Why mepyramine and Cimetidine do not produce any response? Explain the
mechanism
Mepyramine is a H1 antagonist and cimetidine is a H2 antagonist. These drugs do not
have any inherent action. Vascular tone regulation under normal physiological condition
is mainly by adrenergic system and not by histamine. Thus these antagonists do not
produce any response.

c. Why response produced by the histamine is altered after the administration of


‘mepyramine’?
H1 receptor produce rapid vasodilation and H2 receptor produces slow sustained vasodilation.
Mepyrine being a H1 antagonist blocks the H1 receptor present in the blood vessels
and hence the action on it by histamine is prevented. Only H2 action is produced and hence the
response is altered.

d. Why histamine does not produce any response after mepyramine and
Cimetidine administration? Explain the mechanism.
Both H1 and H2 receptors are blocked by mepyramine and cimetidine and hence histamine
cannot act on the blood vessels. Thus no response is noted.

130
NMC- Competency No: PH 4.2b
Ex. No: 29 CAL – Effects of drugs on rabbit’s eye.
A. Aim:
 To demonstrate the effect of various drugs on the size of the pupil, light reflex, corneal
reflex, and intraocular tension in the rabbit’s eye.

B. Requirements:
 Adult New Zealand Albino Rabbit weighing approximately 3 – 5 Kg without any eye
diseases.
 Rabbit holder
 Measuring scale
 Droppers, torch, and Cotton Wool
 Normal saline
 Physostigmine 0.5%
 Atropine sulphate 1%
 Ephedrine 0.5%
 Adrenaline hydrochloride 0.1%
 Lignocaine hydrochloride 1%

C. Procedure (Actual):
 The rabbit is fixed in a rabbit holder so that its head is outside. Too much of neck
compression while fixing should be avoided. The eyelashes are then clipped.
 The baseline reading for pupil size (using measuring scale), light reflex (with torch), and
corneal reflex (using cotton swab) are measured and noted down.
 One eye of the rabbit is kept as control and the other eye is used to test the action of
drugs. Fresh rabbit should be taken to test another drug.
 The lower eye lid of the rabbit is retracted down and a pouch is formed (pouch
method). Using pouch method, drugs/normal saline is applied and the medial canthus is
pressed for 10 seconds. The above measurements are taken again after 10 – 15
minutes.

D. Procedure (Using CAL):

 Software Used: Expharm (Offline-trial version)


 The CAL software was opened and the above experiment was selected. The right eye is
kept as control and left eye is kept as test eye. By clicking and dragging the
measurement scale to the pupil, the pupil size was measured. By clicking and dragging
the torch towards the pupil, the light reflex was noted down. By clicking and dragging
the cotton wool towards cornea, the corneal reflex was noted down. The baseline
readings of both the eyes were entered in the table
 By clicking drug selection button, normal saline was selected and applied to the right
eye. Test drug was selected and applied to the left eye. The same procedure as
explained above was performed, the measurements after the drugs are entered in the
table, and the conclusion was arrived. New rabbit was selected for testing the next
drug.

131
NMC- Competency No: PH 4.2b
Ex. No: 29 CAL – Effects of drugs on rabbit’s eye.
E. Observation:

Intraocular Light reflex Corneal reflex Pupil size


Rabbit tension (Present/abse (Present/abse (mm)
Eye Drug
No (mm of Hg) nt nt)
Bef Aft Bef Aft Bef Aft Bef Aft
Right Saline N
N
P P P P 7 7
1 Epinephrine
Left P P P 7
N L P 10
0.1%
Right Saline N N P P P P 7 7
2 Ephedrine
Left N N
P P P P 7 10
0.5%
Right Saline N N P P P P
7 7
3
Left Atropine 1% N N P A P P 7 11
P 7
Right Saline N N P
P P 7
4
Left Lignocaine 1% N N P P
P A 7 7

Right Saline N N P P P P 7 7
5 Physostigmine
Left N L P P P P 7 3
0.5%

F. Conclusion:

1. Epinephrine, Ephedrine causes active mydriasis and atropine causes passive mydriasis
2. Physiostigmine is a miotic drug
3. Epinephrine and Physostigmine reduces intra-ocular tension
4. Lignocaine is a local anesthetic drug

132
NMC- Competency No: PH 4.2b
Ex. No: 29 CAL – Effects of drugs on rabbit’s eye.
G. Discussion:
1. Carefully observe the effect of drugs on rabbit’s eye in the given chart and answer the
following questions.

a) How does atropine cause mydriasis? Explain the mechanism


Atropine blocks the M3 receptor present in the circular smooth muscles (constrictor pupillae)
of iris and causes paralysis. Thus, unopposed dilator pupillae action lead to mydriasis.

133
NMC- Competency No: PH 4.2b
Ex. No: 29 CAL – Effects of drugs on rabbit’s eye.

b) Why light reflex is absent with atropine administration but present with phenylephrine?
Explain the mechanism
Atropine causes paralyis of constrictor pupillae in iris by blocking M3 receptor in it. The final
pathway of light reflex is constriction of constrictor pupillae via short ciliary nerve. Since atropine
blocks this light reflex is absent.
Phenylephrine does not block the constrictor pupillae and hence light relfex is preserved.

c) What is meant by ‘active mydriasis’ and ‘passive mydriasis’? Give one example for each.
Causing mydriasis by blocking (paralysing) the constrictor pupillae is called passive mydriasis
Example: atropine

Causing mydriasis by active constriction of dilator pupillae is called active mydriasis


Example: Phenylephrine, Ephedrine.

d) Mention the difference between cycloplegic and mydriatic agent.


The drug that caused increase in pupil size only is called mydiratic agent. Ex: Phenylephrine

The drug that causes increased pupil size and paralysis of muscles responsible for accommodation
is called Cycloplegic Ex: Atropine

e) Mention the mydriatic drugs used clinically


Homatropine
Cyclopentolate
Tropicamide
Atropine
Phenylephrine.

134
NMC- Competency No: PH 4.2b
Ex. No: 29 CAL – Effects of drugs on rabbit’s eye.
2. Carefully observe the effect of drugs on rabbit’s ey
eyee in the given chart and answer the
following questions.

a) How does physostigmine causes miosis? Explain the mechanism


Physostigmine causes miosis by stimulation of M3 receptors present in the constrictor
pupillae of iris

135
NMC- Competency No: PH 4.2b
Ex. No: 29 CAL – Effects of drugs on rabbit’s eye.

b) Why corneal reflex is absent with lignocaine but present with atropine? Explain
Lignocaine is an local anesthetic agent which upon instillation on eye causes total anesthesia
in cornea. Thus corneal reflex is absent with lignocaine. Atropine does not cause corneal
anesthesia and hence corneal reflex is preserved with atropine.

c) What is the difference between the mydriasis produced by epinephrine and atropine?
Explain
Epinephrine produces active mydriasis and light reflex is present
Atropine produces passive mydriasis and cycloplegia. Light relfex is absent with atropine.

d) Though both epinephrine and ephedrine are mydriatics, why the intra ocular pressure is
reduced with epinephrine but not with ephedrine? Explain
Ephedrine is a mixed acting sympathomimetic drug and the less minimal action on the
production of aqueous and outflow.

Epinephrine is a direct sympathomimetic drug and produces good reduction in aqueous


formation and increase in outflow activity. Thus IOP reduction is seen with epinephrine
and not with ephedrine.

e) Physostigmine is miotic and epinephrine is mydriatic. Explain how both the drugs
reduce intraocular pressure?
Physostigmine increases the patency of trabeculae by increasing the ciliary muscle tone

Epinephrine decreases the aqueous formation (via alpha 1 and alpha 2 receptor) and
increase in hydraulic conductivity of trabecular filtering cells (via beta 2 receptor)

136
NMC- Competency No: PH 4.2c
Ex. No: 30 CAL – Effects of drugs on frog’s oesophagus.
A. Aim:
 To demonstrate the effect of various drugs on the ciliary motility of frog’s oesophagus.

B. Requirements:
 Adult frog
 Poppy seeds
 Frog board
 Stop watch
 Acetylcholine 10%
 Physostigmine 10%
 Atropine 0.1%
 Frog Ringer solution

C. Procedure (Actual):
 The frog is pithed and lower jaw is removed. The esophagus is slit open from buccal
cavity to the stomach and everted to fix it on a wooden board with pins. Blood is wiped
away by a cotton swab dipped in frog Ringer solution.
 The surface is moistened with frog Ringer. A poppy seed is placed at the cephalic end
and its movements and time taken to travel a fixed distance on the esophagus are
observed. The drugs are applied and the time taken to travel by the poppy seeds is then
compared with baseline value and between the drugs.

D. Procedure (Using CAL):

 Software Used: Expharm (Offline-trial version)


 The CAL software was opened and the above experiment was selected. Frog-Ringer on
the surface of the oesophagus was instilled by clicking ‘instil drug’ button. A poppy seed
was placed at the cephalic end of oesophagus by clicking and dragging the seed from
poppy seed box. The seed starts moving due to ciliary motility.
 When the seed crosses the starting point (cephalic end pins), stop watch / clock was
started. When the seed reaches the distal pins, clock was stopped. The time taken by
the seed to travel was noted down. Three readings were taken in the same manner.
 The drugs were selected and the same measurement was recorded. The findings were
tabulated and the conclusion was arrived.

137
NMC- Competency No: PH 4.2c
Ex. No: 30 CAL – Effects of drugs on frog’s oesophagus.
E. Observation:

Time taken for the poppy seed Inference


to travel (seconds) ciliary
Frog Mean
Drugs motility
No Reading Reading Reading (seconds)
increased/
1 2 3
decreased

Ringer 32 33 31 31 no
change
1
Acetylcholine 10% 22 21 20
21 Increased

No
Ringer 31 32 33
31
change
2
Physostigmine 10% 16 17 15 16 Increased

Ringer 31 31 31 31 No
change
3
Atropine 0.1% 57 Decreased
56 58 56

Ringer 32 31 33 32 No
change

4 Atropine 0.1%
followed by 31
33
32
No
32 Change
Acetylcholine 10%

29 No
Ringer 30 31 30 change
5 Physostigmine 10%
Greatly
followed by 14 14 14
14 increased
acetylcholine 10%

F. Conclusion:

Atropine reduces ciliary motility and acetylcholine increases ciliary motility


Physostigmine increases ciliary motility indirectly but augmenting endogenous ACh

138
NMC- Competency No: PH 4.2c
Ex. No: 30 CAL – Effects of drugs on frog’s oesophagus.

G. Discussion:
1. Carefully observe the effect of drugs on frog’s esophagus in the given chart and answer
the following questions.

a) Why does acetylcholine decrease the travel duration of poppy seed in frog’
frog’s
esophagus? Explain the mechanism
Ach increases ciliary motility and hence duration of poppy seed travel is reduced.
139
NMC- Competency No: PH 4.2c
Ex. No: 30 CAL – Effects of drugs on frog’s oesophagus.

b) Why does physostigmine+ acetylcholine shorten the duration of poppy seed travel in
frog’s esophagus than acetylcholine alone? Explain
Physostigmine blocks the AChE action and hence Ach activity is boosted when compared with
ACh alone. Thus duration of poppy seed travel is shorter in combination of physostigmine+Ach
than Ach alone.

c) Why does atropine prolong the duration of poppy seed travel in frog’s esophagus?
Explain the mechanism
Atropine is a muscarinic antagonist and blocks the M3 receptor present in cilia leading to
reduced ciliary motility. Thus duration of poppy seed travel in increased.

d) What happens to poppy seed time travel when only physostigmine alone is instilled in
frog esophagus? Explain the findings with reason.

When physostigmine alone is instilled, the duration of poppy seed travel is relatively decreased
due to boosting of endogenous ACh activity.

e) Enumerate the sites in the human body where cilia plays important role. Why
acetylcholine is not used clinically?

Respiratory tract and in Fallopian tube

The half life of Ach is very short due to rapid inactivation by the plasma cholinesterase enzyme

140
NMC- Competency No: PH 4.2d
Ex. No: 31 CAL – Effects of drugs on isolated frog’s heart.
A. Aim:
 To demonstrate the effect of various drugs on the isolated frog’s heart.

B. Requirements:
 Adult frog heart large size
 Starling’s heart lever, stand
 Kymograph and smoked drum, or physiograph with chart recorded
 Frog Ringer solution
 Epinephrine 10mcg/ml
 Nor-epinephrine 10mcg/ml
 Isoprenaline 10 mcg/ml
 Propranolol 1mg/ml
 Acetylcholine 10mcg/ml
 Atropine sulphate 100mcg/ml
 Calcium chloride 10mg/ml
 Potassium chloride 10mg/ml

C. Procedure (Actual):
 A frog is pithed and dissected to expose the heart. Sinus venosus is cannulated using a
glass cannula and secured with thread. Then the heart is isolated along with the cannula
and perfused with Ringer through the sinus venosus.
 A curved needle is inserted in the apex and attached to a heart lever for recording
contractions on the smoked paper pasted over the drum of a kymograph. [Instead of
heart lever, drum and kymograph, a physiograph can be used. Force transducer
replaces the heart lever and is connected to the heart. Chart paper and pens are used
for recording.
 Drugs are injected (one by one) into the Ringer piercing the rubber tube and
contractions are recorded on the chart. The heart rate (beats/min) is counted and noted
on the chart. Drug administration is also marked.

D. Procedure (Using CAL):

 Software Used: Expharm (Offline-trial version)


 The CAL software was opened and the above experiment was selected. The drugs were
selected one-by-one by clicking the select button and injected by clicking inject button.
 The parameters like amplitude (force of contraction), tone and heart rate were noted
before and after drug administration.
 The findings were tabulated and the conclusion was arrived.

141
NMC- Competency No: PH 4.2d
Ex. No: 31 CAL – Effects of drugs on isolated frog’s heart.
E. Observation:

Heart rate (bpm)


S.No Drug After Amplitude Tone Inference
Baseline
drug
No No ---
1 Frog Ringer 60
60 change change

2 Epinephrine 2mcg Inc cardiac


60 68 Increase stimulant
Cardiac
3 Nor-epinephrine 2mcg 60 58 Increase Inc stimulant

4 Isoprenaline 2mcg 60 85 Increase Inc Cardiac


stimulant
60 Dec Cardiac
5 Propranolol 200mcg 44 Decrease
depressant

60 Cardiac
6 Acetylcholine 2mcg 41 Dec Dec depressant

7 Atropine 20mcg 62 No No Cardiac


59 change Change stimulant
Calcium chloride
8 -- -- Systolic arrest
2000mcg 60 1

Potassium chloride
9 61 1 -- --- Diastolic arrest
2000mcg

F. Conclusion:

1. Epinephrine, nor-epinephrine, isoprenaline and atropine are cardiac stimulant


2. Propranolol and acetylcholine are cardiac depressants
3. Calcium causes systolic arrest and potassium causes diastolic arrest

142
NMC- Competency No: PH 4.2d
Ex. No: 31 CAL – Effects of drugs on isolated frog’s heart.

G. Discussion:
1. Carefully observe the effect of drugs on frog’s esophagus in the given chart and answer
the following questions.

a) Define tone. How will measure the tone, force of contraction, and heart rate in this
experiment?
Tone is defined as the residual tension in the relaxed muscle. It is continuous and passive partial
contraction of muscle during resting state. Tone is measured by shifting of midline of the graph.
Force of contraction is measured by increase in height of graph. Heart rate is measured by
frequency of cycles per second in the recording.

143
NMC- Competency No: PH 4.2d
Ex. No: 31 CAL – Effects of drugs on isolated frog’s heart.

b) What are the effects of epinephrine on heart rate, tone, and force of contraction?
Explain the mechanism
Epinephrine stimulates beta 1 adrenergic receptors in heart leading to increased cAMP levels
and causes
1. Increase in slope of slow diastolic depolarization in SA node - tachycardia (+ve Chronotropic)
2. Acceleration in rate of conduction (Positive Dromotropic)
3. Increase in cytosolic calcium load (Increase in tone)
4. Increase in opening of calcium channels to increase force of contraction (positive inotropic)
5. Increase in rate of reuptake of calcium into SR (positive luciotropic)

c) How does propranolol and acetylcholine decrease the heart rate, tone, and force of
contraction in isolated frog’s heart?

Propranolol decreases the HR, tone and force of contraction by blocking the beta-1 adrenergic
receptors and thereby decreasing the intracellular cAMP levels.

Acetylcholine decreases the HR, tone and force of contraction by stimulating the M2 receptor
and opens the K+ channels that causes hyperpolarization in myocardial cells.

d) Why does calcium cause systolic arrest and potassium cause diastolic arrest? Explain the
mechanism. Mention the clinical application of this phenomenon.
The amount of calcium ions in t-tubules is directly proportional to its concentration in extracellular fluids.
Therefore, the availability of calcium ions to cause cardiac contraction is directly dependent on the
extracellular fluid calcium ion concentration and hence large amount of rapid calcium administration
causes systolic arrest.
Elevated extracellular potassium concentration (10-40mM) alters resting potential for myocyte,
from -85mV to between -65mV and -40mV, leading to fast sodium channels inactivation.
The new RMP blocks conduction of myocardial action potential, thereby inducing diastolic arrest.

Clinical application: Cardioplegic solution

e) Why does atropine cause insignificant increase of heart rate in isolated frog’s heart?
Explain the mechanism.

In isolated frog heart, vagus nerve will be absent and hence the there is no baseline inhibitory
vagal tone. Atropine increases heart rate in in-vivo by abolishing the inhibitory vagal tone.
However this mechanism fails in isolated heart preparations. Hence atropine has insignificant
increase of heart rate.

144
NMC- Competency No: PH 4.2e
Ex. No: 32 CAL – Bioassay of histamine on guinea pig ileum.
A. Objectives:
 To understand the basic principles and importance of bioassay.
 To use the CAL software for determining the concentration of histamine using bioassay.

B. Requirements:
 Guinea pig ileum
 Organ bath
 Kymograph and smoked drum or physiograph with chart recorder.
 Stop watch
 Tyrode solution
 Histamine solution

C. Procedure (Actual):
 A guinea-pig is sacrificed, abdomen is opened and the ileum is isolated. Mesenteric
attachment is released and the ileum is cut across and placed in Petri dish containing
Tyrode's solution.
 A small segment of ileum (about 5 cm) is cut and cleaned and one end is fixed to a
tissue holder with the help of a thread. The other end is tied to a thread and the tissue
holder along with the tissue is placed inside an organ bath. Then the thread is attached
to a force transducer which is connected to a physiograph.
 The water bath is filled with water and warmed by a heater cum thermostat to maintain
a temperature of 37 ° C. The organ bath filled with Tyrode's solution is connected to a
reservoir. The organ bath can be emptied by opening the outlet and refilled with
Tyrode's solution from reservoir.
 Ileum and tissue holder are placed in the organ bath. The upper end of the ileum is
attached to a force transducer by a long thread. An aerator is connected to the tissue
holder. The tissue holder is fixed in position tightly with clamps and attached to an
aerator through a rubber tube. The air bubbles out at the other end inside the Tyrode's
solution.
 Histamine is added in graded manner and the dose response curve is constructed. From
this standard dose (S) is selected for matching assay. The unknown-U (solution
containing unknown histamine concentration) is added to the organ bath and the
response is noted. The S and U are added alternatively and the volume of U is adjusted
so that it matches with the S. After matching, the 1/2S and 2S are also tested for
matching with the volume of 1/2U and 2U, respectively. Using simple mathematical
calculation, the concentration of histamine in the U is determined.

D. Procedure (Using CAL):


 Software Used: Expharm (Offline-trial version)
 The CAL software was opened and the above experiment was selected. DRC was
constructed and the height of the response was measured by clicking the scale. The
standard S was selected between 30-70% of response. Matching assay was performed
as explained above and the concentration of histamine was determined.

145
NMC- Competency No: PH 4.2e
Ex. No: 32 CAL – Bioassay of histamine on guinea pig ileum.
E. Observation:

The height of maximum response noted with histamine on guinea pig ileum (m)= ____cm
8

% of the response from the


Dose of histamine Height of the response (h)
S.No maximal response
(mcg) cm
= (h/m)*100
1 0.1 1 12.5
2 0.2 1.25 15.6
3 0.4 1.5 18.75
4 0.8 1.75 21.8
5 1.6 2 25
6 3.2 2.25 28.1
7 6.4 2.5 31.25
8 12 3 37.5
9 24 4 50
10 48 6.5 81.25

11 96 8 100

F. Calculation:
24
The dose of standard (S) selected = ____mcg
The volume of unknown (U) that matches with the standard (S) = 0.32
___ml

0.32 ml of unknown matches with 24 mcg of standard

0.32ml U = 24mcg

1ml of U = 24/0.32 = 75mcg/ml


75 mcg/ml
The concentration of histamine present in the unknown solution = _____ mg/ml

G. Conclusion:

By matching assay, using guinea pig ileum, the concentration of histamine present
in the unknown solution is 75 mcg/ml

146
NMC- Competency No: PH 4.2e
Ex. No: 32 CAL – Bioassay of histamine on guinea pig ileum.

H. Discussion:
1. Carefully observe the effect of drugs on frog’s esophagus in the given chart and answer
the following questions.

a) Define bioassay. What are the types of bioassay?

Bioassay is defined as detecting and measuring the concentration of the substance in preparation or
estimating the active principle’s potency in a unit quantity of preparation using biological methods
by observing specific pharmacological effects on living cells, micro-organisms, or animals.

Types:

1. Matching assay
2. Bracketing assay
3. Three point assay
4. Four point assay

147
NMC- Competency No: PH 4.2e
Ex. No: 32 CAL – Bioassay of histamine on guinea pig ileum.

b) Why guinea pig ileum is selected for assaying histamine? What are the other tissues
that are used to assay histamine?
The terminal ileum of guinea pig is rich in histamine receptors and hence it can used for
bioassaying the histamine.

Other tissues used to assay histamine are uterus of guinea pig and guinea pig trachea.

c) What is the reason for selecting the standard between the 30-70% of DRC?

When the response obtained is plotted against dose, a hyperbolic curve will be obtained which
is non-linear. Thus dose are converted to log-dose and plotted; which gives 'S' shaped Dose
response curve. This DRC is linear between 30 - 70% and hence standard is selected in this range

d) What is meant by supramaximal response? Mention the importance of supramaximal


response in this experiment.

The response that is obtained after the maximal response is called supra-maximal response.
It will be usually lesser than the maximal response due to tissue fatigue.

Importance: Maximal response can be identified using supramaximal response.

e) What are the advantages and disadvantages of matching assay?

Advantages:
Easy and relatively simple bioassay method
Used when unknown sample is of low volume

Disadvantages:
Often kymogram will run out of space for recording in a matching assay.
High rate of subjective error.
Exact matching cannot occur occasionally.

148
COMMUNICATION SKILL

OBJECTIVES
At the end of communication skill exercises, the
learner shall be able to:

1. Realize the importance of effective


communication in achieving optimal drug
use.
2. Explain to the patient the relationship
between cost of the treatment and patient
compliance with empathy and ethics
3. Motivate the patient with chronic disease to
adhere to the prescribed management.
4. Demonstrate the ability to educate the public
and patients about drug dependency and OTC
drugs

149
NMC- Competency No: PH 5.1, 5.2 & 5.3
Ex. No: 33 Effective Doctor Patient Communication.

Session plan:

 A short introduction on communication and its importance is given for 20 minutes. The
class will be divided into groups of 4-5 students.
 Prepare role-plays to highlight the salient features of possible adverse drug reactions,
proper usage (when to start/stop), precautions, contraindications, and return for
refill/assessment of the given drug. Focus on methods of establishing rapport, active
listening, usage of lay-terms in explaining, body language etc., when one group has
finished its role-play the other groups have to comment on the play.
 All 4 or 5 groups will perform their respective role-plays. The teacher will comment only
after all the role-plays have finished. If time permits one of the groups will be asked to
re-enact the play with suggested modifications.

Group tasks:

The students should write the following in simple English without using medical terms for the
ease of understanding by the patient.

Instruction/advice regarding the drug use should contain the following


1. Explanation regarding the disease pathology/problem
2. Explanation for investigation to be done (if any)
3. Explanation for the drugs selected to cure that disease
4. Advising regarding any life style and diet modification.
5. Explanation for use of that particular formulation (if any).
6. Advising regarding early identification of possible ADRs due to that drug.
7. Advice regarding prevention of the problem in future (if any)

150
NMC- Competency No: PH 5.1, 5.2 & 5.3
Ex. No: 33 Effective Doctor Patient Communication.
Group Task:

1. A 10 year old boy was brought to the chest medicine OPD with complaints of episodic
breathlessness and wheezing. He also complained of cough during the episode. He had
3-4 episodes per week. Physical examination revealed prolonged expiration and diffuse
wheeze. He was diagnosed to have bronchial asthma. You are prescribing inhaled
budesonide by a metered dose inhaler (MDI). How will you advise the patient regarding
the proper use and care of MDI?

A. Greeting the patient and care taker.

B. Explanation about the disease to the care taker of the child:


You child is having a disease called asthma. It is a long term lung problem that causes
airways (the tubes that carry air into and out of the lungs) to become inflamed, which means that
they swell and produce lots of thick mucus. The muscles surrounding the airways also tend to
tighten, which makes the already clogged airways even narrower. This produces the
whistling sound in your child when he breathes with difficulty. This is called asthma attack.

C. Explanation about the medication:


Thus to prevent the attacks I am prescribing a mild steroid drug called budesonide that will
suppress the swelling and mucus production inside the airways. This drug will prevent the
frequent asthma attacks like this.

D. Use of formulation
This drug is available is metered dose inhaler. You have to use IT daily once. Before using
gently shake the container. Remove the mouth piece cover and Hold the inhaler upright with the
thumb on the base and one or two fingers on the canister. Now breathe out and Keep the mouth
piece in the mouth, close the lips and Press the canister to release one dose. Now take breath
steadily and deeply. Remove the inhaler and hold breath for 10 to 20 seconds and Breathe out
slowly. Finally, replace the mouth piece cover and gargle the mouth after inhalation

E. Care of formulation
Always keep this inhaler loaded. Check the canister for drug. If you feel it is about to run out
buy another one and keep it ready. Do not throw the cannister into the fire as it may explode.
Dispose the cannister properly as whole.

F. Explanation about ADRs.


Sometimes you may develop white patch on your throat. In that case, report to me immediately.

151
NMC- Competency No: PH 5.1, 5.2 & 5.3
Ex. No: 33 Effective Doctor Patient Communication.
Group Task:

2. A 27 year old female who has one child comes to the OPD to consult you for need of
contraception. You wish to start her on combined oral contraceptive pill (OCP). What
instructions will you provide when starting her on OCP?

A. Greeting the patient

B. Explanation about the issue:


The ideal method of contraception for you will be oral contraceptive pills because they have
better efficacy than barrier methods like condom.

C. Explanation about the medication:


These pills contains hormones similar to your body hormones and it will prevent the release of egg
from ovum in your body. Thus you will not get pregnant as long as you take this pills
because there is no ovum to get fertilized.

D. Use of formulation
The Oral contraceptive pill that i prescribe has a total of 28 tablets. Behind the strip, the
pills are numbered which represent the day you have to take. The first day is the first day of
your menstruation. You have to take till you reach 28. The pills should be taken at same time
of every day for better results. The first 21 tablets are hormonal pills and last 7 pills are iron
tablets. You will have menstruation while taking the iron pills. Once the pack is over, you have
to start next pack next day. If you forgot to take the pill on time, take the pill as soon as you
remember and take next day pill as per old time. If you forgot to take pills for three or four
days consequetively, then OCP may not work properly. You have to consult me in such cases

E. Care of formulation
Always keep this strip in cool place and avoid direct sunlight. Do not freeze the tablets.

F. Explanation about ADRs.


Sometime you may develop headache, bloating sensation, break-through bleedings, and
breast tenderness. In such cases, report to me immediately.

152
NMC- Competency No: PH 5.1, 5.2 & 5.3
Ex. No: 33 Effective Doctor Patient Communication.
Group Task:

3. A 20 year old lady was diagnosed to have generalized tonic-clonic seizures and you have
decided to start her on phenytoin sodium. What instructions will you provide to her as
well as her family members?

A. Greeting the patient

B. Explanation about the issue:


You have a disease called Generalized tonic clonic seizure which is seizure involving whole
body. This occurs due to abnormal current generation in the brain. Thus medications are
required to reduce those abnormal current generations in the brain.

C. Explanation about the medication:


I am prescribing a drug called phenytoin which will reduce the abnormal current generation
in brain and thereby controls the frequency of seizures. You have to take this drug regularly
until you have seizure free duration of at least six months.

D. Use of formulation
I am prescribing a standard adult dose of phenytoin. You have to take two tablets of 100mg
strength at night time after food daily.

E. Care of formulation
Always keep this strip in cool place and avoid direct sunlight. Do not freeze the tablets. If you
forgot to take tablet for one day, do not take two tablets on next day as it may cause unwanted
side effects. Do not adjust the dose by yourselves. Also do not change the brand as different
brands of phenytoin will have different efficacy.

F. Explanation about ADRs.


Sometime you may abnormal growth of your gums in mouth, dizziness, excessive sleepiness,
abnormal head spinning sensation, difficult to focus on object, and frequent headaches. In such
cases report to me immediately.

153
NMC- Competency No: PH 5.1, 5.2 & 5.3
Ex. No: 33 Effective Doctor Patient Communication.
Group Task:

4. A 46 year diabetic patient, not adequately controlled with oral anti-diabetic agents,
comes to your clinic. You decide to start him on insulin. How will you educate the
patient on the proper use of insulin?

A. Greeting the patient

B. Explanation about the issue:


Since your diabetes is not controlled with standard oral medications, I am now advising you
to take insulin in injectable form. Diabetes occurs because of low or absence of insulin in body
The oral drugs for diabetes will increase the level of insulin in your body and controls blood
sugar levels. However after some times, the drugs will stop working and in such cases we
have to inject insulin from outside.

C. Explanation about the medication:


I am prescribing a two types of insulin (regular and NPH The insulin which is water like is
regular insulin and the other one which is milk like is NPH insulin. These two insulin should be
mixed and injected beneath the skin by insulin syringe.

D. Use of formulation
First prepare the food and keep it ready. Now take the regular insulin vial and load the
prescribed units in the insulin syringe. Now take the NPH vial and load the prescribed units.
Now wipe the site of insulin administration with spirit. You can use your skin over arm, abdomen
and thighs. You should rotate the sites regularly. After 30 seconds, make a skin fold and inject
the insulin beneath the skin. Use only insulin syringe and half-inch 28G needle. Never rub the
site of administration. Now take the food within 15 minutes of insulin administration.

E. Care of formulation
Always keep the vial in refridgerator. Never keep the insulin vials in direct sunlight. Do not freeze
the vials. Do not adjust the dose of insulin by yourselves. Because too much of insulin will cause
dangerous low-sugar condition.

F. Explanation about ADRs.


Sometime you may develop palpitation, sweating, dizziness after insulin administration. These
are the symptoms of low-sugar. In such cases take a candy or sugar-salt water immediately.
Report to me afterwards to adjust the dose of insulin.

154
NMC- Competency No: PH 5.1, 5.2 & 5.3
Ex. No: 33 Effective Doctor Patient Communication.
Group Task:

5. A 45 year old patient complaint of frequent attacks of retrosternal chest pain


particularly after severe exertion. His ECG reveals ST depression and he is diagnosed to
have exertional angina pain. You plan to prescribe transdermal nitroglycerin patch along
with the other drugs. How will you advise the patient about the proper use and disposal
of the patch?

A. Greeting the patient

B. Explanation about the issue:


Since you chest pain that occurs due to insufficient blood flow to your heart, medications
are required to provide adequate blood supply to heart. This will reduce your chest pain
while exertion.

C. Explanation about the medication:


I am prescribing drug called Nitroglycerine as patch. This drug will widen your blood vessels in and
around the heart so that adequate blood flow to heart is established. It also reduces the work load in
your heart. Hence besides other standard drugs, you have to use nitroglycerine.

D. Use of formulation
You can apply the patch over the upper body. The area should be relatively hair free. Do not shave the
hair. You can trim the hair and apply the patch over it.
Peel off one side of the plastic backing. Use the other side of the patch as a handle, and apply the stick
half to your skin in the spot you have chosen. Press the sticky side of the patch against the skin and
smooth it down. Fold back the other side of the patch. Hold onto the remaining piece of plastic backing
and use it to pull the patch across the skin. Now press down on its center to lift the edges away from the
skin. Hold the edge gently and slowly peel the patch away from the skin.

E. Care of formulation
Never freeze the patch. After removal, fold the patch in half with the sticky side pressed together and
dispose of it safely, out of the reach of children and pets. The used patch may still contain active
medication that can harm others.

F. Explanation about ADRs.


The skin may be red and may feel warm for a short time. You may apply lotion if the skin is dry, and you
should call me if the redness does not go away after a short time. Nitroglycerin patches may no longer
work as well after you have used them for some time. To prevent this, you have to wear each patch for
only 12 to 14 hours each day so that there is a period of time when you are not exposed to nitroglycerin
every day. If your angina attacks happen more often, last longer, or become more severe at any time
during your treatment, report to me immediately.

155
NMC- Competency No: PH 5.1, 5.2 & 5.3
Ex. No: 33 Effective Doctor Patient Communication.
Group Task:

6. A 10 year old boy comes to the pediatrics OPD with complaints of generalized itching.
Physical examination reveals pruritic vesicles on finger webs. You decide to start him on
permethrin. Instruct the mother regarding the proper way of application and the
hygiene measures that has to be taken.

A. Greeting the patient and care taker.

B. Explanation about the issue:


Your child has a type of parasite infestation in his skin called scabies. To get rid of this parasite,
certain medications should be applied over the skin.

C. Explanation about the medication:


I am prescribing drug called permethrin in lotion form. This is paralyse the parasite and make
it fall from the body and thereby curing the disease. This medication will kill parasite and the itching
associated with it will take 6 to 7 days to settle.

D. Use of formulation
Gently shake the container before opening. Thoroughly wash and dry the body. Massage the lotion
into the skin from the head to the soles of the feet, paying special attention to creases in the skin,
hands, feet, between fingers and toes, underarms, and groin. Leave the permethrin lotion on the
skin for 8 to 14 hours. Wash off by taking a shower or bath and Change into clean clothes.
Wash the dirty clothes with hot water and dry it in direct sunlight.

E. Care of formulation
This lotion is meant for application on the skin and not to drink. Store the medicine in a closed
container at room temperature, away from heat, moisture, and direct light. Do not freeze the lotion

F. Explanation about ADRs.


You child may rarely burning, rash, redness, tingling of the skin or scalp.
If it is severe, then report to me immediately.

156
NMC- Competency No: PH 5.4
Ex. No: 34 Pharmacoeconomics.

1. A 40 year old male allergic to sulpha drugs presents at Medical OPD with history of fever,
cough with expectoration for 3 days. He is diagnosed to have upper respiratory infection.
He is working in a factory and gets such attacks frequently. Sputum culture shows
Streptococci and H. influenza sensitive to antibiotics Cephalexin, Azithromycin and
Cotrimoxazole.
Regimen A: C. Cephalexin 500 mg tds - 7days
Regimen B: T. Azithromycin 500 mg od -5days
Regimen C:T.Cotrimoxazole (DS) 1bd -7days
Price:
C. Cephalexin 500 mg - Rs: 11.00/cap
T. Azithromycin 500mg - Rs: 17.00 /tab
T. Cotrimoxazole (SS) 800+160mg - Rs: 1.70/tab
Note: DS = Double strength and SS = single strength
Work out the cost benefit.

Regimen ‘B’ is preferred.


Though Regimen ‘C’ is cheaper than ‘B’ since patient is allergic to sulfonamides it is not
advised.

157
NMC- Competency No: PH 5.4
Ex. No: 34 Pharmacoeconomics.
2. A 7 years old boy is a known case of seizure disorder. He has to be treated with one of the
following schedule. Work out the cost benefit for one month and justify your option.
Regimen A: T. Carbamazepine 200 mg 1bd
Regimen B: T. Phenytoin 100 mg 1bd
Regimen C: T. Phenobarbitone 60 mg HS
Price:
T. Carbamazepine 200 mg -- Rs 1.70/tab
T. Phenytoin 100mg -- Rs 1.00/tab
T. Phenobarbitone 60 mg -- Rs1.20 /tab

Schedule ‘B’ is preferred.


Even though schedule ‘C’ is cheaper since Phenobarbitone has defective memory power &
lower the learning capacity.

158
NMC- Competency No: PH 5.4
Ex. No: 34 Pharmacoeconomics.
3. A 50 year old female with osteoporosis has to be treated with one of the following
schedule. Work out the cost benefit for one week
Schedule A
T. Calcium carbonate 500 mg 1bd
T. Alendronate 70 mg once weekly
Schedule B
T. Calcium citrate 1 g 1bd
T. Risedronate 35 mg once weekly
Price:
T. Calcium carbonate 500mg Rs.1.92/tab
T. Alendronate 70mg Rs. 32/tab
T. Calcium citrate 1g Rs. 4.60/tab
T. Risedronate 35mg Rs. 30/tab

Schedule A is preferred as it is cheaper and equally efficacious

159
NMC- Competency No: PH 5.4
Ex. No: 34 Pharmacoeconomics.
4. A 55years old female patient is diagnosed to have chloroquine resistant malaria fever and
to be treated with the following schedule. Work out the cost benefit ratio.
Schedule A: T. Artemether 80 mg & T. Lumifantrine 480 mg b.d x 3 days
Schedule B: T. Artesunate 100 mg bd x3 days
T. Sulfadoxine 150 mg & T. Pyrimethamine 75 mg --single dose
Price List :
FDC: (T. artemether 20 mg & T. Lumifantrine 120 mg)= Rs. 13.00/tab
T. Artesunate 100 mg = Rs. 20.00/tab
FDC: (T. Sulfadoxine 150 mg & T. Pyrimethamine 75 mg) = Rs. 2.10/tab
Note: FDC = Fixed drug dose combination

Schedule B is preferred as it is cheaper

160
NMC- Competency No: PH 5.4
Ex. No: 34 Pharmacoeconomics.
5. A 25 year old female with primigravida diagnosed to have pregnancy induced hypertension
has to be treated with one of the following schedule. Work out the cost benefit ratio for
one month.
Schedule A: T. Hydrochlorothiazide 12.5 mg od
Schedule B: T. Enalapril 5 mg od
T. Hydrochlorothiazide 12.5 mg od
Schedule C: T.Amlodipine 5mg od
Price:
T. Hydrochlorothiazide 12.5 mg Rs. 1.00/tab
T. Enalapril 5mg Rs.1.50/tab
T. Amlodipine 5mg Rs. 3.00/tab

Though schedule A & B are cheaper, Diuretics are contraindicated in pregnancy induced
hypertension and ACE inhibitors are teratogenic
Hence Schedule C is preferred .

161
NMC- Competency No: PH 5.4
Ex. No: 34 Pharmacoeconomics.
6. A 27 year old female with primigravida diagnosed to have gestational diabetes is to be
treated with the following regimens. Work out the cost benefit ratio for one month.
Schedule A: Inj. Human Insulin (R) 10 Units s.c tds
Schedule B: Inj. Lente Insulin 20 Units s.c. bd
Schedule C: Inj. Human Mixtard-50 15 Units s.c bd
Price:
Inj Human Insulin ® 1 vial 10 ml = Rs.140
Inj. Lente Insulin 1 vial 10 ml = Rs.100
Inj. Human Mixtard-50 1 vial 10 ml = Rs.160
Note: 40 units of insulin = 1 ml

Schedule B is preferred as it is cheaper.

162
NMC- Competency No: PH 5.5 &5.6
Ex. No: 35 Prescribing drugs with dependence liability, OTC drug
abuse and its management.
1. A 55 year old female Mrs. Kumari weighing 60Kg presented to your clinic with bilateral pedal
edema and oliguria. Upon further investigation, her renal function tests were elevated. Her
blood pressure is 160/102 mm of Hg. A diagnosis of acute renal failure was arrived. History of
drug use revealed that she is taking T. Diclofenac and T. Nimesulide without prescription for the
past 6 months for her bodyache.
a) What is meant by OTC drugs? Mention few examples
b) What are the advantages and disadvantages of self medication?
c) Define drug misuse. How does it differ from drug abuse?
d) Mention the ideal management for this patient.
e) What are the advices you will give to the patient and her family regarding the
situation?

a. Drugs available without prescription are called as Over the Counter (OTC) drugs.
They are considered safe and thus can be used without prescription.
Ex: Antacids, analgesics like aspirin, Paracetamol, antacids.

b. Advantages of self medication:


Satisfaction to consumer, reduced burden to management of hospital, reduced
workload on clinicians and profit to pharmaceuticals

Disadvantages of self medication:


Toxicity due to overdose
Delay in appropriate care due to symptomatic relief by OTCs
Unwanted drug interactions

c. Drug misuse is defined as any drug that is used or prescribed without scientific
justification or prolonged use of drug that is not actually required. Ex: Use of antibiotics
in viral fever.
Drug abuse is persistent excessive use of drug inconsistent with or unrelated to
medical or social practice is labelled as drug abuse. Ex: LSD, Cocaine. Drug abuse
is more serious than drug misuse.

d. The patient is suffering from NSAID abuse induced nephropathy. Stopping of all the
strong NSAID should be done. T. Paracetamol 650 mg can be prescribed for the
bodyache and appropriate renal care should be provided.

e. NSAID induced nephropathy is reversible in most of the situations. Hence the patient
should be educated about the NSAID misuse. The family members should be warned
about the future consequences (CKD, Need of dialysis etc) if patient continues to
abuse NSAID drugs.

163
164
NMC- Competency No: PH 5.5 &5.6
Ex. No: 35 Prescribing drugs with dependence liability, OTC drug
abuse and its management.
2. A 55 year old male Mr. Karthick weighing 60Kg presented to your clinic with chest pain and
dyspnea. He is a known alcoholic under de-addiction treatment taking T. DISULFIRAM 500MG
BD daily. He has consumed alcohol under peer compulsion 6 hours back.
a) What is Drug dependence? Mention few examples
b) What is aversion therapy? Mention the differences between psychological and physical
dependence.
c) What is the reason behind the development of his symptoms? Explain the mechanism
d) Discuss the management of alcohol withdrawal syndrome.
e) What are the advices you will give to patient and his relatives regarding the situation?

a. Drug dependence is a psychic and/or physical state resulting from interactions between
a living organism and a drug, characterized by behavioral and other effects that always
include a compulsion to take the drug on a continuous or periodic basis in order to
explore the psychic effect. Example: Amphetamine causing psychological dependence,
Morphine, alcohol and barbiturate causing physical dependence.

b. Aversion therapy is a type of behaviour therapy designed to make patients give up an


undesirable habit by causing them to associate it with an unpleasant effect.

Psychological dependence are less serious and minimal withdrawal symptoms are seen
upon stopping the drug. In physical dependence, abuse is serious and withdrawal
symptoms are severe. Drug abuse is for positive reinforcement in psychological dependence
while drug abuse is to avoid development of withdrawal symptoms in physical dependence.

c. Disulfiram is an aldehyde dehydrogenase inhibitor. It causes accumulation of acetaldehyde


after consumption of ethanol inside the body. Accumulated acetaldehyde produces chest
pain, dyspnoea and other unwanted effects. (Aversion therapy)

d. Injection Lorazepam 0.08mg/Kg I.V stat followed by T. Chlordizepoxide 50 - 100mg OD daily


for 10 to 14 days. Once the withdrawal symptoms are settled, the patients should be put
under counselling for alcohol deaddiction again.

e. Since the patient is already under the aversion therapy, the patient should not consume
alcohol. If consumed it will produce disulfiram reactions. Symptoms of this reaction include
copious nausea and vomiting, severe headaches, flushing, dizziness, sweating, and chest pain.
In some cases, trouble breathing, blurry vision, fainting, confusion, and irregular heart rhythms
can cocur. Occasionally it may be fatal. Thus patient relatives should always monitor the patients
who are on aversion therapy.

165
166
NMC- Competency No: PH 5.5 &5.6
Ex. No: 35 Prescribing drugs with dependence liability, OTC drug
abuse and its management.

3. A 28 year old male Mr. vignesh weighing 45 kg brought to your hospital with loss of
consciousness, pinpoint pupil and tachycardia. He is a known drug abuser of heroin and
morphine. Multiple injection marks were present on his both arms.
a) Define drug addiction. Mention few drugs causing addiction
b) What is the difference between drug addiction and drug dependence?
c) What is the diagnosis? Discuss the ideal management for this patient.
d) What is the role of stomach wash in this patient?
e) What are the advices you will give to patient and his relatives regarding the situation?

a. Drug addiction is a compulsive drug usage. There is a compulsion to obtain drug at any cost.
Addiction is marked by a change in behavior caused by the biochemical changes in the brain after
continued substance abuse. Example: Cocaine, LSD, Alcohol, etc.,

b. When a person abuses a drug despite knowing the consequences and unable to stop using it
and behaves in irrational way to acquire it, then it is called drug addiction. On the other hand,
in drug dependence, there is a compulsion to take the drug on a continuous or periodic basis in
order to explore the psychic effect but there will not be an irrational behavior or self harm to acquire
the drug. Drug dependence is mild form of substance use disorder while addiction is moderate
to severe form of substance use disorder. Now a days, addiction and dependence is replaced with
the term - Substance use disorder.

c. The patient is suffering from acute opioid poisoning.


General measures - Patent airway, 100% oxygen, artificial respiration,
Gastric lavage done with potassium permanganate.
Specific antidote: Naloxone 0.4-0.8 mg i.v repeated every 2-3 minutes till respiration improves.

d. Morphine can secreted into stomach and reabsorbed in case of poisoning by intravenous
administration. Thus, stomach wash is mandatory in case of morphine poisoning.

e. Drug addiction is a serious problem and should be attended immediately. De-addiction should be
done with proper inpatient admission and patient relative should closely monitor the patient
to prevent relapse.

167
168
NMC- Competency No: PH 5.5 &5.6
Ex. No: 35 Prescribing drugs with dependence liability, OTC drug
abuse and its management.

4. A 19 year old male Mr. Vimal weighing 60 kg brought to your hospital with drowsiness, and
disoriented speech. His mother revealed that though he did not have any symptoms of
respiratory illness, he bought the cough syrup as OTC and consumed it excessively. She also
informed you that this incidence is fifth in the present year.
a) Define drug habituation. Mention few examples.
b) Mention the difference between drug habituation and drug addiction.
c) Which components in the cough syrup can cause drug habituation? Explain the
mechanism
d) Mention the ideal management for this patient.
e) What are the advices you will provide to the patient and his mother regarding this
situation?

a. Drug habituation is a pattern of repeated drug usage though the actual physical need is
minimal. Eg: Nicotine and Caffeine.

b. In drug habituation, the actual physical need is minimal and withdrawal symptoms are usually
absent. In case of drug addiction, the need of drug is very high with presence of withdrawal
symptoms.

c. Codeine containing cough syrups can cause drug habituation. If used repeatedly, codeine
being an opioid can evolve to addiction from habituation.

d. The patient is suffering from acute opioid (codiene) poisoning.


General measures - Patent airway, 100% oxygen, artificial respiration,
Gastric lavage done with potassium permanganate.
Specific antidote: Naloxone 0.4-0.8 mg i.v repeated every 2-3 minutes till respiration improves.

e. Drug addiction is a serious problem and should be attended immediately. De-addiction should be
done with proper inpatient admission and patient relative should closely monitor the patient
to prevent relapse.

169
170
NMC- Competency No: PH 5.5 &5.6
Ex. No: 35 Prescribing drugs with dependence liability, OTC drug
abuse and its management.

5. A 48 year old female Mrs. Kamalam weighing 60 kg is in post operative ward followed by
repair of her umbilical hernia. Inj. Pentazocine was prescribed for her post-operative pain. On
her seventh day, it was replaced with T. Ibuprofen 400mg BD orally. Her pain score was only 2
out of 10 in pain scale. After discharge, she went to local pharmacy to get Inj. Pentazocine
without prescription. She was refused by multiple pharmacist for the same.
a) Explain why she was refused by the pharmacist to provide Inj. Pentazocine?
b) What are schedule X drugs? Mention few examples
c) What are schedule H drugs? Mention few examples
d) What are rules and regulations stipulated by the regulatory authority for sale of
schedule X drugs?
e) What are the advices you will provide to the patient regarding this situation?

a. Pentazocine being an opioid drug should not be given without valid prescription.

b. Schedule X drugs includes drugs that have addiction or dependence liability.


Example: Barbiturates, opioids

c. Schedule H drugs includes drugs that are sold only when a registered medical
practitioner prescribes. Example: Digoxin, Etoricoxib etc.,

d. Schedule X drugs must be kept under lock and key always


The details of the person responsible for the access should be entered in the register
Prescription for Schedule X drugs should be in duplicate. One copy should be retained
and preserved by the retailer for 2 years in the records.
A special register should be maintained by the practitioner and retailer for these drugs

e. Misuse of opioid drugs could be dangerous as it may cause addiction over the time.

171
172
NMC- Competency No: PH 5.5 &5.6
Ex. No: 35 Prescribing drugs with dependence liability, OTC drug
abuse and its management.

6. A 3 year female child was brought to your hospital with complaints of excessive diarrhoea
and drowsiness. On examination, the child was severely dehydrated. Her mother revealed that
she was administering syrup. Azithromycin and syrup. Amoxiclave for the past 10 days for
cough and cold. She also informed that previous doctor has prescribed the above syrups for
cough and cold three months before. She bought these syrup from the local pharmacy using
the same prescription and administered for ten days.
a) What is the diagnosis? What is the ideal management for this child?
b) What are the advantages and disadvantages of OTC?
c) Mention two catastrophic hypothetical scenarios where OTCs are dangerous.
d) Provide two remedial measures where you can avoid this situation in future in public.
e) What are the advices you will give to the mother regarding this situation?

a. Antibiotic associated diarrhea due to super infection. The usual cause of antibiotic
associated diarrhoea is C.difficle. Precipitating antibiotics should be stopped. Oral or
intravenous metronidazole or vancomycin for 10 - 14 days should be given.

b. Advantages of OTC:
Satisfaction to consumer, reduced burden to management of hospital, reduced
workload on clinicians and profit to pharmaceuticals

Disadvantages of OTC:
Toxicity due to overdose
Delay in appropriate care due to symptomatic relief by OTCs
Unwanted drug interactions

c. Taking antacid for gastritis or epigastric discomfort in middle aged persons with
comorbidities may mask the underlying evolving Myocardial infarction.

Taking strong NSAIDs for mild pain repeatedly by OTC may evolve into NSAID
induced nephropathy.

d. Public should be made aware about the implications of OTCs.


Recurrent social awareness programs should be conducted about the misuse of OTCs

e. Never administer antibiotic to the child by self. Always consult a healthcare provider
for administration of antibiotics. Never provide antibiotic than the recommended duration.

173
174
Annexure 1 Terms related to Pharmacology
Prescription:
Prescription is an order written by a physician, dentist or any registered medical
practitioner to a pharmacist to compound and dispense a specific medication for the patient.
Parts of a prescription: A prescription should have the following parts:
Superscription:
Date
Doctor’s Name with degree, Registration number, address with phone number
Patient’s name, age and Sex
Inscription:
It is the body of the prescription. It contains the following
Diagnosis
Rx symbol – Considered as prayer to Jupiter/right eye of the Horus God
Drug formulation, Name of the drug, dose, frequency and duration of the
therapy
Subscription:
Contains the instruction to the attending nurse or advice to the patient
Signature of the doctor
Number of refills
Agonist:
A ligand that binds to a receptor and alters the receptor state resulting in a biological
response.

Antagonism:
The joint effect of two or more drugs such that the combined effect is less than the sum
of the effects produced by each agent separately.

Bioassay or Biological Assay:


The determination of the potency of a physical, chemical or biological agent, by means
of a biological indicator. The biological indicators in bioassay are the reactions of living
organisms or tissues.

Bioavailability:
The percent of dose entering the systemic circulation after administration of a given
dosage form. More explicitly, the ratio of the amount of drug “absorbed” from a test
formulation to the amount “absorbed” after administration of a standard formulation.
Frequently, the “standard formulation” used in assessing bioavailability is the aqueous
solution of the drug, given intravenously.

Clearance:
The clearance of a chemical is the volume of body fluid from which the chemical is, apparently,
completely removed by biotransformation and/or excretion, per unit time.

Dependence:
A somatic state which develops after chronic administration of certain drugs; this state
is characterized by the necessity to continue administration of the drug in order to
avoid the appearance of uncomfortable or dangerous (withdrawal) symptoms.

175
Generic Drugs:
Drug formulations of identical composition with respect to the active ingredient, i.e.,
drugs that meet current official standards of identity, purity, and quality of active
ingredient.

Half-Life:
The period of time required for the concentration or amount of drug in the body to be
reduced to exactly one-half of a given concentration or amount.

National Formulary (N.F.):


A reference volume published formerly by the American Pharmaceutical Association
containing standards of purity and methods of assay for some drugs, and formulae and
methods of manufacture for a variety of pharmaceutical preparations. Drugs were
included on the basis of demand as well as therapeutic value.

Pharmacogenetics:
The science and study of the inheritance of characteristic patterns of interaction
between chemicals (drugs) and organisms. Pharmacogenetics involves identification
and description of such patterns, discriminating them from non-heritable patterns, and
elucidation of the mechanism of inheritance.

Pharmacology/Pharmacy/Pharmacognosy:
(Gr. Pharmakon – drug, and Logos – word) is the study of drugs and its interaction with
biological matter in all their aspects.
Pharmacy is an independent discipline concerned with the art and science of the
preparation, compounding, and dispensing of drugs.
Pharmacognosy is a branch of pharmacy that deals with the identification and analysis
of the plant and animal tissues from which drugs may be extracted.

Side Effects:
Drug effects which are not desirable or are not part of a therapeutic effect; effects
other than those intended. Pharmacological side effects are true drug effects. With
increasing doses of a drug, the intensity of pharmacological side effects in individuals,
and/or the frequency with which a pharmacological side effect is observed in a
population is increased.

Tolerance:
A condition characterized by a reduced effect of a drug upon repeated administration.
In some cases, it may be necessary to increase the dose of the drug to attain the same
effect, or the original level of effect may be unattainable. Tolerance typically develops
over days to weeks, and is distinguished from tachyphylaxis, a more rapid decline in the
effect of a drug.

Tachyphylaxis:
A decline in the response to repeated applications of agonist, typically occurring over a
relatively short time scale.

176
Annexure 2 Latin terms commonly used in prescription

S.No Abbreviation Latin Meaning


1 a.c Ante cibum Before meal
2 Ad lib Ad libitum Use as much as one desires
3 a.m Ante meridiem Before noon
4 b.i.d Bis in die Use twice a day
5 c cum With
6 Dieb alt Diebus alternis Every other day
7 e.m.p Ex modo prescripto As directed
8 gtt gutta Drop
9 h.s Hora somni At bedtime
10 lin linimentum Liniment
11 Nebul nebula A spray
12 Non rep Non repetatur No repeats
13 n.p.o Nil per os Nothing by mouth
14 o.d Omni die Every day
15 o.h Omni hora Every hour
16 o.m Omni mane Every morning
17 o.n Omne nocte Every night
Part affecte
18 p.a.a To be applied to the affected part
applicandus
19 p.c Post cibum After meals
20 p.o Per os By mouth
21 p.r.n Pro re nata Use as needed
22 Pulv Pulvis Powder
23 q quaque Every
24 q.a.d Quaque alternis die Every other day
25 q.d Quaque in die Use every day
26 q.h Quaque hora Use every hour
27 q.i.d Quarter in die Four times a day
28 S.o.s Si opus sit Whenever needed
29 Stat Statum Immediately
30 t.i.d Ter in die Thrice a day

177
Annexure 3 Weights and measures in metric system

178
Annexure 4 Sterilization methods
Sterilization:
Sterilization refers to any process that eliminates (removes) or kills (deactivates) all
forms of life and other biological agents (as well as viruses which some do not consider
to be alive but are biological pathogens nonetheless), including transmissible agents
(such as fungi, bacteria, viruses, spore forms, unicellular eukaryotic organisms such as
Plasmodium, etc.) present in a specified region, such as a surface, a volume of fluid,
medication, or in a compound such as biological culture media.

Methods of Sterilization:
Physical Method:
Dry Heat
Moist Heat
Irradiation
Filtration
Chemical Method:
By Gas
By heating with bactericidal agents

Sterilization by Dry heat:


Dry heat was the first method of sterilization, and is a longer process than moist heat
sterilization. The destruction of microorganisms through the use of dry heat is a gradual
phenomenon. With longer exposure to lethal temperatures, the number of killed
microorganisms increases. Forced ventilation of hot air can be used to increase the rate
at which heat is transferred to an organism and reduce the temperature and amount of
time needed to achieve sterility. At higher temperatures, shorter exposure times are
required to kill organisms. This can reduce heat-induced damage to food products. The
standard setting for a hot air oven is at least two hours at 160 °C. A rapid method heats
air to 190 °C for 6 minutes for unwrapped objects and 12 minutes for wrapped objects.
Dry heat has the advantage that it can be used on powders and other heat-stable items
that are adversely affected by steam (e.g. it does not cause rusting of steel objects).

Sterilization by Moist heat:


A widely used method for heat sterilization is the autoclave, sometimes called a
converter or steam sterilizer. Autoclaves use steam heated to 121-134 °C under
pressure. To achieve sterility, the article is heated in a chamber by injected steam until
the article reaches a time and temperature set point. Meantime almost all the air is
removed from the chamber, because air is undesired in the moist heat sterilization
process. The article is then held at that set point for a period of time which varies
depending on the bio burden present on the article being sterilized and its resistance
(D-value) to steam sterilization. A general cycle would be anywhere between 3 and 15
minutes, (depending on the generated heat) at 121 °C at 100 kPa, which is sufficient to
provide a sterility assurance level of 10−4 for a product with a bio burden of 106 and a
D-value of 2.0 minutes. Following sterilization, liquids in a pressurized autoclave must
be cooled slowly to avoid boiling over when the pressure is released. This may be
achieved by gradually depressurizing the sterilization chamber and allowing liquids to
evaporate under a negative pressure, while cooling the contents.

179
Sterilization by irradiation:
Sterilization can be achieved using electromagnetic radiation such as electron beams, X-
rays, gamma rays, or irradiation by subatomic particles. Electromagnetic or particulate
radiation can be energetic enough to ionize atoms or molecules (ionizing radiation), or
less energetic (non-ionizing radiation).

Sterilization by filtration:
Fluids that would be damaged by heat, irradiation or chemical sterilization, such as drug
products, can be sterilized by microfiltration using membrane filters. This method is
commonly used for heat labile pharmaceuticals and protein solutions in medicinal drug
processing. A micro filter with pore size 0.2 µm will usually effectively remove
microorganisms. In the processing of biologics, viruses must be removed or inactivated,
requiring the use of nanofilters with a smaller pore size (20 -50 nm) are used. Smaller
pore sizes lower the flow rate, so in order to achieve higher total throughput or to avoid
premature blockage, pre-filters might be used to protect small pore membrane filters.

Membrane filters used in production processes are commonly made from materials
such as mixed cellulose ester or polyethersulfone (PES). The filtration equipment and
the filters themselves may be purchased as pre-sterilized disposable units in sealed
packaging, or must be sterilized by the user, generally by autoclaving at a temperature
that does not damage the fragile filter membranes. To ensure proper functioning of the
filter, the membrane filters are integrity tested post-use and sometimes before use. The
non-destructive integrity test assures the filter is undamaged, and is a regulatory
requirement. Typically, terminal pharmaceutical sterile filtration is performed inside of
a clean room to prevent contamination.

Sterilization by Chemical methods:


Chemicals are also used for sterilization. Heating provides a reliable way to rid objects
of all transmissible agents, but it is not always appropriate if it will damage heat-
sensitive materials such as biological materials, fibre optics, electronics, and many
plastics.
In these situations chemicals, either as gases or in liquid form, can be used as sterilants.
While the use of gas and liquid chemical sterilants avoids the problem of heat damage,
users must ensure that article to be sterilized is chemically compatible with the sterilant
being used.
In addition, the use of chemical sterilants poses new challenges for workplace safety, as
the properties that make chemicals effective sterilants usually make them harmful to
humans. Ethylene oxide, formaldehyde, ozone beta propiolactone are some examples.

180
Annexure 5 Drug regulatory act and schedules
The Government of India has promulgated Drugs and Cosmetics Act long before to regulate the
import, manufacture, distribution and sale of drugs. However, subsequent amendments for this
act have been passed from time to time as per the requirements. The following drug schedules
are important for physicians which are prescription elated schedules.

1. Schedule H (prescription drugs)


Drugs included in this schedule are sold by retailers only on production of valid
prescription given by a registered medical practitioner.

2. Schedule W
This includes only few drugs that shall be marketed under generic names only. For
example:, aspirin, chlorpromazine, ferrous sulphate and piperazine.

3. Schedule X
Drugs having dependence liability are included in this schedule. Strict directions are
given regarding labelling, prescription, storage and sale of these drugs. These drugs
must be stored under lock and key in a safe so that only responsible persons will have
access. Importantly, the supply of these drugs has to be maintained and recorded in a
register.

4. Schedule G
This provides list of drug preparations with a label that states “Caution: It is dangerous
to use this preparation except under medical supervision”.

5. Schedule J
This is an informative schedule which gives the list of ailments for which no drug should
claim prevention or cure. For example HIV and atherosclerosis.

6. Schedule K
This defines the conditions under which certain circumstances registered medical
practitioners and hospitals are exempted from provisions as given in Chapter IV of
Drugs and Cosmetics Act 1940 of India.

7. Schedule C
This appraises the clinical values of biologicals and other special products. For example
vaccines, sera, insulin and antibodies.

8. Schedule F
This depicts specifications for standard ophthalmic preparations.

181
Annexure 6 Causality assessment – WHO scale

A clinical event, including laboratory test abnormality,


occurring in a plausible time relationship to drug
administration, and which cannot be explained by concurrent
disease or other drugs or chemicals. The response to
1.CERTAIN
withdrawal of the drug (dechallenge) should be clinically
plausible. The event must be definitive pharmacologically or
phenomenologically, using a satisfactory rechallenge
procedure if necessary.
A clinical event, including laboratory test abnormality, with a
reasonable time sequence to administration of the drug,
2.PROBABLE/ unlikely to be attributed to concurrent disease or other drugs
LIKELY or chemicals, and which follows a clinically reasonable
response on withdrawal (dechallenge). Rechallenge
information is not required to fulfil this definition.
A clinical event, including laboratory test abnormality, with a
reasonable time sequence to administration of the drug, but
3.POSSIBLE which could also be explained by concurrent disease or other
drugs or chemicals. Information on drug withdrawal may be
lacking or unclear.
A clinical event, including laboratory test abnormality, with a
temporal relationship to drug administration which makes a
4.UNLIKELY causal relationship improbable, and in which other drugs,
chemicals or underlying disease provide plausible
explanations.
A clinical event, including laboratory test abnormality,
5.CONDITIONAL/ reported as an adverse reaction, about which more data is
UNCLASSIFIED essential for a proper assessment or the additional data are
under examination.
A report suggesting an adverse reaction which cannot be
6.UNASSESSIBLE/
judged because information is insufficient or contradictory,
UNCLASSIFIABLE
and which cannot be supplemented or verified.

182
Annexure 7 – Causality assessment Naranjo Scale

Don't
Question Yes No
know
Are there previous conclusion reports on this reaction? +1 0 0
Did the adverse event appear after the suspect drug was
+2 -1 0
administered?
Did the adverse event improve when the drug was
+1 0 0
discontinued or a specific antagonist was administered?
Did the adverse event reappear when drug was
+2 -1 0
readministered?
Are there alternate causes [other than the drug] that
-1 +2 0
could solely have caused the reaction?
Did the reaction reappear when a placebo was given? -1 +1 0
Was the drug detected in the blood [or other fluids] in a
+1 0 0
concentration known to be toxic?
Was the reaction more severe when the dose was
+1 0 0
increased, or less severe when the dose was decreased?
Did the patient have a similar reaction to the same or
+1 0 0
similar drugs in any previous exposure?
Was the adverse event confirmed by objective
+1 0 0
evidence?

SCORING FOR NARANJO's ALGORITHM

> 9 = definite ADR


5-8 = probable ADR
1-4 = possible ADR
0 = doubtful ADR

183
Pharmacology record notebook – Blank page for rough use

184
Pharmacology record notebook – Blank page for rough use

185

You might also like